MENU
未分類

第95回日本社会学会大会 11/12(土)15:00~18:00 報告要旨

報告番号124

ペット飼育の主体者間のエンゲージメント
早稲田大学 藤田 典子

【1、目的】本報告では、「ペット共生社会論」の視点に基づき、日本社会においてペット飼育に直接関わる様々なアクター(ペット飼育の主体者)が、他者との関係性をより良好に保ち、ペットが暮らしやすい社会を形成するためにとっている行動について、質的インタビュー調査から得られたデータを用いて考察する。 【2、背景】第94回大会テーマセッションの徳田報告では、「ペット共生社会論」の視点から、ペット飼育を人と人との関係にとらえなおすことの重要性が説かれた。特に社会空間において、ペットオーナーと非オーナーの間や、ペットが好きな人と嫌いな人、あるいは無関心な人との間で、共生が図られるための技法について議論された。「家族ペット」(山田 2004: 189-94)で提起されたペットを取り巻く様々な社会問題に関して、今日もさらなる研究の蓄積が求められている。そこで本研究は、コロナ禍で、新たにペットとして迎えられた頭数が増加(ペットフード協会 2020)した社会環境の中で、ペット飼育の主体者の意識と行動に着目し、共生社会に向けた人々の歩みを探った。 【3、方法】2022年6月までに、都市部にて、ペット(犬・猫)飼育の主体者(ペットオーナーや獣医師、動物専門学校教員、トレーナー等)に平均1.5時間のインタビュー調査を実施した。なお報告者も、ペットオーナーであり、主体者の1人である。 【4、結果】これまでの調査対象者に共通していたことは、以下の三点である。第一は、対象者の全員がペットを取り巻く社会問題を何らかの形で察知し、敏感に反応していたことだ。第二に、その社会問題は、すでに先行研究が論じてきたように、公的空間における人と人との間で生じていた。第三の共通点として、数々の問題を体験した対象者は、その問題に主体的に抵抗し、より良い道を追い求め、行動していた。例えば、犬・猫の販売・譲渡業者は、新たにオーナーとなる人に対し、ペット飼育で必要なこと(ワクチン接種やノミダニフィラリア薬の投与、ペット保険への加入、クレートトレーニングを含む基本的なしつけ等)を入念に説明した。獣医師や看護師は動物病院で無料のパピーパーティを開催し、犬の社会化の支援をしたり、診察時に犬・猫の行動や持って生まれた性格について親身に解説したり、死を迎える際に治療だけでなく精神面で寄り添ったりしていた。動物関連企業や愛護センターで働く人は、自前のツールを利用し、適正な飼育方法ついての情報発信や啓発活動を行なっていた。動物トレーナーは犬・猫の問題行動に正面から向き合い、オーナーの相談相手になっていた。各オーナーは、熟練になれば新規のオーナーを直接指導したり、動物関連業者へのクチコミを通じて善良な業者の情報を拡散する役割を担っていた。以上のように、本報告でのペット飼育の主体者は、それぞれがペット共生社会の行為者として、お互いに対話し、支え合って「エンゲージ」(大倉 2020: 193)していることが分かった。今後は、この主体者間のエンゲージメントが、公的空間において非オーナーやペット嫌い、無関心の人にどのような影響を及ぼしているか、多情・多恨の関係の解消に貢献しているか等にも注目することを検討している。

報告番号125

現代日本における社会的カテゴリーとしての〈猫〉に関する一考察
日本大学 木下 征彦

1.目的  本報告の目的は,地域社会における人と猫をめぐるコンフリクトを,猫という生物に特有の問題として捉えなおすことである.報告者は過去の報告で,野良猫問題とその解決過程としての地域猫活動を地域社会における人と猫をめぐるコンフリクトとして捉え,その問題構造と問題構築過程の分析を試みた.これらを通じて猫をめぐる問題を社会学の視点と方法によって地域問題として位置けたが,猫に特有の問題としての側面については未だ掘り下げの余地が残っている.そこで本報告においては現代日本社会における猫の位置づけを幅広く検討する. 2.方法  まず(1)報告者自身のこれまでの研究成果を整理の上,猫を社会学的に捉える視点と方法を示す.続いて(2)猫に関する諸研究・データを用いて主に法的・政治的ならびに社会・文化的な側面から猫の位置づけを検討し,現代日本社会における社会的カテゴリーとしての〈猫〉の特性を記述する. 3.結果 猫をめぐる問題においては,猫の生物学的特徴から住宅密集地における糞尿等の生活被害を戸別に防止することは現実的に困難である.したがって猫を広く社会的な観点から捉える必要がある. 日本社会において,猫は法的には動物愛護管理法の対象となる愛護動物として位置づけられ,鳥獣保護法の対象となる野生動物と区別される.しかし実際には愛護動物である野良猫と野生動物であるノネコの区分は不明瞭であり,便宜上の区別にすぎないことがわかる.また,動物の「所有者又は占有者」には所有権とともに愛護と管理の責任がともなうが,長い間,外飼いが当たり前とされていた猫の場合,誰に所有権があるかを特定することが難しい.こうした所有や管理責任の所在は猫をめぐる問題における難しい論点の一つであるが,現行の行政施策でここに踏み込んだものは少ない. 社会・文化的にみると,これまで猫はさまざまな位置づけがなされてきた.近代以前,猫は信仰の対象として祀られる一方で恐怖の対象としても語り伝えられている.また古今,猫はしばしばネズミなどを駆除する益獣とされるが,時には生態系や社会生活に被害をもたらす害獣とみなされる.とはいえ,猫をめぐる問題はいわゆる獣害問題と同一視されることはほとんどなく,野良猫の駆除をめぐっても明らかに野生動物とは異なる一線が引かれている.現代日本における猫は,市場経済における消費財として時に遺棄・処分の対象とされつつも,コンパニオンアニマルとして人々に寄り添いながら愛情を向けられる対象となっている. 4.結論 日本社会における猫の位置づけは,政治経済文化と結びついて社会的に構築されたものである.あえて単純化するならば,猫をその社会でどう扱うかは,当該社会のメンバーによって決まるといえる.したがって,地域社会における人と猫をめぐるコンフリクトにともなう困難――猫が愛護の対象となっていること,法的な境界上に存在すること,そしてその管理責任の所在が不明瞭なこと――は,これまで共有されてきた社会的カテゴリーとしての〈猫〉に特有の性質といえる.この論点に向き合って〈猫〉をめぐる問題の解決を目指すためは,これまで自明視されてきた社会的カテゴリーをどのように正当化できるか,あるはその枠組みを再考するかについて,公共的な議論を喚起する必要があろう.

報告番号126

猫・人間・自然の関係の変容をめぐって
国際ファッション専門職大学 東 由美子

本報告では、猫・人間・自然の関係の変容という点から、「猫社会学の応用と展開」というセッションテーマへの接近を試みたい。まず、「猫社会学」に関連する報告者のこれまでの研究の概要と関心について述べ、次に本報告で問題としたいことがらについての方向を提示する。 報告者は、2004年に刊行した単著書『猫はなぜ絞首台に登ったか』において、ロバート・ダーントンが、論文「労働者の叛乱―サン・セヴラン街の猫の大虐殺」(『猫の大虐殺』所収)の中で用いた資料を検討し直し、資料の中に登場する「猫」の意味について、ダーントンとは異なる解釈を呈示した。ダーントンが用いた資料は、1762年にパリの印刷工ニコラ・コンタが書き残した『印刷業界こぼれ話』の一節であった。そこには、猫の虐待/猫の愛玩という正反対する態度を有する印刷工たちと親方夫婦(ブルジョワ)が登場し、印刷工たちが多数の猫を虐殺するエピソードで締めくくられる。ダーントンは、このエピソードに、印刷工と親方夫婦との階級対立を読み取り、物語の中で印刷工に殺される「猫」とはすなわち、印刷工たちに比喩的な意味でレイプされた親方の妻を意味すると考えたのである。これに対し、報告者は、コンタの物語に加えて、ダーントンも論文中で触れている画家ウィリアム・ホガースが動物を虐待するロンドンの人々を描いた銅版画『残酷の四段階』(1751年)、さらにはフレイザーの『金枝篇』などの記載も参照しながら、社会構造が農村共同体型から都市型へと変化しつつあった18世紀半ばのヨーロッパの大都市で生じた印刷工たちの猫殺しに、穀物霊の供犠という、古代から続く神話的な思考の片鱗を読み取った。 産業革命前夜である18世紀半ばのヨーロッパの大都市は、動物虐待から動物愛護へと、人が動物に抱く観念の転換期にあったと考えられる。ただし、18世紀半ばという時代のロンドンやパリでは、動物愛護の思想を有する者、動物愛護の姿勢を公的に表明する者の数は少なく、民衆の多くはまだ神話的な思考を有しており、動物に対して残酷な態度を取っていたとみなすことができる。 さて、猫に残酷なふるまいをしても、とりたててとがめ立てされなかった時代から時を経て、場所を変え、現代の日本は、動物愛護の観念がほぼ浸透してきているとみなしうる。その象徴が現代日本における「猫」ブームであろう。猫に対する態度や猫に対する観念の決定的な変容の徴候は、日本では1980年代半ば頃から見られるようになったと報告者には思われる。以上の経緯を踏まえ、本報告では、現代日本における猫と人間と自然という関係の変容の一端に触れ、あわせてその変容が示す意味について考えてみたい。 【関連文献】ロバート・ダーントン1990「労働者の叛乱―サン・セヴラン街の猫の大虐殺」、『猫の大虐殺』海保真夫・鷲見洋一翻訳、岩波書店(同時代ライブラリー)。 森洋子編著1981『ホガースの銅版画―英国の世相と諷刺―』岩崎美術社。 東ゆみこ(筆名)2004 『猫はなぜ絞首台に登ったか』光文社新書。

報告番号127

猫からみる現代インド社会の変容――試論的考察
東京外国語大学 栗田 知宏

【1. 目的】本報告では、猫と人とが取り結ぶ関係性からみた現代社会のありようを、インドにおける猫の社会的位置づけという事例から検討し、猫社会学の今後の展開に国際比較の視座を付け加えたい。インドでは、動物といえばヒンドゥー教にとって聖なる意味を持つ牛や、家畜としての水牛や山羊、羊などが人間の生活にとって比較的身近で、動物をペットあるいはコンパニオンアニマルとして飼う習慣は一部の富裕層を除いて限られてきた。また、野良猫は野良犬に比して数が非常に少ないのに加え、動物が媒介する感染症などの衛生上の問題から避けられやすい。こうした背景から、猫の社会的な存在感は大きいとは言い難く、日本における「猫ブーム」とは対極的な状況にある。しかし、インドの著しい経済発展は、中間層の拡大や欧米的な消費文化をもたらし、経済的豊かさを享受する人々が犬猫をペットとして飼う例が近年増えてきた。これを都市中間層の「癒しブーム」の一例として捉える見方もありうるが(中島 2006)、ペットケア市場の拡大がもたらす社会へのインパクトや人々の動物観の変化など、そこには多くの議論の余地がある。【2. 方法】本報告では、インドのメディア報道の内容検討および一般のインド人数名へのインタビュー調査から、インドの人々の猫に関する意識の一端を探る。近年のインドの社会変容は、人々の猫に対する印象やペット飼育という新たなトレンドへの意識とどのように関連づけて捉えることができるだろうか。【3. 結果】予備調査からは、猫をペットとして愛でる習慣は一般的にはまだかなり少ないことがうかがえる。周りに例があまりないため、経済的な余裕があってもペットを飼うという発想自体が生じにくいといった理由もあろう。一方で、猫に関する意識にアプローチする上では、居住地域、家族形態、社会階層、住環境、気候、動物にまつわる浄/不浄の観念、動物への宗教的な意味づけ、動物病院やペットケア産業へのアクセスなど、様々な要素を考慮する必要がある。報告ではこれらに目配りしながら、より詳細な検討を試みる。【4. 結論】暫定的な結論としては、家族形態の変化などを背景とした猫の「家族」化や、人間の自然との関わりに対する欲求(バイオフィリア)を満たすものとしてのペットという位置づけ(養老・的場 2008)は、インド社会の一部においてもある程度まで見出すことができそうである。また、コロナ禍におけるペットブーム後の飼育放棄といった日本と共通の課題もある。その一方で、インドという文脈からの考察を通じて、日本社会と猫との関わりの「特殊性」も浮かび上がってくるだろう。【参考文献】中島岳志,2006,『インドの時代――豊かさと苦悩の幕開け』新潮社.養老孟司・的場美芳子,2008,「動物は自然――ペットからコンパニオンアニマルへ」森裕司・奥野卓司編『ヒトと動物の関係学 第3巻 ペットと社会』岩波書店,102-30.

報告番号128

ペットの家族化の進展とその帰結
中央大学 山田 昌弘

報告者が、「ペットを家族とみなして関係を築くこと」、すなわち、ペットの家族化を指摘し出版したのは、1989年『ジェンダーの社会学』(江原由美子他)の一節「家族の定義をめぐって-ネコは家族か?」の中においてである。そこでは、家族の範囲が曖昧になり「主観化」していること論じ、ペットを家族として扱う人々が出現していると論じた。年配の研究者からはふざけるなと面罵されることもあったが、それ以降、大規模な質問紙調査を行う度に、「ペットを家族と思うか」「ペットと親密な関係の有無」を質問紙に加えており、おおむねその割合は上昇している。  この30年間の間に、ペットの家族化は日本社会に浸透し、受け入れられているように見える。ペットを家族のように扱うとは、代替不可能な存在として、その幸福に責任をもち、親密な感情を抱くという近代家族が設定する家族概念に合致した存在として、ペットが選ばれていることを意味すると暫定的に定義しておく。  ペットの家族化が進む背景には、「家族形成の困難」と「親密性の変容」という二つのトレンドがある。  1990年代以降は、未婚化、少子化、離婚増大、高齢化が進む。その中で、標準的な核家族(夫婦+未成年の子)を形成し維持していくことが困難な人々が減少する。報告者が2003年に実施した「ペットを家族とみなす人々」9名に対するインタビュー調査では、ペットをまるで恋人や子どもであるかのような語りがあった。未婚者、離死別独身者、子どもがいない夫婦、子どもが巣立った高齢夫婦が増えている中、その足りないと思われるう部分を埋めるような形でペットが入り込んでいることを示唆している(山田2004『家族ペット』サンマーク出版)。  次のトレンドは、親密性の変容である。ギデンズは、制度的な役割関係であったパートナーシップ関係は、現代化と共に「純粋な関係性」に置き換わると論じた(A. Giddens1992 ‘ The Transformation of Intimacy’)。しかし、野口裕二は、共同性と親密性を分け、日本社会では家族関係は必ずしもギデンズの言うような親密性を必要としないと論じる(野口野口2013「親密性と共同性」庄司洋子編『親密性の福祉社会学』)。家族関係では十分に満たされない親密性の対象の一つとして、ペットが選ばれているという側面がある。  標準的家族を形成し親密な関係を築いている人でもペットの家族化が生じているわけで、そうではない人との関係性の違いがある可能性もある。  更に、ペットを家族と見なす人の増大によって、社会の中でのペットの位置づけの変化も生じている。ペットにかかる手間やお金が増え、「ペットを飼える層」と「飼いたくても飼えない層」に別れる兆しもある。これは日本の少子化と同様のロジックが始まっているとも言える。  本報告では、ペットの家族化の実態を、以上の観点から整理するとともに、その現状を質問紙調査の結果をもとに示す。

報告番号129

オーガニック食品の選択と社会階層
立命館大学 安井 大輔

本報告の目的は、どのような社会的な要因が、どのように食品の選択に作用してきたのかという課題を探求するため、食品選択に与える「社会的要因の交互作用効果」について計量社会学的に分析することである。  日々食べるものの選択はいっけん自由におこなわれるようでいて、人びとの属性や所属する諸集団の社会経済的地位に規定されている側面も大きい。食品選択に社会経済的地位による違いがあること自体は、経験的に想像可能であるが、数量データ分析においても実証されてもいる。社会階層の違いがどのように食材選択の違いとして現れているのか全国的な傾向を調べた(安井2021)では、2015年SSMデータを用いて、国産の牛肉や野菜(国産食品)と無農薬・有機栽培・無添加の食品(オーガニック食品)の優先的な購入と属性(性別、年齢、婚姻状態)や社会階層(収入、学歴、職業、階層帰属意識)、社会活動との関係を検証した。結果、男性よりも女性、年配の人ほど、未婚者より既婚者、世帯収入が高いほど、両方の食品をより購入しており、また学歴・職業・居住地域・好んで参加する社会活動によって購入頻度に違いが見られることが観測された。  これらの結果から、食品選択に格差があることは示しうる。しかしながら、この結果は、あくまでそれぞれの社会的要因が食品選択に与える個々の影響について明らかにしたことにとどまる。たとえば、都市よりも地方居住者のほうがよりオーガニック食品を購入しているとしても、大都市居住クリエイティブクラスと地方居住ワーキングクラスではどちらがより購入するのかといったことは十分には明らかではない。食品選択に与える社会的要因は、単独で作用するだけでなく、複数の要因が働きあって作用する。食をめぐる格差の実際はより詳細に検討されるべき問題であろう。  それゆえに、本報告では、これまでの知見に対して、食品選択に与える社会的な効果の組み合わせを分析する。食品選択に与える効果の大きい要因を選出して、それらの変数どうしの交互作用項を投入して、その効果を測定する。この作業を通して、社会経済的な要因がいかに組みあわさって食品選択に影響するのか、というインターセクショナルな効果を明らかにしたい。  もし社会学が、食の実践的な課題にまで応えることを目指すならば、全般的な傾向分析だけでなく、個別の状況に対応しうるような知見が求められるであろう。もし本稿で設定したような課題を一定程度明らかにしえたならば、栄養指導や購入アドバイスなど実践の現場でより具体的な回答を提供することも可能となるかもしれない。 文献 安井大輔, 2021, 「食選択と社会に働きかける活動―国産食品とオーガニック食品の購入をめぐって」『ソシオロジ』65(3): 59-78

報告番号130

食肉代替食品をめぐる社会意識の分析――全国紙を対象としたKH Coderによる計量テキスト分析
九州大学 藤原 なつみ

【1.目的】  食料不足に対する危機感などを背景に代替タンパク質による食品素材開発への注目が高まっている。とくに本研究が着目するのは、見た目や食感、味を現在食べられている食肉に近づけようとする「食肉代替食品」である。食肉代替食品は、環境負荷の低減や動物福祉への配慮など持続可能な食の観点からも期待を集めており、国内外で研究開発が進められている。一方で、「肉を食べる」という食の実践は、宗教、歴史、文化等と深く結びついて社会に定着していることから、「食肉代替食品を食べる」という実践が、今後社会的に受容され定着していくかという点については検討が必要である。社会学者であるショブらによれば、新たな〈実践〉は、〈物質〉、〈意味〉、〈能力〉の3要素が結びつくことで生みだされ、定着していく(Shove et al. 2012)。「食肉代替食品を食べる」という新たな〈実践〉においては、どのように3要素が結びつき、定着する/定着しないのだろうか。本報告では、〈意味〉に着目しながら、食肉代替食品が社会の中でどのように意味づけられ、価値判断されていくのかを検討したい。 【2.方法】  社会意識の探索にあたっては、全国紙の内容分析が一定程度まで有効とされていることから(樋口 2011)、本研究では、『朝日新聞』『読売新聞』『毎日新聞』の全国紙3紙を対象として計量テキスト分析を行うこととした。食肉代替食品をあらわす語として、「培養肉(培養ミート)」「代替肉(代替ミート)」「植物肉(植物ミート)」「植物由来食肉用食品」「Plant-based meat」「大豆ミート(大豆肉)」「クリーンミート」「フェイクミート」を検索対象語として設定し、いずれかの語を含む記事を収集したうえで、KH Coder 3による計量テキスト分析を行った。 【3.結果】  収集の結果、計390件の記事が得られた。その内訳は、「大豆ミート(大豆肉を含む)」169件、「代替肉(代替ミートを含む)」131件、「培養肉」65件、「植物肉」15件、「フェイクミート」6件、「クリーンミート」4件であった。さらに、収集した記事を対象としてKH Coder 3による分析を行ったところ、「大豆ミート」と「代替肉」では、いずれも「使う」と「食べる」が関連語の上位に検出された。さらに、「代替肉」の関連語として「大豆」が上位に検出された。実際の記事をみると、「大豆など植物由来の原料を使い、本物の肉のような味や食感に近づけた『代替肉』市場が拡大している」(『読売新聞』2020.10.7朝刊)といったように、「代替肉」は大豆など植物由来の原料を用いたものとして説明されていた。「培養肉」では、「研究」や「細胞」、「開発」といった語が関連語の上位に検出された。 【4.結論】  「代替肉」という語にはさまざまな含意があるが、現状では「大豆ミート」とほぼ同義で使用されていることが明らかになった。「培養肉」に対しては、研究段階にあるという認識が示唆された。 【引用文献】 樋口耕一, 2011, “現代における全国紙の内容分析の有効性―社会意識の探索はどこまで可能か”, 行動計量学, 38(1),1-12. Shove, E., and M, Pantzar., and Watson, M., 2012, The Dynamics of Social Practice: Everyday Life and how it Changes, SAGE publications Ltd. 【謝辞】 本研究は、公財)味の素食の文化センター、ならびにJSPS科研費(22K13526)の助成を受けたものです。

報告番号131

食実践の変更が社会関係に与える影響
神戸女子大学 大淵 裕美

“近年、コロナ禍を契機として、自身のライフスタイルを見直し、これまでの食生活から異なる食生活に自発的に変更する現象が見受けられる。とりわけ、「筋トレ」や「減量」といった身体を鍛える際には、多かれ少なかれ食事の見直しや変更が伴われる。その他にも、食物アレルギーや糖尿病、摂食障害、肥満などの疾患、妊娠・出産・授乳などの身体状況に応じて、外部から自身の食生活の見直しと編み直しを課せられることもある。つまり、「食事を自分で管理できることは、自負心を高める意志の行為であるが、意志を否定するものとして、外部から課せられることもある」(Turner1984=1999,190)。
こうした食生活の自発的・強制的変更は、食事に関する準拠集団やコミュニティの変更も生じさせる。例えば「美食」が趣味であり、それを楽しむコミュニティに所属していた人が、食事規制を伴う病気を発症した場合、それまで当たり前として無自覚に行われていた食事を常に意識化する必要が生じる。その結果、個人レベルでの食事のみならず、他者と食事を共にする「共食」や、自らの食実践と異なるルールで行われている「外食」の場面において、様々な選択を迫られる。特に、食事を変更する以前に所属していたコミュニティにおいて、食実践を変更した人は少数派=「食のマイノリティ」となる。そのため、これまでの食事に基づく人々との関係性を継続するか否か、もしくは新たな食事の実践を共有する人々との出会いに向けて人間関係を再構築するかについても、選択を迫られることになる。いうならば、「食事は他者を受け容れるだけでなく、「われわれ」と「彼ら」の境界線を明確にし、特定の人間や集団を重要な社会的関係から排除する役目を果たす」(Guptill, Copelton and Lucal 2013=2016:47)のであり、食事は「社会的包摂であると同時に排除にもなる」(ibid:47)のである。
そこで本研究では、食事の変容を経験する際に、人は何を基準にどのような食事の選択を一食一食行っているのか、自身と異なる食に対する価値基準を持つ人々とはどのような関係性になっていくのか、また新たなコミュニティとの出会いをどのように行い、自身の実践を継続したり他者に譲歩したりすることになるのかの解明を目指す。食に関する既存研究を整理しつつ、先鋭的な事例を取り上げ、試論的な検討を行う。
「食生活の選択という日常実践はいかなるプロセスによって人間関係や社会関係の変容をもたらすのか」という本研究の問いは、「社会的世界論」(宝月 2021)の視点から「食のマイノリティ」をとらえる出発点となるだろう。
【参考文献】
Guptill, Amy E., Copelton, Denise A., and Lucal, Besty, 2013, Food and Society: Principles and Paradoxes, Polity Press Ltd.(=2016、伊藤茂訳、『食の社会学――パラドクスから考える』NTT出版)
宝月誠、2021、『シカゴ学派社会学の可能性-社会的世界論の視点と方法』東信堂.
Turner, Bryan S.,1984, The Body and Society, Basil Blackwell Publisher Ltd.(=1999、小口信吉・藤田弘人・泉田渡・小口孝司訳、『身体と文化―身体社会学試論』文化書房博文社)”

報告番号132

食物アレルギー症状を持つ成人の食の課題解決に向けて――消費者のニーズを起点として
法政大学大学院 水野 京子

本稿で論じる食物アレルギー症状のある人々とは,食物アレルギーまたは食物アレルギー様の症状を抱えながら生活をする人々と定義する.子どもの食物アレルギーはすでに社会に広く認知されているが,成人の認知度は高くない.そこで食物アレルギー症状のある成人のケアとサポートのあり方に焦点を当て,持続的な生活の質向上を目指すために,食の市場をその選択肢の1つとして考える.  東京都の世論調査では,満18歳以上の「外食産業に望むこと」の回答のうち,「アレルギー物質の表示」と答えた人が19.5%(東京都生活文化局 2019),また惣菜に関する消費者動向の調査では,惣菜のアレルギー表示を必ず見る人は9.7%(一般社団法人日本惣菜協会 2021)となっている.社会の中にアレルギー症状を持つ人々はたしかに存在している.食物アレルギー症状を持つ人々の悩みとは,カミングアウトの問題や,人の誘いや頂きものを断る罪悪感,好き嫌いという非難,人と同じものを食べられない疎外感などが考えられる.そして,その症状やアレルギーの原因となる食品により,生活への影響も千差万別である.  食物アレルギーに関する社会学的な研究は少ないが,ここで福祉社会学の視点より,食物アレルギー症状を持つ人々のケアへと議論を進める.社会福祉学より,障がいを抱えた人々の自立生活運動を例に挙げると,障がいを抱えた人々が家族のケアに依存した生活から,自立して地域で暮らすために,福祉サービスの充実が進められてきた(庄司1999).一方食物アレルギーでは,医療的なケアと並行して,食物アレルギーの子を持つ親の相談から食品企業が対応を開始し(日本ハム株式会社 2022ウェブサイト閲覧),結果として親の会の活動との連携も行われている.子どもたちがやがて成人した時,また成人になってからアレルギー症状を発症する人々がその生活を障壁なく暮らしていくために,より広義の社会での理解や行政の対応が求められる.それと同様に,食品企業や外食産業におけるの商品の開発や表示義務の遵守,サービスによる対応が生活の課題を解決する一つの方法となる. 医療や福祉の領域を超えて「人」のケアを考える時,多様なケアの実践(三井 2004)が問われている.  それぞれに症状も原因となる物質も異なる,食物アレルギー症状を持つ人々の生活に寄り添った解決方法を考えるには多角的な視点から捉える必要がある.分野を超えて相互に連携し,それぞれの症状やその原因となる食品に応じて幅広い選択肢が提示されることが,その生活の課題を解決する一助となる.そこで,今後はそうした人々の顧客満足度の視点から,消費者主導での製品開発や食品表示などの取り組み(Almeida et al. 2020).が望まれる.アレルギー症状を持つ人々の多くは社会に分散して存在し,困難を抱えながらも症状とつきあいながら生活している.医療にも福祉にも繋がらず,個々が抱える課題に対して,あるべき支援の形にも多様性が必要であり,さまざまな分野から広く知見を集め,支援の方法論が多種多様に模索されることが求められている.

報告番号133

消費標準の歴史社会学――近代日本における消費経済をめぐる「経済的統治」の系譜
日本学術振興会 林 凌

【1.目的】  本発表では近代日本における知識人の議論を分析対象として、国家による経済介入を希求する言説において消費経済の統治がいかに構想されていたのかを、標準化という観点から明らかにする。かつてミシェル・フーコーは、「政治経済学」により可視化される集合的な「人口」を、国家や社会という観点から最適な方向へと導こうとする統治形態を、近代社会において新たに現れた「経済的統治」と捉えた(Foucault 2004a=2007: 143-161)。彼は「生産者と消費者、所有者と非所有者、利潤を生み出す者といったそれぞれ特有の役割」が「理論の領域にも経済的実践の領域」にも導入されることで、新たな国家権力による介入が可 能になったと考えたのである(Foucault 2004a=2007: 93)。  この観点からすれば、近代日本における「政治経済学」を用いた言説空間の形成は、日本国民という「人口」を統御しようとする「経済的統治」の試行過程として考えることができる。近代日本における「人口」をめぐる規範を検討した研究は、フーコーの「人口(Popularité)」概念を、狭義の「人口」概念に限定して解釈してきた(山田 2019)。その結果、本概念が本来「生産者」や「消費者」といった「政治経済学」に依拠したカテゴリーに適用されうるものであることを、先行研究は論じていない。本発表はこの問題を克服しようとするものでもある。 【2.方法】  分析対象とするのは、1900年代から1910年代において、社会政策学会を中心に活動していた経済学者や社会運動家の議論である。特に本発表では、初期社会政策学会の代表的論者である金井延と桑田熊蔵、および第2世代に位置する河上肇の残したテキストに着目する。 【3.結果】  1900年代は、ドイツ歴史学派の知見に基づく経済介入策が、本格的に近代日本の言説空間の中に現れるようになった時期である。特に、この時期の議論がそれまでの議論と大きく異なるのは、殖産興業や貿易政策だけでなく、労働者/消費者としての日本国民の生活状況に対し、具体的に介入しようとする考えが語られるようになったという点にある。つまり、国内における貧富の格差や所得の非均衡が問題化されるとともに、現状の社会構造を維持したまま、この問題の是正を可能とする介入策が求められたのである。  このような状況下において近代日本の知識人は、自由放任状況にある消費を標準化し、いかにして社会全体の効率化に資するものにするかという点を論じるようになる。ただし、「所得分配の正確なる統計は極めて少な」い状況においては、この構想は絵空餅でしかないと、当事者にとっても認識されていた(金井 1902: 864-869)。 【4.考察】  こうした知識人の関心と議論は、経済統計が整備された1930年代以降、後年の研究者に受け継がれることで次第に具体的な政策構想へと移行していき、「総力戦体制」の礎として結実する。重要なのはこの歴史的過程において、知識人の問題意識は一貫として、消費標準化を介した国民全体の生活水準の向上にあったということである。近代日本における消費経済に対する国家の強制介入は、生の剥奪的性質を持つことをしばしば前提される。本発表は、この介入が生の支援的性質も持っていたこと、それが「標準化」と呼びうる考えのもと生じていることを明らかにするものである。

報告番号134

「規格形エレベーター」から集合住宅の居住実践を解読する――戦後日本におけるメーカーの開発過程に着目して
慶應義塾大学大学院 水田 綾奈

【1.目的】本報告は、戦後日本において規格化されたエレベーター(以下、EV)の開発過程を題材に、当時の集合住宅における居住実践がいかにEVの規格に反映され、またEVがいかに居住実践を条件づけてきたのか、その相互関係について考察することを目的とする。規格形EVは、戦後、都市部における住宅不足解消のために設立された日本住宅公団からの要請により、量産性と経済性を追求した標準化が進められ、集合住宅の生活環境に深く溶け込むことを目標に開発された。そしてその発売以降、集合住宅の日常生活やEV利用実態をふまえながら開発・改良が行われた。つまり、規格形EVというテクノロジーは戦後の集合住宅という社会との「相互作用」(Latour 2005=2019)の中で登場し変化したモノであり、それらの相互関係を辿ることで、EVがいかに集合住宅の生活をまなざしていたかを理解しうる。それにより、居住という文化的実践を「モノ」の水準から検討することを可能にする。【2.方法】規格形EVの開発に関して、メーカーによる技術誌(『三菱電機技報』、『日立評論』)や日本エレベーター協会による業界誌(『エレベータ界』)、その他技術論文や建築雑誌を参照し、開発者の意図や議論の内容、仕様の背景を分析した。【3.結果】上記の資料から、開発側の集合居住に対する認識とそれをふまえたEV規格の検討に関する言説を分析すると、以下の変化が読み取れる。規格形EVが登場した1960年代では、EVのかごの大きさやドアの形状といった仕様の決定に際して、居住者世帯の平均人数を3.5人と捉えた上で、「ピアノ」や「ダブルベッド」、「乳母車」といったモノの運搬を想定して規格が設計されていた。その後EVの普及が進んだ1970年代では、子どもによる「EV遊び」が利用上の問題として語られ、その対応として制御装置やドアの開閉装置の改良、非常ボタンのボックス化などが打ち出されていた。そして1980年代以降は、家族の在り方や子どものいたずらに対する言説は後景化し、「防犯」「防災」として、ガラス窓の設置や有効なシステムの開発に取り組む記述が前景化した。【4.結論】以上より、規格形EVの開発過程から集合住宅における居住実践を考察すると、①高度成長期における集合住宅は核家族世帯が中心であり、「ピアノ」や「ダブルベッド」に象徴されるような西洋式のライフスタイルがめざされていたこと、②そうした生活の中で、子どもたちはEVを閉鎖的な集合住宅における数少ない「自由空間」(松田 1988)の1つとして利用していたこと、③その後、システムの改良や利用者への規律訓練によってEVと居住者の相互作用が変化すると、EVそして集合住宅では犯罪や災害に対する不安やリスクが浮かび上がるようになったことが結論として導かれる。 参考文献)Latour Bruno,2005,Reassembling the social: An introduction to Actor-Network-Theory, Oxford: OUP.(伊藤嘉高訳,2019,『社会的なものを組み直す アクターネットワーク理論 入門』法政大学出版局.)/ 松田道雄,1988,『わが生活 わが思想』岩波書店.

報告番号135

レンタルビデオ店の大規模化と標準化――TSUTAYAを可能にした技術に着目して
大妻女子大学 近藤 和都

1 目的 本報告では、1980年代から2000年代にかけて映像文化のインフラとして機能したレンタルビデオ店を取り上げ、その大規模化と標準化のあり方を、TSUTAYAチェーンの形成プロセスとそれを可能にした技術から論じていく。レンタルビデオ店は初期においては個人経営の店舗が多く、店舗ごとの「個性」が収集ソフトのリスト、分類方法、価格設定等に表れていたものの、競争が激しくなるにつれて店舗の大規模化が必須となり、大規模資本を投下可能な企業が市場を席巻するようになる。その過程で、特にTSUTAYAを中心としてフランチャイズ・チェーン化(FC化)が進み、標準化された経営方法が導入されるようになった。それはレンタルビデオ店と利用者の関係にも影響を与えるが、具体的にどのような条件のもと大規模化・標準化は可能になったのかを考えたい。 2 方法 大規模な店舗は、大量のソフトの動きを管理・記録し、分析するための技術がなければ効率的に運用できない。また、大量のソフトは無秩序に提示されると、利用者に対して不自由さをともなう経験をもたらす。つまりTSUTAYAのような大規模な店舗を多数擁するFCが成り立つためには、大量のソフトをめぐるさまざまな技術の利用が欠かせない。こうした観点から本報告では、1980年代に大手レンタルビデオ店が電機メーカーと協力しながら開発していったPOSシステムや、秩序だった店内レイアウトを可能にする分類システム、大量のソフトを供給可能にする経済的条件等に関する言説を渉猟し、分析を行う。   3 結果 TSUTAYAなどのFC化を展開した企業は、膨大な個人情報を特定の基準に基づいて収集・分析することで、大量の選択肢のなかから効率よく各店舗に配置すべきソフトを選別していった。従来、店舗の個性は店長のセレクションによって担われていたが、利用者のデータとアルゴリズムがその基準に取って代わった。また、店舗が大規模化するにつれて、店舗内レイアウトも標準化されていった。従来、ソフトの配置と順序は店舗ごとに異なり、場合によってはあいうえお順すらも導入されておらず、特定のソフトを探す手順は確立していなかった。それが徐々に、ジャンル的な基準で分類されるようになり、またその分類システムがFC内で標準化されることで、利用者はどの店舗でも同じ基準で、効率的にソフトを探せるようになった。 4 結論 上記のように、レンタルビデオ店の大規模化はさまざまな技術を前提として成り立ち、そうした技術を通じていくつかの水準で標準化が進んだ。標準化は、利用者の経験の前提となり、映像文化に対する認識を変容させる。この変化がデジタル・プラットフォーム時代にどのように連続しているのか/いないのかを考えることが今後の課題となるだろう。

報告番号136

計画的陳腐化と社会学的記述
東海大学 加島 卓

標準化や規格化に注目する際、計画的陳腐化をめぐる議論は重要だと思われる。計画的陳腐化は成熟した市場において需要をさらに喚起するための方法と言われており、消費者に商品の買い換えを促すことを可能にしているからである。  例えば、エジソンが商用化した白熱電球の耐用時間は1500時間(1881年)から2500時間(1924年)まで伸びたが、ポイボス・カルテル(1925年)という国際的な取り決めは耐用時間を1000時間に制限して電球を発売してきた。またデュポン社が発明したナイロン(1935年)で製造された女性用ストッキングは実際の耐久性よりも破れやすい状態で商品化されていた。このように計画的陳腐化は市場のニーズを技術的に操作したものだと考えられ、計画的陳腐化に注目することは標準化や規格化がそもそも誰によってどのように実践されているのかを検討することにもつながる。  そこで本報告では、まず計画的陳腐化と呼ばれる議論がいかなるものであるのかを整理したい。次に、従来の社会学が計画的陳腐化をいかに論じてきたのかを整理したい。そして最後に近年の事例に注目し、社会学が計画的陳腐化をどのように記述できるのか、その可能性を検討してみたい【目的】【方法】。  計画的陳腐化に関する議論はJ・ジョージ・フレデリックの「漸進的陳腐化の原理」論文(1928年)にまで遡られることもあるが、多くの人に知られるきっかけとなったのは、ヴァンス・パッカードの著書『浪費をつくり出す人々』(1960年)である。  パッカードによると、計画的陳腐化は①機能の陳腐化、②品質の陳腐化、③欲望の陳腐化の三つに区別される。機能の陳腐化とは、より良い機能をもった新製品を導入することで現行品を旧製品にすることである。品質の陳腐化とは、比較的短い期間で製品が壊れるか、部品が消耗するようにすることである。欲望の陳腐化とは、機能は同じでもスタイルを変えることで心理的に現行品を古く感じさせることである。  こうしたなか、社会学がしばしば論じてきたのは③欲望の陳腐化である。なかでも消費社会論はJ・K・ガルブレイスの限界効用逓減の理論に対してデザインやモードによる限界差異の創出を位置づけ、この社会は欲望を自ら創出し、維持し、操作していると論じた。「美術と色彩の部門」に大幅な権限を与えたゼネラル・モーター社によるモデル・チェンジが実用重視のT型フォードの生産停止を決意させたという事例(1927年)は、リースマンに始まり多くの社会学者が言及するエピソードである。  ところがこうした記述は三つのバリエーションを持つ計画的陳腐化論の一つに注目しているにすぎない。実際のところ、ボードリヤールの最初の著作『物の体系』(1968年)もパッカードの三分類を参照したうえで、③欲望の陳腐化が①機能の陳腐化や②品質の陳腐化を飲み込み、その運動自体が固有の目的に設定された点に注目したのであった【結果】。  そこで本報告はこれまで記号やイメージに注目して論じられてきた消費社会論的な計画的陳腐化を踏まえたうえで、イノベーション(技術革新)やマテリアル(材料)にも注目した計画的陳腐化を社会学的に記述する可能性を探ってみたい。具体的には、インクジェット・プリンタやスマートフォンのバッテリーなどに注目して、パッカードが示した三分類の関係が消費社会論の想定とは異なる関係にあることを提起してみたい【結論】。

報告番号137

日本の医療提供システムの今後――COVID-19パンデミックの教訓を踏まえて
防衛医科大学校 金子 雅彦

日本の医療提供システムは原則フリーアクセス(保険証1枚でどの医療機関も受診できるシステム)である。フリーアクセスはパンデミックの状況になると、人々が医療機関に我先に殺到することになるので、医療ひっ迫が生じやすい。2020年からの新型コロナパンデミックではまさにこうした状態となった。これへの対応策の一つは、患者(陽性者)が多数発生しないように人々に行動制限を課すことである。緊急事態宣言やまん延防止等重点措置が医療ひっ迫・崩壊させないことを主な目的として何回も出された。他方、これらは日本社会経済に大きなダメージを与えてきた。人々に行動制限を課す以外の対応策を検討する必要がある。医療システム内の対応としてはフリーアクセスの見直しがある。本報告では、こうした動き(かかりつけ医機能が発揮される制度整備など)を家庭医制度の確立している国(イギリス)と比較しながら検討する。

報告番号330

K防疫の自負心がCOVID-19の予防行為に与える影響
ヤン・ジュンヨン

COVID-19の世界的な流行は、各国の大衆が自国の防疫体系を認識・評価する契機となった。韓国社会においては、防疫体系が高評価され、国家ブランド化する特異な現象が見られたが、2020年3月頃に初めて登場した「K-防疫」という言葉は韓国人の間で瞬時に流行し、国家自負心の主要な根拠となった。本研究は、韓国人のK-防疫に対する自負心が、実際にCOVID-19の予防行為を刺激したかについて、健康信念モデルに基づいて確認しようとしたものである。分析は2020年8月に収集された「COVID-19危険認識と予防行動の調査」における成人927名を対象に進められ、相関分析、一元配置分散分析(One-way ANOVA)、構造方程式モデリング(SEM)等を実施した。分析の結果、20代、男性、低学力者において防疫自負心が相対的に低く表れ、予防行為については男性の取り組みが女性より少ないという状況が見られた。各種共変数を統制した状態において、防疫自負心は認知された恩恵と認知された障害を経路として予防行為に影響を与えており、認知された恩恵と認知された障害は防疫自負心と予防行為間を完全媒介していた。一方、研究参加者を平均以下防疫自負心集団と平均以上防疫自負心集団とに分類した際、平均以下防疫自負心集団においては保健当局による頻繁な介入の試みを危機状況の深化として認識し、かえって保健当局に対する信頼を低下させる蓋然性が見出された。このような研究結果は、防疫自負心が低い集団に対してより効果的な介入メッセージを開発する必要性を提起すると同時に、防疫体系に対する自慢と同様に自虐もまた大衆の健康に悪影響を与えうることを示している。

報告番号138

新型コロナウィルスによる海外渡航/入国制限は韓国の海外(日本)就労希望者にどのような影響を与えたのか
名古屋産業大学 松下 奈美子

新型コロナウィルス(Covid-19)の世界的な流行は、ヒトの国境を超える移動にこれまでにないほどの影響を与えた。世界各国が厳しい出入国制限措置を取り、日本と韓国の間でも国境を越える移動が一時的にほぼ不可能になった。
2000年代以降、韓国政府は自国の高度人材の海外就労を支援し、来日する韓国人人材は増加傾向にあった。高い能力や学歴など豊富な人的資本を持つ人材は、国境を越える際の制度的障壁は低いとされてきた。また情報通信技術の発展により、ヒトや情報の物理的な移動は大幅に軽減されたはずだった。しかし、Covid-19はそうした前提をことごとく覆した。
Covid-19の感染拡大を防ぐという各国の最優先政策による、物理的に“移動させない”、“移動できない”という状況は、高度人材の移動/就労にどのような影響を与えたのか。韓国の海外就労支援政策(K-MOVE)担当者や、来日韓国人人材を雇用するIT企業担当者らへのヒアリングをもとに、移動制限下における日韓間のヒトの移動について考察する。

報告番号331

韓日コロナ期における離婚現象から見る社会変化
釜慶大学校 中国学科 イェ・ドングン

2020年4月、日本のある女性がブログにコロナ離婚に関する内容を書き込み、日本のメディアで爆発的な注目を浴びた。コロナが世界へ拡散し、「国家封鎖」「都市封鎖」によって形成された狭い空間で家族が共同生活するなかで生まれる各種葛藤がCOVID-19と結びつき、「コロナ離婚」は世界的に注目されるようになった。
しかし、2020年下半期へ入り、韓国、日本等の国家においてコロナの統制と日常がある程度回復し、離婚関連統計資料も一定程度蓄積されてくると、COVID-19によって「離婚率」が爆発的に上昇したことが明らかではなく、ある国家では離婚率がある程度かえって低下し、コロナが夫婦関係に肯定的な影響を与えたとする議論も韓国のニュースで提示されるようになり、日本の一部学者らも懐疑論を提起した。
それでは、果たして「コロナ離婚」の実態とはどのようなものか。その実態がどうであれ、コロナ期に離婚の巨視的な環境変化が生じ、離婚に影響を与える諸要因が再発見されたといえる。疾病、世代、都市がそれに当たる。これらについてコロナ期以前と明確な差異が見られるという点は明らかである。
離婚に関する研究、より正確に言えば「離婚率はなぜ上昇するのか」に関する研究において、研究者らは産業化、都市化を最も重要な巨視的要因と見ている。また、急速な経済環境の変化に比べた「家父長(伝統?)的文化の遅滞」が挙げられる(クァク・ベヒ,2002; チェ・ジュンファン, チェ・ジョンシク,2016; 孟秋丽,2000;. 孙晓娟, 陈维涛, 赵东红,2012)。
本研究では、コロナ期における韓中日の離婚率の比較から見出されたいくつかの現象に注目し、離婚率を説明できる新しい強力な変数を探る。「伝染病災難」時期に離婚率の曲線パターンが変化している。コロナ流行期において離婚率に最も大きな影響を与えるものは何か。しばしば疾病(恐怖)、隔離(空間・場所変化)が注目されるが、離婚率統計の統制方式の変化にも注目する必要がある。離婚率の変化を見る際に、韓国の場合、秋夕(チュソク)離婚の減少という新しい変化が生じている。日本でも離婚原因の順位に変化が見られており、韓国と日本において国際結婚家族の離婚率が低下している傾向も見られた。

報告番号139

Institutional agency of Russian medical professionals in the context of COVID-19 pandemic
Tempere University BOROZDINA Ekaterina

“The COVID-19 pandemic has constituted a major challenge to healthcare systems across the globe. The development of biomedical knowledge about new infection plaid crucial role in handling this challenge. However, public health scholars repeatedly underscore that efficiency of the response to the pandemic was also conditioned by socio-political and institutional factors that mediated implementation of scientific findings on the level of clinical routine (Greer et al 2020). As comparative research on institutional responses to the pandemic proliferates, the cases of some countries appear to be particularly puzzling. In this paper, I focus on one of them – the case of Russian Federation.
Russia has experienced one of the most severe impacts from the pandemic among the states with developed healthcare systems. According to the WHO data, in July 2021 Russia was leading the list of European countries with the highest COVID-19 mortality rates (3,7 deaths per 100 thousands people per week), and was also the world leader in yearly excess mortality (Karlinsky, Kobak 2021). According to some accounts, in May 2020 Russia was also characterized by the world highest COVID-19 mortality rate among healthcare practitioners (https://zona.media/article/2020/05/19/martyrology).
While addressing this sinister empirical puzzle of exceptionally high mortality rates in Russia, this paper explores how Russian medical professionals have been coping with COVID-19. It analytically contributes to two fields of social science research. First, it adds to the social science literature on managing crises and disasters. The existing works on the topic usually explore structural factors – policy response, institutional regulations, and infrastructures – to unpack the social aspects of emergency (see for e.g. Perrow 2011, Lakoff 2019). By relying on qualitative interviews with medical professionals, this study adds to this discussion by providing an account of how pandemic has been experienced and treated at the level of hospital routine. As the contradictions of different regulatory logics constitute a characteristic feature of the ever-reforming Russian healthcare (Mateveev, Novkunskaya 2021), this case is particularly instructive for depicting the formation of emergency regime under conditions of institutional inconsistency. It also allows for grasping (a somewhat compelled) institutional agency of the ground-level actors that unravels as they attempt to navigate through both the uncertainties of pandemic situation and formal institutional regulations.
Second, this paper builds into the balk of literature on social contingencies of evidence-based clinical practice by exploring how new knowledge about COVID-19 has been enacted within post-socialist healthcare. In 1990’s on the wave of internationalization evidence-based medical paradigm has entered Russian healthcare. While its principles were accepted as the gold standard on the level of medical discourse, researchers still express doubts if this approach can be consistently incorporated in post-Soviet clinical practice due to excessive bureaucratization and lack of resources within healthcare system (Geltzer 2009, Carroll 2011). This paper contributes to this discussion by exploring strategies of Russian physicians who have strived to implement the cutting-edge medical knowledge about the new infection despite institutional constrains and lack of professional autonomy (Saks 2015).
On the methodological level, the paper relies on social constructivist approach to pandemic. The paper considers pandemic not only as an obdurate event that disrupts normal way of life, but also as a phenomenon that acquires the status of emergency within the realm of social action and interaction. While not underestimating the actual consequences of the pandemic, I explore how the COVID-19 infection is enacted as an emergency when being refracted through actions and understandings of medical professionals as social actors. The empirical base of the research consists of 52 semi-structured qualitative interviews with Russian medical professionals. These interviews were in 2020 during the first wave of pandemic. The majority of the informants were from the biggest Russian cities of Moscow and St. Petersburg, however, the lesser amount of interviews was also collected with medical practitioners from small towns and settlements.
By exploring the entanglements between the introduction of evidence-based standards regarding COVID-19 treatment and post-socialist medical professionalism, this paper singles out physicians’ efforts to reconcile EBM paradigm with organizational constrains as an indicative instance of professionals’ ground-level institutional agency. I define the following compounds of such agency (1) selective application of guidelines and use of foreign (mostly, Western European) clinical recommendations; (2) establishing local professional solidarity; (3) developing relations of personalized trust with the patients.”

報告番号140

Homeostasis amid COVID-19? Changes in social network types among older adults before and during the COVID-19 pandemic
Duke-NUS Medical School SUNG Pildoo

“Background.
Less is known about how older adults’ social networks changed during the COVID-19 pandemic. Older adults’ social networks are expected to contract during the pandemic period, due to the COVID-19 restrictions and social distancing measures. However, it remains uncertain whether the COVID-19 pandemic is solely attributable to social network contraction among older adults. Socioeconomic selectivity theory and social convoy models propose social network concentration in later life. Older adults focus on a few meaningful relationships as they age, while losing social ties due to retirement and health decline. Recent studies, however, have shown that older adults cultivate a new tie when they lose one. In turn, homeostasis in social networks tends to be maintained in a long run: sequential gains and losses of social ties offset each other, leading to stability in older adults’ social networks over time. According to these perspectives, changes during COVID-19 should be closely related to changes prior to the COVID-19 pandemic. It is thus imperative to compare changes in social networks before and during COVID-19 to have a holistic understanding of pandemic-induced changes in later life social networks.
Additionally, during the COVID-19 pandemic period, changes in social networks may not uniformly occur among different components of social networks. For instance, decline in social engagement due to the COVID-19 restrictions does not necessarily accompany with decrease in phone or online contact with family and friends among older adults. This necessitates a typology approach in social networks, which identifies distinct types of social networks, such as diverse, family, friend, and restricted types, by using a set of network indicators such as marital status, family/friend contact, and social engagement. This study thus explored a pattern of changes in social network types prior to and during the COVID-19 pandemic.
Lastly, factors associated with changes in social network types during COVID-19 should be examined, controlling for changes in social network types before COVID-19. This is to identify older adults who are more vulnerable to the COVID-19 pandemic concerning their social networks, net of their prior social connectedness. Specifically, studies have shown that older adults with poor health are more likely to experience social network contraction over time. This study thus tested whether physical, mental, functional, and cognitive health influence changes in social network types during the COVID-19 pandemic.
In summary, the present study identified social network types among older adults; compared changes in social network types among older adults before and during the COVID-19 pandemic; and examined the association of health with changes in social network types during the COVID-19 pandemic.
Data.
Three recent waves (waves 6, 7, and 8) of the Korean Longitudinal Study of Aging, collected in 2016, 2018, and 2020, were used. Changes in social network types from 2016 to 2018 (before the COVID-19 pandemic) and from 2018 to 2020 (during the COVID-19 pandemic; 2020 survey period: August-December) were compared. The analytic sample consisted of 5,465 older Koreans, aged 55 years and above in 2016, who participated in all three waves.
Measures.
Five dichotomized social network indicators identified distinct social network types: (1) living together with others, (2) married, (3) weekly contact with one or more children, (4) weekly contact with close friends/relatives/neighbors, and (5) weekly participation in organizational activities. The health status comprised (1) a number of chronic conditions, (2) a number of functional difficulties, (3) depressive symptoms, and (4) cognitive function. Other covariates included age, gender, education, employment status, household income, and urban residence.
Analytic strategies.
The analyses had sequential steps. First, Latent class analysis (LCA) identified social network types at three different time points (2016, 2018, and 2020), using five network indicators. Second, Latent transition analysis (LTA), a longitudinal extension of LCA, calculated transition probabilities between social network types between 2016 and 2018, and between 2018 and 2020. Third, random-intercept latent transition analysis (RI-LTA), a multilevel extension of LTA, further refined transition patterns by isolating between-individual variation from LTA, allowing transitions solely attributed to the within-individual level.
Once social network types and their patterns of change between the three time points were identified, each respondent was assigned to the most likely transition patterns based on the highest posterior probabilities. Multinomial regression then investigated (1) the impact of change before the COVID-19 pandemic on changes during the pandemic, and (2) the impact of physical, functional, mental, and cognitive health on change during the pandemic, taking change before the COVID-19 pandemic into account.
Results.
Empirical analyses yielded three main findings. First, across the three time points, the five social network types were identified. These were (1) diverse, (2) unmarried and diverse, (3) spouse and friend, (4) spouse only, and (5) restricted, from the most to the least diverse types. Over time, older Koreans tended to experience social network contraction. Between 2016 and 2018, 8.3% of older Koreans experienced social network expansion, whereas 17.5% experienced social network contraction. Between 2018 and 2020, 7.6% experienced network expansion, while 18.7% experienced network contraction.
Second, older adults whose social networks contracted before the pandemic were more likely to expand their social networks during the pandemic. Conversely, those who expanded their social networks before the pandemic were more likely to contract their social networks during the pandemic.
Third, older adults with better cognitive function before the pandemic were more likely to expand their social networks (versus retaining their social networks) during the pandemic. Older adults with a higher level of depressive symptoms before the pandemic were more likely to contract their social networks (versus retaining their social networks) during the pandemic.
Discussion.
Among older Koreans, differences in change in social network types before and during the COVID-19 pandemic were not substantial. Notwithstanding the COVID-19 pandemic, the findings largely supported the homeostasis hypothesis: older adults who expanded their social networks before the pandemic tended to contract their social networks during the pandemic, and vice versa. The importance of cognitive and mental health in later life social networks was further highlighted.”

報告番号141

Associations between internet use and frailty among Japanese adults:the power of socialization
Osaka University YAYA Li


報告番号142

Social Determinants of COVID-19 Vaccine Attitudes in the United States
California State University, Dominguez Hills Kelin LI

Background
The COVID-19 pandemic poses an overarching impact on human societies across the globe. While there have been successes in timely development of vaccines, public efforts are still being made to diminish the unvaccinated populations. In the US, wide offering of COVID vaccines to the general public started in May 2021, but vaccination rates have been relatively stalled since late 2021 and the fully vaccinated population only increased from 63.6% on January 1, 2022 to 67.4% on June 14, 2022. Social disparities also persist as the Black population and politically conservatives in general have had lower vaccination rates.
The purpose of this study is to investigate individual-level predictors of COVID vaccine intentions and attitudes (i.e., trust, confidence, concerns) using a nationally representative sample of US adults surveyed in late 2020. Identifying these predictors can help reveal various social determinants that influences people’s vaccine decisions in combating a global epidemic.

Data and Methods
Data come from the Understanding America Study (UAS) launched by the University of Southern California. The UAS is a nationwide longitudinal survey tracking about 9,500 adults living in the US. Beginning March 2020, UAS have also been asking respondents COVID-related questions, with an average of about 6,200 respondents (response rate 88%) participating in the ongoing COVID surveys.
This study analyzed one cross-sectional wave of UAS (Wave 16) that was collected during October 14-November 10 of 2020, because it is the only UAS wave obtaining information about respondent’ attitudes towards COVID vaccines. Specifically, respondents were asked (1) whether they plan to get vaccinated if it becomes available; (2) how much they trust vaccine safety in general; (3) how much they trust the government’s approval process of COVID vaccine’s safety; (4) confidence in COVID vaccine’s health benefits; and (5) concerns about COVID vaccine’s side effects.
Individual predictors include sociodemographic (i.e., age, gender, race/ethnicity, immigrant status, marital status, household size) and socioeconomic profiles (i.e., educational attainment, household income, employment status, insurance status), pre-existing health conditions that are risk factors for worsened COVID outcomes, whether having any family or friend who was infected with COVID, and self-perceived risk of getting COVID in the next 3 months. Bivariate analysis was used to test for significant associations between a respondent’s vaccine intentions and their vaccine attitudes (i.e., trust, confidence, concerns). Multivariate regression analysis was conducted to identify significant correlations between individual-level predictors and a respondent’s vaccine attitudes.

Results
The full sample included 6,181 respondents. Regarding vaccine intentions, 23.9% said “yes, they will get COVID vaccine as soon as it’s available,” 35.9% said “Yes, but they will wait and see,” 9.6% said “No, but they’ll wait and see,” 14.8% said “No, they won’t get a vaccine,” and 15.8% were “not sure”). 52.3% of respondents fully/mostly trust vaccine safety in general, while 31.8% somewhat trust and 15.9% don’t trust. For COVID vaccine’s safety, 37.3% fully/mostly trust, 36.4% somewhat trust, and 26.2 don’t trust. The average score of respondents’ confidence in COVID vaccine’s health benefits is 14.8 (possible scale is 5 to 20) and the average score of their concerns about COVID vaccine’s side effects is 7.7 (possible scale is 3 to 12).
Bivariate analysis shows that respondents who trust vaccine safety in general (p<0.001) or those who trust COVID vaccine (p<0.001) are more likely to plan to get vaccinated. Respondents with more confidence in COVID vaccine’s health benefits (p<0.001) or those with fewer concerns about its side effects (p<0.001) are also more likely to plan to get vaccinated. Finally, multivariate regression analysis shows that being male, older age, being white/Asian, foreign-born, never married, currently working, higher income, more education, being insured, self-perceived risks of getting COVID, and having pre-existing health conditions are significantly associated with increased general trust in vaccine and confidence in COVID vaccine’s health benefits. However, when it comes to trust in COVID vaccine and concerns about its side effects, immigrant status, self-perceived risks of getting COVID, or having pre-existing health conditions was no longer a significant predictor. Conclusion Various measures of vaccine attitudes are all significantly associated with vaccine intentions among US adults during the COVID-19 pandemic. However, individual predictors vary between two dimensions of vaccine attitudes. Specifically, while higher-risk individuals (subjectively measured by self-perception and objectively measured by pre-existing conditions) have more favorable attitudes towards vaccine in general and its health benefits, they do not differ from lower-risk individuals in terms of COVID vaccine and its side effects.


報告番号143

Effects of Vaccination on Confidence in the Central and Local Governments of Japan during the COVID-19 Pandemic in 2021
Hitotsubashi University SUDO Naoki


1. Introduction
The COVID-19 pandemic since 2020 has caused a global social crisis among both infected and non-infected individuals. Coping with this crisis has become one of the most important tasks for governments. Doing so effectively would earn the government the high marks and strong confidence of its citizens, while failure to do so would cause citizens to lose confidence in it. Thus, citizens’ confidence in their government during a social crisis depends on the performance of infection control policies. In this study, I examine the relationship between confidence in the government and social crises by focusing on the timing of COVID-19 vaccination.
To explain the relationship between confidence in government and the COVID-19 pandemic, I theorized the process of changing confidence in the government as follows: First, the pandemic strengthened insecurity among individuals owing to the threat of infection. Next, vaccination enhanced their security by reducing the threat of infection. Thus, confidence in the government would be strengthened through effective vaccine distribution. Based on this inference, the following hypotheses were posited:
Hypothesis 1. During the pandemic period, vaccinated individuals are more likely to have confidence in governments than non-vaccinated individuals.
Hypothesis 2. Individuals with expectations for COVID-19 vaccination will be more likely to have confidence in the government through effective distribution of the vaccine than individuals without such expectations.
Hypothesis 3. As individuals residing in highly infected areas are more threatened by infection, they will be more likely to have confidence in the government through effective distribution of the vaccine.

2. Data and Method
To test my hypotheses, I used data from the Social Stratification and Psychology Survey in 2021 (SSPW2021-Panel), which includes online panel survey data collected during the COVID-19 pandemic period. In particular, I focused on the data for Waves 3 (conducted in August 2021) and 4 (conducted in November 2021) from the SSPW2021-Panel to confirm the effects of vaccination on confidence in the local and central governments of Japan (N = 3,324). In this study, confidence in the government (1–4), evaluation of the central government’s infection control (1–4), and evaluation of the local government’s infection control (1–4) were treated as dependent variables. Additionally, COVID-19 vaccination (vaccinated = 1, non-vaccinated = 0) was treated as an independent variable.
I analyzed data from the SSPW2021-Panel based on a difference-in-differences research design. Less than 20% of the respondents in Wave 3 were vaccinated, while approximately 80% of the respondents in Wave 4 were vaccinated. In other words, COVID-19 vaccination in Japan was widely expanded during the intervening period. On the other hand, confidence in government and evaluations of the central and local governments’ infection control policies among respondents changed sharply during that period. Considering the differences in the vaccination rate of respondents, I estimated the effects of vaccination on confidence in the central and local governments of Japan during the COVID-19 pandemic.

3. Results
All dependent variables (confidence in government and evaluations of the central and local government infection control policies) significantly improved from Wave 3 to Wave 4. As mentioned previously, the COVID-19 vaccination rate was greatly increased during the same period. However, it should be noted that the infection rate of COVID-19 caused by the delta variant also decreased simultaneously. This suggests that changes in infection rate of COVID-19 might affect changes in the dependent variables. Therefore, to estimate the effects of vaccination on the dependent variables, the effects of changes in infection rate of COVID-19 should be controlled.
The results of these analyses clarified that even with the differences in Waves 3 and 4, vaccinated respondents are more likely to have confidence in the government and to positively evaluate the local and central government’s infection control policies. Furthermore, it was observed that COVID-19 vaccination itself had a positive effect on the central government’s infection control policies. Interestingly, the effect of COVID-19 vaccination on the central government’s infection control policies was more prominent among respondents with expectations for vaccination in Wave 3 than among respondents without such expectations. Similarly, it was observed that confidence in the government regarding vaccination had a positive effect among areas with low infection rates.

4. Discussion and Conclusion
As vaccinated individuals tend to evaluate the local and central government’s infection control policies highly and have strong confidence in governments, it can be said that vaccination has positive effects on confidence in governments. This fact supports Hypothesis 1. On the other hand, differences in confidence in governments between Waves 3 and 4 remain even after controlling for the effects of vaccination. Therefore, factors other than vaccination (e.g., infection rate) also account for the changes in confidence in governments.
Additionally, the effect of vaccination on confidence in governments tends to be prominent among individuals with expectations of vaccination, rather than among individuals without such expectations. This finding supports Hypothesis 2. This suggests that it is important for the government to provide vaccination effectively to people who need it.
Lastly, the effect of vaccination on confidence in governments tends to be more prominent among individuals residing in areas with low infection rates than in those with high rates. This does not support Hypothesis 3; however, it suggests an interesting point. With the wide spread of COVID-19, the effect of vaccinations has weakened. To benefit from the effect of vaccination on confidence in the government, the government should promptly distribute vaccines before the spread of COVID-19. To ensure that citizens evaluate infection control policies positively and have confidence in government, governments should take action to prevent the spread of infection as soon as possible.”


報告番号144

主観性と自然環境のあいだ――19世紀フランス社会と社会学
龍谷大学 村澤 真保呂

気候変動や生物多様性喪失などの自然環境問題は、いまや人類と地球の持続可能性を脅かす「エコロジーecology問題」として、文系・理系を問わず学術界全体の再編を迫る大きな潮流を引き起こしている。というのも、自然環境問題の原因はきわめて複合的であり、人間社会の活動が多くの要因を占めている以上、そのメカニズムを解明し、解決策を探求するには、自然科学だけでなく社会科学や人文科学など広範な学術領域の協力が不可欠だからである。他方で学術界はいまなお専門分化と厳密化を進めており、学際化・複合化を要求する現実的要請にうまく対応できておらず、教育や研究の現場でも混乱が生じていることは否めない。そして社会学は国家の枠内における市民社会に対象を限定され、グローバルなエコロジー危機に対応できず、その役割は人類学や地理学が担っているように見受けられる。しかし、それは社会学のほんらいのあり方に沿うものなのだろうか? 現在のエコロジー問題が学術界に引き起こしている事態は、150年近く前のヨーロッパの産業社会が経験した事態と多くの点で重なっている。すなわち急激な産業都市化にともなって深刻化した貧困や犯罪などの「社会問題」とそれを対象とする学術領域としての「社会学」の形成である。当初のシオロジー(sociology)は現在のエコロジー(ecology)と同様、きわめて曖昧な概念であると同時に、広範な対象や方法を含む概念であった。そこから多くの議論を経て現在の専門学術領域としての社会学とその方法論が確立されていったことは、社会学徒にとって常識に属することであろう。 本発表では、社会学がグローバルなエコロジー危機に対処する学問として生まれ変わる可能性を考えるために、現在の地球環境問題と学術の関わりについてIPBES(生物多様性と生態系サービスに関する政府間パネル)の事例を取り上げ、そこで生じている科学と宗教をめぐる論争が、19世紀後半のフランス社会学の成立時に起こった論争と主題も対立構図も同一であること、つまりいずれも「主観性」をめぐる論争であることを示す。続いて、その主題が20世紀にどのように変容したかを歴史的に整理しつつ、最終的に「社会問題」が「地球環境問題」の、また「ソシオロジー」が「エコロジー」の源流であることを示すとともに、アメリカ社会学におけるBurawoyらのPublic Sociologyをめぐる議論との関連も意識しつつ、19世紀フランス社会学史が照らすこれからの社会学の方向性について論じてみたい。

報告番号145

離村と習俗の軟化――19世紀のフランス社会と社会学
龍谷大学 稲永 祐介

【1.目的】本報告の目的は、19世紀のフランス社会が抱えた離村問題を検討し、習俗が市民性に置き換えられていく歴史的プロセスの一側面を政治社会学的に明らかにすることにある。【2.方法】離村はどのような文脈で議論され、いかなる歴史的・文化的課題として解決されるべき政治問題となったのだろうか。こうした問題設定から本報告は、離村にたいする共和派とカトリック教会派による中間集団の支援活動に焦点を当てた分析を行う。資料には講演記録、公民教科書や団体の冊子などを扱う。【3.結果】フランスでは、鉄道網の飛躍的な拡大と産業構造の変化に伴う人口移動や家族形態の変化が社会変動を引き起こしていた。当時の産業化をめぐる共和派の認識には、蒸気機関の推進力が法の前の平等の理念を普及させるという平等主義の理想(セレスタン・ブーグレ(1870-1940)、1899年)を見出すことができるが、習俗の観点から農村の実態を見ると、離村問題は、青年が土地や長男としての義務を放棄し都市に移住するという、家父長制の道徳的な債務から離脱する問題であった。離村が都市の不安定な賃金労働者を供給し続けたことから、共和政府は、政治経済と道徳の領域で衰退する農村を放置することが政権と社会全体の安定に多大なリスクを招く素因であるとみなし、離村に適切な施策を講じることで深刻な危機を未然に防ごうとした。1881年、レオン・ガンベッタ(1838-1882)を中心とした共和派は、「農村のデモクラシー」を提唱し、農商務省から農業省を分離独立させるなか、小作の自立を支援する結社「農業奨励会」の設立を提唱した。この官製型アソシアシオンは、1868年にカトリック教会派が大地主や大資本家とともに設立した「フランス農業者の会」の活動に対抗する政治的な制度であった。共和派は、「農業奨励会」の活動によって小作や自作農に科学技術と経営手法を提供するとともに、土地所有の啓発に努め、地方農村の再編をめざしたのに対し、カトリック教会派は、「フランス農業者の会」の伝統的な習俗であるパトロナージュの奨励によって、教会が保護する父権的な情愛が社会不安を解消させると主張した。【4.結論】離村対策において第三共和政初期の共和派は、強い国家が社会的な領域に直接介入する手段を取らなかった。その理由は、戦後の復興と社会的分断のなか、「フランス農民はなぜ二度もボナパルトに騙されたのか」(ジュール・フェリー(1832-1893)、1885年の発言)という問いのもとに、市民の私的なイニシアティブを重視したからであった。この文脈において、「農業奨励会」の官製型の活動は、産業社会の進展のなかで懐疑的となった旧来の共同性に代わる、新しい社会的な価値の創造の試みであったことが確認できる。共和派の離村対策のねらいは、法律に従い政策を実施する国家の領域と、諸個人の自由な良心に基づき自発的に行動する社会の領域との新しい関係をめざす農村の政治的社会化であった。

報告番号146

フュステル・ド・クーランジュからデュルケームへ――19世紀のフランス社会と社会学
西南学院大学 北垣 徹

フュステル・ド・クーランジュが『古代都市』(1864)で示すのは、社会関係の起源にある信仰である。すなわちそれは、人間精神の内で強制的に命じる祖先崇拝の信仰である。この信仰をもとに、祖先を奉る墓の周りに社会が形成され、墓所tombeau/家族famille/地所propriétéという三様態が生じる。このような死者崇拝による社会形成という発想は、コント派の実証主義に近い。オーギュスト・コントは「生者はますます死者によって支配されるようになる。それが人間秩序の必然的法則である」といい、またジャン=フランソワ・ロビネは「墓地なき社会はない」という。他方でフュステルの議論は、ヨーロッパ各地の労働者調査の結果「直系家族」を理想とするフレデリック・ル・プレにも近い。つまりそれは反ルソー的発想である。ルソーが契約を見出すところに、フュステルやル・プレは信仰を見出す。このとき、出発点にあるのは孤立した諸個人ではなく家族である。かくして19世紀フランスにおいて、社会関係についての個人主義的な契約論を忌避する流れが存在する。人は自然状態においてすでに社会的であり、家族がその原初的形態である。「幾何学的平面が線や点に分解できないのと同様に、社会は個人に分解することはできない」(コント『実証政治体系』)。 フュステル・ド・クーランジュがこのような議論を行う際、古代の自由と近代の自由を峻別しようとする狙いがある。つまり、バンジャマン・コンスタン「近代人の自由と古代人の自由」(1819)同様、古代社会の根源的な異質性を示し、フランス革命期における古代の模倣を批判しようとしたのだ。古代都市citéとは、市民citoyenにはすべてが与えられるが、個人individuの自由はまったく存在しない「国家の全能」状態である。その基盤が政治的というよりも宗教的なものであることを、フュステル・ド・クーランジュは見抜くのだ。  エミール・デュルケームは1880年頃、高等師範学校でフュステル・ド・クーランジュから直接教えを受けている。デュルケームの博士論文『社会分業論』(1893)においては、古代ローマの氏族(ゲンス)解釈をめぐって、フュステルに批判的な言及を行っている箇所もある。しかしフュステルのデュルケームへの影響が明確に現れるのは、1895年以降に展開される宗教論である。フランス社会学派の創始者は『古代都市』の著者から、教義よりも儀礼を重視する宗教観や、聖なるものへの信仰が社会をつくるという命題を受け継ぐ。そして『社会学講義』(1950)に窺われるように、聖なるもの/社会/所有を連続したものとして論じる。他方でデュルケームは、家族にたいして個人を対置することなく、個人を社会に再び招き入れることで、近代人の自由を新たなかたちで描こうとする。本報告では19世紀の思想史の文脈において、デュルケームの受け継いだ古いものと、彼のもたらした新しいものとを確認する。そして同時に、デュルケームが切り捨ててしまったことで、社会学が忘れ去ってしまったものを思い返す。

報告番号147

19世紀フランスにおける社会主義と社会学――19世紀のフランス社会と社会学
神戸大学 白鳥 義彦

【1.目的】  本報告は、デュルケームによる講義録『社会主義およびサン=シモン』等も念頭に置きながら、19世紀のフランス社会と社会学の諸相の一端を明らかにするために、19世紀フランスにおける社会主義と社会学との関係を検討することを目的とする。  フランスにおける社会主義の歴史を一瞥すると、1904年に「ユマニテ」紙を創刊し,統一社会党(SFIO)の結成にも中心的な役割を果たしたジャン・ジョレス(1859-1914)は、高等師範学校においてデュルケーム(1858-1917)の一年先輩にあたる。また「ユマニテ」紙の刊行に大きな役割を果たし、ジョレスやレオン・ブルム(1872-1950)(高等師範学校を中退している)ら社会党の指導者や知識人に強い影響を与えたリュシアン・エール(1864-1926)は、高等師範学校を卒業後、同校の図書館司書をつとめていた。このように、19世紀のフランスには知識人社会主義と言うべき流れが見出される。こうした観点にも注目しながら、当時のフランスにおける社会主義と社会学との関係について考察する。 【2.方法】  フランスにおける社会主義の流れを概観したのち、デュルケーム自身の議論については、『社会主義およびサン=シモン』やJ. C. フィユー編の『社会科学と行動』第二部「社会学と社会主義」に収録されている諸論文等をもとに考察を行う。またジョレス、エール、マルセル・モース(1872-1950)らの議論も適宜参照する。 【3.結果】フランスにおいて社会主義の思想を先駆的に示した者として、サン=シモン(1760-1825)やシャルル・フーリエ(1772-1837)らの名を挙げることができるが、彼らはフリードリヒ・エンゲルス(1820-1895)によって、自らとカール・マルクス(1818-1883)による科学的社会主義との対比の中で空想的社会主義者と位置づけられたことは知られている。現実のフランス社会における社会主義の動きを見ると、多くの党派が分立していたフランス社会主義運動は1890年代以降、統一組織形成への模索がなされるようになった。だが、いわゆる革命派と改良派の対立は根深く、ゲード派とブランキ派の合同した党と、ジョレス派とアルマーヌ派の党とに運動は二分されていた。その対立を超えて1905年に成立したのが統一社会党である。  デュルケームは「現に拡散的である経済的諸機能の一切、またはそのうちの若干のものを、社会の指導的で意識的な中枢部に結びつけることを要求するすべての学説を、社会主義的と呼ぶ」(『社会主義およびサン=シモン』訳31頁)と述べる。また「社会主義は、何よりもまず、〔社会〕有機体の全体のなかで産業装置を現在とは異なった位置におき、何が何だかわからぬまま無意識のうちに動いていた暗闇のなかから産業を引っ張り出し、明るいところに呼び出して、意識の規制のもとにおくことになるよう、社会全体を再編成しようとする熱望である」(39頁)とする。さらに「経済的領域に直接関係していないが、それにもかかわらず前述の事柄とかかわりをもっている諸理論をも社会主義的と呼ぶ」(41頁)と述べている。 【4.結論】  階級問題、経済問題等の観点から社会主義を論じることが可能であるが、デュルケーム社会学の立場からは社会的事実としての社会主義を、経済的なものに限定されずに社会の枠組みという観点から論じようとしていた。これは「社会的なもの」について考える際の重要な論点となり得る。

報告番号148

戦後のアドルノと「社会学のアクチュアリティー」――「諸関係の脱人間化」と「人間の形成」
城西大学 表 弘一郎

【目的】  本報告の目的は、アドルノの講演「社会学のアクチュアリティー」(1951)を詳細に検討して、アドルノが戦後ドイツ社会の再形成に積極的に関与しようとした企図を再構成することにある。 【方法】  本報告はアドルノが西ドイツ帰国直後に行なった講演の位置価を、コンテクストと照合してある程度明らかにし、そのうえでテクスト本体を詳細に検討して、アドルノの社会理論の形成史とアドルノ思想に占める意義、および戦後ドイツ社会学史に占める意義、さらには戦後ドイツ社会の知的再建に占める意義を思想史の方法によって明らかにする。 【結果】  同講演によれば、哲学の仕事は、「哲学の問題圏がその中心において社会諸関係と衝突するという直観」にあるのであり、社会の諸問題を外側から倫理学や認識論の諸問題に持ち込まなければならないといったものではないという(Adorno(2019)34)。社会学は、一方で国家社会主義レジームを阻み、今日別の全体主義的システムによって同様に阻まれ、人間の尊厳に相応しい社会の前提を作り上げうる「社会の自己省察」(S.32)と、他方でまさに戦後の状況下で必要とされている事柄に、その必要の度合いに応じて計画的に取り組んでいく試みとの合致と捉えられている。  社会学が取り組むべきドイツの諸問題として、アドルノは具体的に3つの問題を挙げる。難民問題、住宅問題、そして社会学と政治学との関係である(S.35-37)。なかでも彼が重視するのは社会学と政治学との関係であり、それは社会学の哲学的アクチュアリティーにとって直接的なモデルを提供する(S.37)。すなわち、そこで出発点になるのは「生が物のように硬化した概念上の鋳造物を、…意識の自己批判によって乗り越えるという使命」(ebd.)である。だが、制度的に硬化し疎外された生に形而上学をただ対置する場合にはこうした克服は可能ではないため、ここにこそ社会学の特筆すべき使命があるとアドルノは言う(ebd.)。すなわち、社会学の使命とは、「人間には疎遠で、決定論的な観点では非人間的な世界として対置される世界が、まさに同時に人間の世界であるという事実を、再び人間の意識にもたらすこと」、そして「最後には、こうした諸関係の形成と人間の形成そのものから変えうるということ」である(S.37-38)。  ここで見るべきは、社会学の哲学的アクチュアリティーと社会学固有のアクチュアリティーとを弁別し、前者の哲学的アクチュアリティー、すなわち「意識の自己批判」という哲学の伝統的な方法の限界を踏まえながら、社会学の方法の固有性に注目を促している点である。いわば社会学とは、物象化した社会の克服の方途を社会関係の再形成可能性と「人間の形成」に即して具体的に提示しうる実践的な学問なのである。その際、経験的社会科学の意義は、人間を再度捉えかねない全体主義的な妄想を阻止しうる「最も重要な対抗力(Gegenkräfte)」(S.40)にあるのだ。 【結論】  アドルノにとって社会学のアクチュアリティーとは、「国家社会主義レジームのもとでの社会学の抑圧」(S. 508)という事実から始めて、伝統的なドイツ社会学の性質としての「事実からの疎外(Faktenfremdheit)」(S.38)を乗り越えるべく、経験的調査を用いて、そうしたレジーム再興を阻止し、人間の尊厳に相応しい社会の前提条件を提示するところにあったことが明らかになった。

報告番号149

存在論的転回と「ネイティヴ」の理解:社会学と人類学の交錯
東京藝術大学 磯 直樹

社会学において、研究対象としての人間を理解するとはどういうことかという問題は、古典的であると同時に現代的でアクチュアルである。本報告では、この「理解」をめぐって社会学と人類学の議論を交わらせることで、20世紀の社会学理論においては十分に考察の対象とならなかった「自然」の問題を社会学に採り入れる。その上で、「理解」のために必要な反省的認識とは何かを考察する。  ヴェーバーは『社会学の基礎概念』、「カエサルを理解するためにはカエサルになる必要はない」と述べつつ、独自の「理解社会学」を構想した。他方で、人類学者のマリノフスキーは『西太平洋の遠洋航海者』において、民族誌的フィールドワークが3つの方法で行わなければならないと述べた上で、それらの方法はエスノグラファーが見失ってはならない最終的な目標に通ずると主張する。その目標とは、「ネイティヴ(現地人)の世界に対する彼(ネイティヴ)の見方、彼と生活との関係をよく理解(realise)し、彼(ネイティヴ)の視点を獲得すること」、である。こうしたヴェーバーとマリノフスキーの議論を批判的に継承し、独自の解釈人類学を展開したのがクリフォード・ギアツである。  ギアツの立場はしかし、存在論的転回と呼ばれる人類学の一連の議論によって、根本から問われることになる。浜田(2018)によれば、人類学における存在論的転回の存在論とは、まずもって自然と文化の関係に関する西洋の前提と、それを相対化しうる異なる発想のことを指している。こうした発想は、様々な存在のあり方を問うことから「存在論的転回」と呼ばれる(Holbraad & Pedersen 2017)。このような存在論的転回の潮流を代表する人類学の一人が、フィリップ・デスコラである。彼の仕事が、ヴェーバーともギアツとも異なるのは、自然に着目していることである。特に『自然と文化を超えて』では自然と文化の関係から四種類の存在論を提示し、「自然の人類学」を用いて、人間が自らを対象化する」ことを目指している。他方で、デスコラは「理解」の問題を検討していない。  「自然」に着目しつつ、「理解」の問題も扱った研究として、ブルデューとサヤドの『デラシヌマン』(1964年)を挙げることができる。このタイトルは「根こぎ」を意味する。この著作は、アルジェリア戦争期のアルジェリアの多くの地域で、農民の強制集住が行われた社会的帰結を問題にしている。自然としての土地と農業の営みと一体であった伝統的農民の文化は、フランス当局の強制集住政策によって破壊される。農民たちの多くは、住み慣れた土地から離されて、新たな土地に根を張ることができなかった。根こぎにされたのである。ブルデューとサヤドは、自然と文化を一体的に捉えつつ、「ネイティヴ」と言えるかどうか分からない人たちを理解しようとしている。『デラシヌマン』では、ヴェーバーの『プロテスタンティズムの倫理と資本主義』がところどころで意識されており、強制集住の帰結による農民の変容が様々な因果関係とともに記述されている。  二人の著者が『デラシヌマン』で試みたことの一部は、1993年に刊行された『世界の悲惨』 に引き継がれている。本報告では、両者の1980年代以降の仕事も取り上げ、彼らの実践した反省的認識を通し、存在論的転回が見落とした「理解」の問題を再考する。

報告番号150

性愛なき結婚からみる恋愛・親なり・社会的承認――友情結婚カップルの調査を通じて
日本大学 久保田 裕之

【1.目的】 少子化の延長に未婚化・晩婚化が議論されるなか、性愛関係なしで結婚し/ときに子どもを産み育てる「友情結婚」が注目されている(小川2019)。友情結婚とは、恋愛も性関係も否定する形で行われる結婚および結婚生活を指す。この点、日本で友情結婚を扱う事業者スタッフを通じた調査からは、友情結婚が同性愛者や無性愛者にとっての偽装的結婚として機能しているだけでなく、恋愛と性を首尾よく実践しにくい異性愛男性にとっての社会的承認の手段としても機能している可能性が示唆されている(久保田2022)。 しかし、友情結婚当事者が実際に、結婚を目指す過程でどのように性愛規範(Brake 2012=2019)に直面し、友情結婚に何を求めているのかは、十分明らかになっていない。というのも、当事者の多くは周囲に友情結婚を秘匿しており、その実体はブログなど匿名で断片的な発信に頼るほかないためである。 そこで本報告では、日本の友情結婚当事者を対象とした聞き取り調査を行い、友情結婚に至る過程と結婚生活実態から、日本における結婚の理想が性愛規範と親なり規範とどのような関係にあるのかを明らかにする。 【2.方法】 方法として、機縁法および日本で友情結婚を扱う事業者からの紹介を通じて、友情結婚当事者6名に対して、各90分程度の半構造化インタビューを行う。具体的には、友情結婚に至る過程と友情結婚生活実態に関して、対象者のセクシュアリティとの関係から、1)恋愛結婚からのプッシュ要因、2)友情結婚へのプル要因、3)友情結婚に至る困難、4)恋愛結婚の偽装に関する困難を中心に聞き取る。 【3.結果】  結果として、日本における友情結婚は、1)同性愛男性にとっては自らの性的生活や同性パートナーの存在と両立する形で、2)無性愛女性にとっては望まない恋愛・性関係を受忍することなしに、3)性愛過程に困難を抱える異性愛男性にとっては高度に認知化された恋愛と性の高いハードルを越えることなしに、結婚と親なりを通じた社会的承認を得る手段として機能していることが当事者からも追認されただけでなく、4)日本で友情結婚が求められる背景には、夫婦の性愛が生殖を超えて規範化されず夫婦の性愛よりも子を基軸とした家族愛を中心とした日本型近代家族の特徴や、親子関係の現代的親密化と呼応する親へ報恩の論理など、性愛規範よりも親なり規範に強く依拠する社会的承認の構造とも深く関わっていることが明らかになった。 【4.結論】 以上の議論から、日本における友情結婚には、現行の結婚制度が性愛規範に立脚するゆえに排除された非異性愛的セクシュアリティの受け皿という側面のみならず、現代的で子ども中心的な親なり規範によりも強く立脚するゆえに、容易に性愛の回避・隠匿を可能にし、その中核たる親なりを通じた社会的承認への迂回路となっている側面があることが示唆された。 参考文献 Brake, Elizabeth., 2012, Minimizing Marriage: Marriage, morality, and the law, Oxford University Press(久保田裕之監訳,2019,『最小の結婚――結婚をめぐる法と道徳』白澤社). 久保田裕之,2022,「友情結婚と性愛規範――日本における仲介事業者の調査から」牟田和恵編『フェミニズム・ジェンダー研究の挑戦』松香堂書店:44-58. 小川祐樹,2019,『友情結婚という選択――幸せな家庭を築くための4つのステップ』幻冬舎.

報告番号151

コロナ禍で親になる――第一子妊娠中のカップルに対するインタビュー調査
東京都立大学 伊藤 大将

1 目的  コペアレンティングとは、「協力関係にある二人以上の大人が、保護者として責任を共有する子どもを育てるために必要な、すべての活動を行う組織のような関係」(Talbot and McHale 2004)を指す言葉である。コロナ禍での先の不透明さや環境の変化は、出産や子育てが未知である第一子を妊娠中のカップルがコペアレンティングを形成していく過程に、どのような影響を及ぼしたのかを明らかにするのが、本報告の目的である。具体的には、コロナ禍で親になっていくプロセスをカップルがどのように経験し、家事・育児の分担をどう準備しているかに焦点を当て、シンボリック相互作用論と認知社会学の視点を用いて分析する。 2 方法  2020年8月から2021年3月までに44組、2021年12月から2022年1月までに4組、計48組のカップルにインタビュー調査を行った。調査対象者は、双方にとって第一子を妊娠中の妊娠後期のカップルである。配偶者が外国人であるカップルが、48組中3組おり、他は双方が日本人のカップルだった。調査参加者は、SNS、マタニティサイト、マタニティアプリ、産婦人科、調査員の知り合いからの紹介等を使って募集した。 調査員5名が、カップルに別々で半構造化インタビューを行った。コロナ禍ということもあり、ほとんどのインタビューはオンラインだったが、4組は対面だった。インタビューは約45分から1時間超の長さで、すべてのインタビューを録音し、書き起こした。分析には、グラウンデッド・セオリー・メソッドのLaRossa(2005)のアプローチを用いた。 3 結果  コロナ禍でリモートワークが増加し、家から働くことができたカップルは、夫妻で過ごす時間が増えた。一方では、夫が妻の体調や身体の変化を目にし、家事をする機会や会話が増えることで、カップルとしてコペアレンティングをしていく自信につながっていた。しかしもう一方では、妻は夫が忙しく働く様子を目にし、産後、夫に家事・育児を頼ることに限界を感じ、妻が一人で家事・育児の多くを担う覚悟をしている様子もうかがえた。加えて、親からの支援を得られないカップルや、リモートワークができない職業についている夫や妻は、限られたリソースを最大限に利用して親になる準備をしていた。家事のオートメーション化は、そのうちの一つの工夫であった。 4 結論  コロナウイルスのまん延と妊娠をほぼ同時に経験した本調査参加者は、コロナ禍で変化した環境に、ポジティブな側面とネガティブな側面の両方を感じていた。コロナウイルスまん延の影響は、主に働き方を通して生じており、カップルは居住地の感染数や職場環境といったメソレベルの要因とそれぞれが持つリソースといったミクロレベルの要因とをやりくりして出産とその後の準備に取り組んでいた。

報告番号152

市場経済期における中国高学歴女性のキャリア意識――親の影響に着目して
東京都立大学大学院 湯 婉秋

【目的】本研究の目的は,中国社会における社会経済制度の変遷と,性別役割分業意識の現状,そして,歴史的経緯を踏まえて,高学歴若年女性の性別役割分業意識,キャリア意識や就職行動を考察することである.また,そこに表れた親からの影響を検討することである. 社会主義革命以前の中国社会で,女性は農民家庭経済を維持するための重要な労働力であった.安定的で,また継続できる生産様式や労働力を確保するために,家父長的な倫理や家族規範が中国社会を長年支配してきた(Stacey 1983).故に社会主義革命から計画経済期までは,中国女性が家庭内の労働から脱出し,公的領域で就労することは,女性解放や社会主義化の象徴として重視され,農耕社会時代の性別役割分業制度を強調する伝統的な家父長制規範を打破する姿勢が強く表れていた(瀬地山1996). 市場経済体制となった現代中国社会においても,女性の就労は依然として重視されており,若年女性の理想のライフコースは生涯継続就労とされる(宮坂 2014).一方,若年層の性別役割分業意識は保守化している(第三期中国婦女社会地位調査課題組 2011).つまり,女性が就労することを当然としつつ性別役割規範が維持されるという,矛盾した意識が存在する. 家父長的規範を維持しながらも女性の就労を前提とする社会経済体制は,ミクロレベルでの親子の相互行為にも影響を及ぼす.若者の社会化において子は,ロールモデルである親からの影響を受け(神林 2000),また親から子という性別役割分業意識の世代間伝達(小川 2015)や親の期待がもたらす影響(林川 2015)も指摘されている.さらに,中国人は家族との繋がりが強く,特に高学歴者は父権尊重意識が高いことが指摘される(伊達 2015).故に中国若年女性のキャリア意識の形成を考察する際,親からの影響は不可避の視点である. 【方法】主な調査方法は,中国在住の若年層に対するインタビューである.対象者はスノーボールサンプリングで,広東省戸籍を持つ大学・修士課程の在籍者や,卒業5年以内の女性12名を抽出し,キャリア意識や親からの影響について半構造化インタビューを行った. 【結果】調査からは以下の結果を得た.①市場経済期の中国では資本主義が浸透し,ジェンダー不平等が顕在化していることが示唆された.市場化に伴う過酷な労働環境や激しい競争を回避するため,仕事より家庭を主とする傾向が見られる.②「男性は仕事,女性は仕事と家庭」の新・性別役割分業が広く認められる一方,就労は女性個人の持つ能力の体現であるという意識は根強い.それは公的労働への参加を重視し,強調する社会主義女性解放論がいまだに影響力を持っていることを示唆する.③ロールモデルとする親の影響が見られた.女性は主に母親をロールモデルにし,家族優先の前提で就業する.男性は父親を模倣し,将来は稼ぎ手となることを見越し,仕事中心のキャリアプランを立てる.④親世代からの期待や養育態度に男女差がある.息子には仕事の成功を,娘には家事やケア役割を期待する.そして子世代は親の期待を内面化していく.⑤家族間の家父長的なヒエラルキーが存在する.父親は家族を支配し,母親は,協力者として子どものケアや管理をする.故に子世代はキャリアを選択する際,母親の干渉や管理を受けながらも,自発的に父親の意向を重視する.

報告番号153

父との同居と母との同居―日本と台湾における規定要因の比較― ――日本と台湾における規定要因の比較
関西大学 大和 礼子

1.問いとその背景 親-成人子の同居では、どの親との同居なのか、どのタイミングでの同居なのか等、同居のあり方は多様である。これまで東アジアでは、「夫親との、結婚当初からの同居」という父系規範に沿った形態が、近代化によってどのように変容しているかという問題関心から、「夫親との同居/妻親との同居」や「結婚当初からの同居/途中同居」等が主に比較されてきた。一方「父との同居/母との同居」の比較についてはあまり注目されてこなかった。その背景として規範的形態である「結婚当初からの同居」においては、実態として父母ともに若く健在であることが多く、父との同居と母との同居は「一体のもの」という暗黙の想定があったからだと考えられる。 しかし、定年年齢の延長や老齢年金などの充実により高齢者の経済的自立が可能になると「結婚当初からの同居」は減少し、親がかなり高齢になってからの「途中同居」が増え、「途中同居」では、配偶者との死別等により「単身になった親」との同居という形をとることが多くなる。こうした単身の親との途中同居が増えると、父との同居と母との同居は「一体のもの」という想定は成り立たなくなる。そこで本研究では、父との同居と母との同居で規定要因がどう異なるかを、日本と台湾で比較する。 2.データ East Asian Social Survey (EASS) 2006から得られたデータを分析する。 3.結果の要約と考察 まず、日本においては、父と母で同居の規定要因が異なり、ニーズ要因は、父との同居をより強く促進(+)し、一方、規範要因は、母との同居をより強く抑制(+)する傾向が見られた。この傾向は、夫親と妻親の両方で見られた。さらに親の配偶関係を考慮に入れると、多くの要因が、父母そろっている場合に比べて、単身の父との同居はより促進(+)し、逆に、単身の母との同居はより抑制(-)した。 それに対して台湾では、同居の規定要因の効果は、父と母でほぼ同じであり、さらその効果が、親の配偶関係によって異なることは、ほとんどなかった(ただし台湾では、妻親同居のケース数が非常に少ないため、夫親についてのみ分析した。) 以上の結果をやや単純化してまとめると、日本では、父と母で、さらに父母そろっている親と単身の親で、同居の規定要因は異なった。一方、台湾では、父と母で、また父母そろっている親と単身の親で、同居の規定要因はほぼ同じだった。 なぜ父と母、それぞれとの同居の規定要因が、2つの社会でこのように異なるのか。第1に、文化的規範(父-子関係が母-子関係より重視され、また性別役割では、父は家庭ではケアの受け手、母はケアの与え手とみなされる)の面では、日本と台湾にはそれほど大きな違いはないと考えられる。しかし第2に、制度(公的年金やなど)に注目すると、上記の文化的規範(父との同居の方が、母とのそれより重視される)が、現実の生活で顕在化するための制度的条件が、日本と台湾では異なる。そのため、日本の方が、上記の文化的規範が、現実の同居のあり方としてより顕在化しやすいのではないかと考えられる。

報告番号154

シングルマザーの睡眠と健康――JGSS2021を用いた分析から
富山大学 中村 真由美

シングルマザーの睡眠と健康――JGSS2021の分析から  富山大学 中村真由美  子育てと労働の二重負担は日本のシングルマザーの健康や睡眠にどのような影響を与えているのだろうか。本研究は、JGSS2021データを用いて、日本のシングルマザーの直面する睡眠と健康格差を明らかにする。  日本のシングルマザーは国際的にみても就業率が高いが、経済的に困難を抱える傾向がある(厚生労働省2016)。厚生労働省(2016)によれば、シングルマザーの就業率は高く、81.8%もが就業しているが、シングルマザーの平均年間個人収入は243万円、世帯収入は348万円であり、子どもを持つ一般世帯の49.2%に過ぎない。シングルマザー世帯の8割は離婚によるものであるが、元配偶者から養育費を受け取っている世帯は24.3%にすぎない。 配偶者なしで子育てをしながら、生活費を得るために就業もしなければならないという二重負担は、シングルマザーの健康に負の影響を与えている。Nakamura and Akiyoshi (2021)のJGSS2012を用いた分析によれば、 シングルマザーは既婚マザーに比べて労働時間が長く、健康状態が悪かった。さらに、子どもの数が多い上に本人の健康状態が悪い場合には、既婚マザーは労働時間を減らすのに、シングルマザーは労働時間を減らしていないことが明らかになった。  そこで、本研究で新たに注目したのは「睡眠」である。女性の労働と家庭役割の関係を考える際に、これまでの研究では「睡眠」という視点が抜けていることが多かった。しかし、労働時間が長く、子どもの世話も一手に引き受けなければならないとすれば、睡眠時間は短くなることが考えられる。「睡眠」は心身の健康や幸福感と関わるキー概念である。そこで本研究ではJGSS2021に新たに導入された、睡眠についての質問項目を用いた。  分析結果としては、シングルマザー(離婚)は既婚マザーに比べて、睡眠時間が短く、さらに、健康状態は悪い傾向がみられた。クロス表分析と独立性の検定の結果では、健康状態を(「最高に良い」+「とても良い」+「良い」)と答える割合は、既婚マザーで64.4%いたのに、シングルマザーでは36.4%であった(p<.05)。また、睡眠時間については、既婚マザーで6.43時間だったのに対し、シングルマザーは6.07と36分も短い。睡眠の質には有意差はないが、「良い(非常に良い+良い)」と答えた割合は既婚マザーで55.3%、シングルマザーで36.4%であった(p<.10)。当日はさらに分析を進めた上で発表を行う予定である。 注:当該データでは、未婚マザーや死別マザーの該当ケース数が極端に少なく、また睡眠時間や健康状態などの傾向が離婚マザーとはかなり異なるため、本稿の分析では除外している。 Nakamura, Mayumi and Mito Akiyoshi, 2021 “Health deprivation of single mothers in Japan” ⅣISA Forum of Sociology. 厚生労働省.2016. 「平成28年度 全国ひとり親世帯等調査の結果」厚生労働省ホームページ,(2022/06/20取得,https://www.mhlw.go.jp/stf/houdou/0000188138.html).

報告番号155

政策からみた「圧縮された近代」における「家族主義」――高等教育支援を事例として
東京大学大学院 朴 慧原

1. 目的 『リーディングス——アジアの家族と親密圏』では、現代のアジア社会における家族や結婚、ジェンダー、セクシュアリティ等についての多様性・重層性が示されている。アジアは西欧とは異なる近代化としての「圧縮的な近代」(Chang 2010; 落合編 2013) を経験している。アジア内部での多様性・重層性は、この特異な社会変動の「圧縮」の度合いや政策決定の質、そして文化的差異によって生み出されてきたといえる。その中核には「家族主義」の多様性がある(Ochiai 2019: 29)。そこで本報告では、とりわけ2000年代以降の日本の社会政策の内実を実証的に検討することを通じて、アジアの「家族主義」の多様性とその由来を理解する手がかりを得ることを目指す。 2. 方法 第一に、社会政策の「家族主義」的傾向やジェンダー構成を問うレジーム論の再編(Orloff ほか1993)を手がかりにしつつ、アジアの政策決定が、①政策プログラム、②政策の利用実践、③政策の結果という3つの水準における家族の位置づけおよびジェンダー化の度合いから分析できることを示す。第二に、この方針から、日本では近年までその責任が主に家族に任され(家族中心主義的)、進学率において男女差が著しかった(ジェンダー化)高等教育をめぐる社会政策である高等教育支援の分析を行う。 3. 結果 ジェンダー化の度合いからみると、日本の高等教育支援は①の水準では脱ジェンダー化されているが、②奨学金利用実践や③奨学金の返還・延滞率においてはジェンダー化されている可能性がある。他方、家族の位置づけからすると、奨学金制度は、①の水準では対象となる若年層の位置づけを「子ども」から「大人」へと再設定されており、家族中心主義的な傾向の見直しが行われていた。しかし、生活保護世帯の若年女性が世帯の収入源として奨学金を借りるという事例など、②③の水準では、依然として家族中心主義的な側面が残存している状況が確認できた。この状況は2000年代以降の人口・経済的な変化と、日本における高等教育が持つ社会的な意味の変化が重なったことによると考えられる。 4. 結論 以上の分析結果は、①政策プログラム②政策の利用実践③政策の結果の3つの水準それぞれで、家族の位置づけやジェンダー化の度合いは異なり、その組み合わせの差異は社会的・文化的条件によってもたらされる可能性があることを示唆する。このことから、「圧縮的な近代」を経験する社会における「家族主義」の多様性は、政策における家族の位置づけとジェンダー化の度合いに関する複数のケーススタディを比較分析することでその一端が明らかになると考えられる。 文献 Chang, K., 2012, South Korea under Compressed Modernity, Routledge. 落合恵美子編,2013,『親密圏と公共圏——アジア近代からの問い』京都大学学術出版会. Ochiai, Emiko, 2019, “The logics of gender construction in Asian modernities”, Jieyu Liu and Junko Yamashita ed., Routledge Handbook of East Asian Gender Studies, Routledge, 13-35. Orloff , Ann Shola, 1993, “Gender and the Social Rights of Citizenship: The Comparative Analysis of Gender Relations and Welfare States, Americal Sociological Review 58(3): 303-328.

報告番号156

三大都市圏における格差拡大の進行過程とその社会的帰結に関する研究――(1)研究の概要と三大都市圏の階級間格差
早稲田大学 人間科学学術院 橋本 健二

1.研究の背景  日本では1980年前後から経済格差が急速に拡大を続けてきた。この格差拡大は、企業規模間・産業間、階級・階層間などのの格差拡大とともに、地域間格差の拡大をも伴っていた。しかし都道県間格差は、景気変動にともなって拡大・縮小するにとどまっており、都道府県内の格差拡大、とりわけ大都市部での格差拡大が顕著である。つまり、都市分極化の進行である。都市分極化は、貧困地域の形成、ジェントリフィケーションによる富裕地域の形成、地域間・地域内での利害対立の発生など、さまざまな変化や問題を生み出す可能性がある。本研究は、大都市部における格差拡大の進行過程と、その社会的帰結の解明を目指すものである。 2.研究の方法  上の目的のため、本研究が採用するのは、都市社会学的な社会-地区分析の方法と、階級・階層研究で一般に用いられてきた質問紙調査による分析を統合するという研究方法である。具体的には、次のような作業を行なった。  まず「国勢調査」の市区町村別集計と小地域別集計をもとに社会-地区分析を行ない、各地域の社会経済指標を算出するとともに小地域単位のクラスター分析を行なって、各地域の特性と類型を明らかにした。次に東京圏、名古屋圏、京阪神圏を対象とするインターネット調査を実施した。回答フォームには居住する小地域に関する設問を設け、作成したデータには、各回答者が居住する市区町村および小地域の特性と地域類型に関する変数を付加して、分析に用いることができるようにした。調査は東京駅から半径60km、名古屋駅から半径40km、大阪駅から半径50km以内に居住する20-69歳の住民を対象とし、2022年1月から2月に実施され、有効回収数は東京圏26001人、名古屋圏6218人、京阪神圏11601人、合計43820人だった。 3.階級間格差の概要  階級分類としては、資本家階級、新中間階級、労働者階級、旧中間階級の4階級分類を基本に、必要に応じて労働者階級を正規労働者階級、既婚女性の非正規労働者階級であるパート主婦、パート主婦以外の非正規労働者階級であるアンダークラスに3分し、また専業主婦、その他無職を区別した。  三大都市圏全体の階級構成は、資本家階級3.3%、新中間階級47.4%、労働者階級41.6%(うち正規労働者20.5%、アンダークラス8.9%、パート主婦12.2%)、旧中間階級7.7%だった。三大都市圏を比較すると、東京圏で新中間階級、名古屋圏で正規労働者階級とパート主婦、京阪神圏でアンダークラスがやや多かった。  階級間格差についての詳細は当日発表に譲るが、ここでは新型コロナ感染症によるインパクトの階級による違いについて触れておきたい。感染を経験した人の比率は、資本家階級と正規労働者階級で高く、他の階級で低かった。しかし症状があったにもかかわらず検査を受けなかったという回答はアンダークラスで2.4%(全体1.7%)と多く、これを感染者に加えると、ほぼ正規労働者階級並みとなった。収入が減ったとする回答は旧中間階級(40.5%)、ついでアンダークラス(27.5%)で多く、新中間階級(13.3%)で少なかった。同様の傾向は他の設問でも確認でき、新型コロナによるインパクトに階級による違いがあったことは明白だといえる。

報告番号157

三大都市圏における格差拡大の進行過程とその社会的帰結に関する研究――(2)三大都市圏の空間構造と各地域類型の特徴
早稲田大学大学院 平原 幸輝

都市社会学においては、各都市圏の空間構造を明らかにしようとする研究が蓄積されてきた。東京圏や大阪圏を中心に、各都市圏の空間構造を、社会地図などを用いて解明しようとする研究が行われてきた中で、東京圏・名古屋圏・京阪神圏という三大都市圏を対象として、その空間構造を示し、比較・検討することが、本研究プロジェクトの課題である。  これまでの東京圏を対象とした研究などにおいては、市区町村単位での東京圏の空間構造、地域メッシュ単位での東京圏の空間構造を解明する研究が行われてきた。本研究では、人々の生活する地域の単位として、小地域を単位とした社会地図を作成した。この際、質問紙調査においては人々が生活する小地域を尋ねているために、小地域単位の地域データと質問紙調査の結果をドッキングさせることが可能になる。こうしたことを踏まえた上で、本研究においては、クラスター分析を実行し、小地域を各地域類型に分類した。  本研究においては、まず小地域単位の社会地図を作成するために、地域データを用いて、三大都市圏の空間構造を明らかにした。東京圏においては、都心に高所得層の多い地域類型が位置し、その周囲に人口密度の高い地域類型が位置していた。名古屋圏においては、名古屋市の周囲に第三次産業従事者比率が平均的な地域類型が位置し、その周囲に第二次産業従事者の多い地域類型が位置していた。京阪神圏においては、大阪・神戸・京都市という中心市に人口密度の高い地域類型が位置していた。  こうして得られた地域類型について、地域データ、及び、質問紙調査の結果をドッキングさせた。その結果、各地域類型の社会経済的特徴に加えて、地域住民の住民がどういった状況にあるのかも示唆された。例えば、人口密度の高い地域類型は、人口密度が高く、ブルーカラー層が少ないという社会経済的特性がある中で、地域住民においては近所付き合いが少なく、コミュニティに参加する機会もあまりないという地域住民の実情が示唆された。  本研究においては、小地域単位の社会地図を作成することによって、三大都市圏の空間構造を示した。その上で、各地域類型の特徴を、地域データだけでなく、個人を対象とした質問紙調査データも踏まえながら、示した。小地域を各地域類型に分類したことによって、人々の居住地を各地域類型に分類することが可能になり、質問紙調査データとの統合が可能になるわけであるが、これによってマルチレベル分析の礎が構築されるなどの意義も有するようになったと考えられる。

報告番号158

三大都市圏における格差拡大の進行過程とその社会的帰結に関する研究――(3)健康格差の要因分析:社会地区分析と質問紙調査の統合
早稲田大学 浅川 達人

【目的】本報告の目的は、社会地区分析の結果と質問紙調査(三大都市圏調査)の結果とを統合して作成したデータセットを用いて、マルチレベル分析により個人レベルの要因と地域レベルの要因を弁別することで、居住地域の社会空間構造が個人の健康観におよぼす影響を考察することにある。個人の健康状態には、個人レベルの要因のみではなく社会経済的要因が関連していることが指摘されて久しいが、地域レベルの要因の解明は進んでいない。本報告では、地域レベルの要因を社会地区分析によって析出することを試みる。 【データと方法】三大都市圏調査は2022年1〜2月に、WEB調査として行った。調査会社の登録パネルを対象として調査を行い、43,820名から有効回答を得た。WEB調査から得られた個票データに、回答者の居住地の市区町村コードと小地域コード、および小地域を表章単位とした社会地区分析から得られたクラスター所属(地域類型)を追加してデータセットを作成した。  マルチレベル分析の従属変数は主観的健康である。独立変数は、個人レベル変数としては、属性および所属階級、社会関係資本、健康関連指標を、地域レベル(市区町村単位)変数としては、小地域を表章単位とした社会地区分析から得られたクラスター所属(地域類型)を用いた。詳細は第2報告において報告するが、小地域を表章単位としてK-means法クラスター分析を行った結果、13クラスターが最適解として得られた。このうち十分な回答者が得られなかった1クラスターを除く12クラスターを地域レベル変数としてマルチレベル分析を行なった。 【結果】はじめに個人レベルの要因分析を行なった。属性については、男性および年齢が高いほど健康状態が悪いと認知していた。階級については、労働者階級に比べて資本家階級は健康状態が良いと認知しており、一方、新中間階級とアンダークラス、無職において健康状態が悪いと認知していた。社会関係資本については、町内会・PTA・趣味の集まりに加入している人は健康状態が良いと認知しており、一般的信頼感・互酬性規範などを有している人ほど健康状態が良いと認知していた。健康関連指標については、BMIが高い人、抑うつ尺度得点が高い人は健康状態が悪いと認知しており、健康習慣がある人は健康状態が良いと認知していた。  これらの個人レベルの変数の影響をコントロールした上でも、地域レベル変数であるクラスター所属(地域類型)によって、主観的健康が有意に異なることが示された。「単独世帯集中地域」に比べて、「ホワイトカラー単独世帯集中地域」「核家族世帯集中地域」「高所得層集中地域」では健康状態が良いと認知されていた。 【考察】マルチレベル分析の結果、個人レベル変数については、個人の社会関係資本および健康関連指標の影響をコントロールしてもなお、主観的健康には階級間格差が見られることがわかった。また、これらの個人レベルの変数の影響をコントロールしてもなお、健康感には地域間格差が見られ、社会地区分析から得られた地域類型によって地域間格差が説明されることが示された。

報告番号159

三大都市圏における格差拡大の進行過程とその社会的帰結に関する研究――(4)三大都市圏のアンダークラスに関する基礎的分析
東京大学 佐藤 香

1. 本報告の目的と方法  本報告では、三大都市圏データをもちいて、アンダークラスの地域的・空間的な分布に着目し、各都市圏のアンダークラスにみられる共通性と差異を明らかにすることを目的とする。佐藤・橋本(2020)で指摘したように、アンダークラスには「未婚女性型」「若年男性型」「シングルマザー型」「下流老人型」の4つの類型が想定される。アンダークラスの分布および、類型の分布が都市圏によって異なっているのかに焦点をあてることとする。なお、本報告では分析を60歳未満に限定するため、「下流老人型」の析出はおこなわない。 2. 基礎的分析の概要 (1) まず各都市圏の階級構成をみると、いずれの都市圏でも男性よりも女性でアンダークラス比率が高い。東京圏では男女とも炉心からの距離圏と階級構成に0.1%水準で有意な関連性がある。名古屋圏では男性のみ1%水準で有意な関連性があり、他の都市圏よりも労働者階級が多く、とくに10km圏外に多い。大阪・京都・神戸という複数都心をもつ京阪神圏では女性のみ5%水準で有意な関連性があり、10km圏内に新中間階級が多く居住している。 (2) アンダークラスに着目すると、東京圏では距離圏と性別に5%水準で有意な関連性があり、男性と比較すると女性アンダークラスは都心に近い距離圏に居住する傾向がみられる。名古屋圏では男女とも20km圏内と30km圏外に多い。京阪神圏では男女とも20km圏内に集中する傾向がみられる。 (3) アンダークラスの年齢構成は三大都市圏に共通しており、20歳代と30歳代が4割強、40歳代と50歳代が6割弱を占める。 (4) アンダークラスの学歴構成についても三大都市圏で共通している。男性アンダークラスでは半数が四大以上、女性アンダークラスは「高校まで」「短大・専門」「四大以上」に三分されている。 (5) アンダークラスの現職は都市圏によって異なる。男性アンダークラスについては、いずれの都市圏でも生産・技能職が最も多いが、東京圏では30%、名古屋圏では46%、京阪神圏では35%となっている。女性アンダークラスの現職で最も多いのは事務職であるが、東京圏40%、名古屋圏32%、京阪神圏37%で、逆に生産技能職は東京圏9%、名古屋圏19%、京阪神圏9%となっている。なお、初職についてみると、男女とも現在のアンダークラスの半数は正規雇用であった。 (6) 配偶関係をみると、男性アンダークラスでは7割強が未婚、2割強が既婚で、いずれの都市圏でも共通する。一方、女性アンダークラスでは都市圏によって配偶関係の分布が異なる。東京圏は未婚が多く、名古屋圏では離死別が多い。京阪神圏は中間的といえる。 (7) 同居家族数の分布は三大都市圏で共通しており、男女とも最も多いのは単身(男性37%、女性38%)であるが、性別・年齢によって異なっている。20歳代男性では3人家族が41%を占めるが、20歳代女性では31%と、男性よりも少なくなっている。

報告番号160

三⼤都市圏における格差拡⼤の進⾏過程とその社会的帰結に関する研究 ――(5)アンダ―クラスのメンタルヘルスの規定因:非正規の基幹労働化
東北学院大学 片瀬 一男

【1.目的】本研究では、大都市アンダ―クラスのメンタルヘルスを悪化させる要因をその仕事の条件から明らかにする。1990年代初頭のバブル経済の崩壊に始まった雇用状況の悪化は、世紀をまたいで続き、その後も長期化している。こうした中で、非正規雇用からキャリアを始めた者は、その脱出困難性ゆえにそのまま中高年期を迎え、「非正規雇用の基幹労働化」が進行しつつある。この状況が「高要求度-低コントロール」という「高ストレイン」な仕事の条件を生み出すと考えられる。 【2.方法】データは連携する報告とおなじく三大都市圏調査を用いる。仕事の条件の測定には、Karasekら(1990)のJob-Demand-Controlモデルの日本版を用いた。これは、仕事に対する要求度と仕事の自己裁量から仕事のストレインを測定するもので、「高要求度-低コントロール」という条件がもっともストレインを生むという仮定にもとづいている。メンタルヘルスは抑うつ尺度であるKessler et al, (2002)によるK6の日本版(Furukawa et al, 2008)を用いた。なお、K6のスコアは、尺度値を変換・合算して連続変数として用いた(値域は0~24)。また、「非正規雇用の基幹労働化」という分析課題に照らして、分析対象を35~59歳男性に絞った。 【3.結果】まず非正規雇用の基幹労働化の実態をみるために、階級別に週労働時間が法定労働時間(40時間)を超える者の比率を求めたところ、アンダークラスではその比率は他の階級より少ないものの、東京圏では26%、名古屋圏では28%さらに京阪神圏では33%が週40時間を超えて働いており、量的基幹労働化が部分的に生じていることがわかった。他方、JDCモデルの2つの仕事の条件をみれば、まず仕事の要求度については、いずれの都市圏でも、他の階級が尺度値で8点台であるのに対し、アンダークラスは東京圏で7.1,名古屋圏6.9,京阪神圏で7.3と他に比べて低くなっていた。仕事のコントロールはどの都市圏でも階級ラダーが下がるにつれ低下し、アンダークラスではもっとも仕事の裁量度が低くなっていた。最後に本研究の従属変数となる抑うつ度(K6スコア)について、都市圏別・階級別に記述統計量を見ると、いずれの都市圏でもアンダークラスの抑うつ度が最も高く、東京圏で7.1、名古屋圏で6.9、京阪神圏で7.3となった。 そこで、この抑うつスコアを従属変数にして、3つの都市圏別に階層的重回帰分析をおこなった。まずモデルⅠは年齢と階級ダミー(基準は労働者階級)による基準モデルである。東京圏ではすべて変数が、また名古屋圏では資本家階級ダミーを除く変数が有意となった。また、京阪神圏では年齢と新中間階級ダミーのみが有意な負の効果もった。ここで注目されるのは、京阪神圏以外の2つの都市圏でアンダークラスであることが抑うつに有意な正の効果をもったことである。モデル2では、ここに要求度/コントロール比を入れたところ、これが有意な負の効果をもち、東京・名古屋圏のモデルⅠで有意だったアンダークラスの効果が消えている。ここからみて、仕事の要求度に応じたコントロールがある場合は抑うつが軽減されるが、アンダ―クラスはこの要件を欠いていたためにメンタルヘルスを悪化せていることになる。 【4.結論】本研究の分析から階級とメンタルヘルスをつなぐ経路として仕事の要求度-コントロールを仮定するJDCモデルの有効性が確認された。

報告番号161

三大都市圏における格差拡大の進行過程とその社会的帰結に関する研究――(6) 大都市公共圏の階層性と社会関係資本の機能
名古屋大学 丹辺 宣彦

【研究の背景・目的】  日本の三大都市圏で住民の日常的な集合行為はどのように異なり、それはまたどのような階層性、地域性によって規定されているのだろうか、このような研究課題に大規模な広域データを用いて検討した例は少ない。またとくに社会関係資本の量と都市度により、集合行為の種類と参加水準がどのように異なってくるのか、この点も日本の都市社会・地域社会の公共性の特質として興味深いと同時に研究上の空白が多い課題なので、三大都市圏調査データを通じて検討してみたい。 【方法】  調査項目に入れた10種類の社会活動に参加した割合(最近1年以内)を都市圏別に比較しその特徴をみる。また、社会活動のジャンルを「地縁型」の活動と「テーマ型」の活動に大別し、参加の規定要因、とくに階級性を多変量解析により明らかにする。その際に、とりわけ社会関係資本の所有とその規定要因に注目したい。さらに、都市度のちがい、社会関係資本の所有量により、地縁型の活動と地縁型活動の活動水準に差が出るかどうかを検討する。 【分析結果の考察・結論】  本調査のデータでは、同一項目を用いた先行研究に比べて地縁型活動の参加率が相対的に大きく低下していて、コロナ感染問題の影響の大きさが窺えた。そうしたなかでも都市圏別での比較からは、名古屋圏、とくにその郊外都市圏で地縁型活動への参加が相対的に高く、地縁的紐帯の相対的強さ、定住性の高さが反映されていた。  大都市圏全体で社会活動参加の規定要因についてみたところ、単純なモデルでは資本家階級と中間階級への所属が促進要因となっているように見えたが、いくつかの社会関係資本の所有という変数を入れたモデルではその効果がほぼ消失する。しかし他方で、社会関係資本自体の所有に対しては二つの所有型階級への所属の促進効果がさまざまな変数でみられた。したがって、所有にもとづく階級に所属するか否かは、弱いながらも間接的に社会活動参加を通した自己実現にも影響を与えていると言えそうである。  集合行為については「地縁型」の活動は人口密度で測った都市度が低いと高くなり、「テーマ型」活動では違いがみられなかった。個々の住民がもつ社会関係資本の量は地縁型はもちろん、テーマ型の活動に対してもおおむね促進効果を示していた。報告では、こうした点について、R. ペッカネン(2008)の言う「地域的市民社会」、R. パットナムのSC論、C. S. フィッシャーの下位文化論と関連させながら検討してみたい。

報告番号162

日本における企業と組織の社会学再考
日本学術振興会 園田 薫

【1. 目的】  本報告の目的は、現代日本の社会学において企業と組織を主体として設定した社会学的論考がどのように営まれているのかを検討し、今後の発展可能性について考察していくことである。本報告では主に企業を対象とした組織研究と組織に対する理論的考察から出発し、どのように企業と組織に関する視座が社会学のなかで培われてきたのかを俯瞰したい。 【2. 方法】  本報告では、主に企業や組織について扱ってきた既存研究を領域社会学の垣根を超えて渉猟し、そうした研究が海外の研究群とどのような影響関係にあったのかを意識しつつ、時系列的にその研究史を検討するという手法をとる。 【3. 結果】  日本における企業や組織の研究は、主に産業社会学・経済社会学・経営社会学・組織社会学といった領域を中心に展開してきた。産業社会学ではメイヨーやロスリスバーガーなどのハーバード学派によって展開された人間関係論が下地となって、主に経営者と労働者の関係性を円滑に進めるための可能性と労務管理のあり方を模索してきた。これはブリーフスやゲッグなどのドイツの経営学を中心に発展した経営社会学の思想と近く、日本において経営社会学と産業社会学は同じく経営者中心的な思想的立場をとるものだと理解されてきた。この認識は、両領域と経営学の領域設定を曖昧にし、その社会学的な立場を潜在化させる要因となった。  この領域における議論が縮小する一方で、アメリカ社会学における流行を受けて議論を拡大しているのが、経済社会学と組織社会学である。ウェーバーやジンメル、スメルサーなどの経済現象に対する理論的考察をマクロなシステムとの関連において拡張したのが経済社会学であったが、近年はグラノヴェッターを旗手として、人々が埋め込まれた社会関係に着目して経済現象を読み解こうとする新しい経済社会学の影響が色濃くみられるようになる。また組織社会学は、ウェーバーやブラウ、セルズニックらによって論じられてきた組織構造に関する旧制度派の議論から展開し、システム理論などの純理論的考察をその射程に含めながら拡大してきた。のちにマイヤー・ローワン、ディマジョ・パウエルなど、アメリカを中心として環境への認知が組織の制度化を方向づけるとする新制度派の組織理論が流行すると、日本の社会学においてもその理論的な輸入が進められた。 【4. 結論】 日本における企業と組織の社会学的論考は、産業・経営社会学という領域から経済・組織社会学という領域へと関心が移行しているようにみえる。これは、両領域が経済学・経営学や既存の社会学的知見との差異化を図った〈新しい〉経済社会学・組織社会学の勃興というアメリカ社会学の影響下にあって、企業や組織の研究に新たなブレイクスルーがもたらされたことを意味する。しかしそうした真新しさのみに社会学の認識利得があるわけではない。この点は、経営学や組織論において社会学と通底する学問的視座を有した理論・実証研究を参照するなかで反証していく。

報告番号163

構造的空隙理論の最先端における論点整理及び新概念の導入――Burtによる構造的空隙理論への新概念の導入に対する理論的考察
関西大学 安田 雪

本報告においてはバート(Burt, Ronald S.)の「構造的空隙理論」(1992)の現時点における展開と発展について概括する。  構造的空隙理論は「橋渡し」か「結束」かという対立概念、あるいは「密か疎か」という比較概念して我が国で対比的に論じられ、この比較構造や対立構造を軸に応用研究がなされてきた(安田,2011)(若林,2009)(金光,2020)。構想的空隙理論とその実証研究は、イノベーション、アントレプレナー研究にも展開され、世界標準の経営理論の重要な32分野のうちの一分野として紹介されているなどの広がりをみせている(入山,2019)。  だが、現在、バート自身の空隙理論は、橋渡しか結束かという概念をこえて発展している。今、バートの理論において注目すべき重要なのは評判とタイミングという二つの概念と構造的空隙理論との結びつき、そして、応用研究の蓄積の進展である。構造的空隙理論と、評判とタイミングという概念が組み合わせて用いられる実証研究は世界でも稀少である。近年のバートの理論においては、評判の概念は、結束か橋渡しかを選ぶ際の、選択の合理性を説明するために用いられる。なぜ、いずれを選択するのかの、説明のメカニズムに評判の概念が使われるだが、このロジックには若干のデータ解釈の強引さと危うさもある。だが、これは国際的なビジネスマンの人的ネットワーク調査のデータベースをシンガポールを拠点に構築・運用する、マーティン・ガウジーロ(Martin, Gauguilo)の保持するデータ及び彼との共同研究から計量的に得られた知見でもある。  一方、タイミングの概念は、結束か橋渡しかを、いつ、どのような時に、誰が、選択すべきかという選択を説明する。研究目的は社会的な規範論を解くことではない。目的関数が特定企業内の社員や、企業間の相対的な優位性であるので、社内における競争的人事、あるいは企業間競争が対象となり、タイミングにより正しい解は異なるというのが、両概念の用いられ方の特徴である。もちろん、行為の説明だけではなく、実証データにもとづいて、タイミングが計られるが、このタイミング理論を構築するためには、当然長期にわたる時系列データが必要となる。バートの構造的空隙理論が1992年からタイミング問題に発展するために、膨大な時間がかかった所以である。  さらにネットワーク分析とゲーム理論とのつながりについても、チェ(2022)が大胆な展開をし、数学と数字に依存しない、目線と視認などのつながりを事例にゲーム理論の政治学的な構築を示唆するなど、新しい理論枠組みの再構築を感じさせる動きもある。最後に、現時点で進行している国内におけるネットワーク調査上の具体的な限界や経済的・社会的制約条件の存在について言及する。 <参照文献> Burt, Ronald Stuart (1992) Structural Holes, Harvard University Press. マイケル・S-Y,チェ (著) 安田雪(訳) (2022) 『儀式をゲーム理論で考える』みすず書房. 入山章栄(2019)『世界標準の経営理論』ダイアモンド社. 金光淳 (2020)『「3密」から「3疎」への社会戦略——ネットワーク分析で迫るリモートシフト』明石書店. 安田雪 (1997) 『ネットワーク分析』新曜社. 安田雪 (2011) 『パーソナル・ネットワーク』新曜社. 若林直樹(2009)『ネットワーク組織―社会ネットワーク論からの新たな組織像』有斐閣.

報告番号164

組織論としてのシステム論
福岡大学 樋口 あゆみ

1. 目的  本報告はニクラス・ルーマンの組織論について、1980年代以降の議論を意思決定論と境界の議論に焦点を当てて整理する(Luhmann 2000)。それによって一部では「古典的」と誤解されることもあるルーマンの組織論の現代的意義を明らかにする。 2. 方法  2018年から2022年にかけて組織論文集„Niklas Luhmanns Schriften zur Organisation“(2018ほか)が6分冊で相次いで発刊され、その議論への注目が高まっている。本報告では国内外の関連研究を参照しつつ(Nassehi 2005; Cooren and Seidl 2020; 奥山 1986; 佐藤 2011; 長岡 2012)、組織システムの開放性が重視されるにつれて問題化されてきた、「組織をどのように捉えるか」という現代的な課題に対して理論的に検討する。 3. 結果  システム論においてシステムは要素が要素を生むものとして定義され、なかでも組織システムは組織の意思決定を要素とするシステムとされる。ハーバート・サイモン(Simon 1997)をはじめとする従来の意思決定論と顕著に異なるのは、その時間と境界の扱いである。ルーマンは意思決定の概念を流れゆく時間と結びつけ、さらに多数の意思決定が連鎖する過程で組織システムが作動的に閉鎖していくと捉えた。そうすることで、組織の開放性と閉鎖性が記述可能になった。 4. 結論 近年ではヴァーチャル組織や、オープン・イノベーションなど、従来考えられてきた組織境界を越えるような組織行動が数多く見られ、また推奨されている。そのような現代的組織行動を、サイモンのように組織境界の問題を不問にするのではなく正面から扱うには、上述のように組織システムの開放性と閉鎖性を同時に扱えるような理論が必要である。 【参考文献】 Cooren, François, and David Seidl. 2020. “Niklas Luhmann’s Radical Communication Approach and Its Implications for Research on Organizational Communication.” Academy of Management Review 45 (2): 479–97. Luhmann, Niklas. 2000. „Organisation Und Entscheidung”. Wiesbaden: Westdeutscher Verlag. ———. 2018. „Schriften zur Organisation 1.” Edited by Ernst Lukas and Veronika Tacke. Wiesbaden: Springer Fachmedien Wiesbaden. Nassehi, Armin. 2005. “Organizations as Decision Machines: Niklas Luhmann’s Theory of Organized Social Systems.” Sociological Review 53 (1_suppl): 178–91. Simon, Herbert. 1997. ”Administrative Behavior.” 4th ed. New York: Free Press. 佐藤俊樹. 2011. 『社会学の方法』. ミネルヴァ書房. 奥山敏雄. 1986. 「組織を捉える視角と装置」『ソシオロゴス』10: 120–42. 長岡克行. 2012. 「意思決定とは何をどうすることか?」『東京経大学会誌』276: 65–77.

報告番号165

労働をめぐる時間的秩序をいかにして記述するか――ドナルド・ロイから「組織のプロセス理論」へ
長野大学 松永 伸太朗

【1.目的】 労働時間問題は、数々の労働問題のなかでも主要なトピックの一つであり、現在でも労働研究をめぐる社会科学のなかで研究が積み重ねられてきている。しかし、近年進行している柔軟な働き方や労働時間管理のあり方は、労働時間をめぐる問題を複雑化させており、それを捉える社会学的視点についても刷新が求められている。本報告では、労働過程論や組織社会学の議論を参照しながら、現代的な労働問題を労働社会学的に捉える視点はどのようなものでありうるかについて議論する。 【2.方法】  まず、職場における時間的秩序の編成を扱った研究として一つの嚆矢であるドナルド・ロイ(Roy 1952; 1959)の議論を出発点とし、その議論を発展させた現代的成果を検討する。さらにその課題を乗り越えうる視点を提供する議論として組織社会学者トール・ヘルネスの「組織のプロセス理論」(Hernes 2014)の知見を参照する。 【3.結果】 ロイは、職場の時間的秩序を記述するにあたって、労働者がインフォーマルに時間の区切りを設け、その区切りごとに特定のテーマをもった相互作用を行うことにまず着目した。さらに、使用者が出来高レートを引き上げることのないように時間当たりの作業量を巧妙に調整する「ゲーム」を自ら行っていた。ロイの知見は労働者がさまざまな仕事上の関心を達成するために行う実践が時間的秩序の編成とともに遂行されることを明らかにしていた。ロイに影響を受けた労働社会学研究はロイの時代よりも労務管理が厳格化した状況化での時間的秩序の編成を解明したが、そこで描かれる実践の内実は多岐にわたっており、理論的な見通しを立てにくい状況が生じていた。 「組織のプロセス理論」を提唱したヘルネスは、組織現象が生じるうえでの不可欠な要素として時間的秩序の分節化に着目していた。ヘルネスは、組織の同一性を支える意味構造に時間的幅をもった「歴史性」が含まれる場合があり、そのような歴史性を帯びた意味構造を「出来事」として定式化していた。そのうえで、間主観的分節化・実践的分節化・物質的分節化・テキスト的分節化・暗黙的分節化の5つの分節化の類型を提示した。ヘルネスの分節化のタイプを用いてロイや後続研究が扱った労働者の実践を整理することができ、その整理に基づいてさらなる職場の時間的秩序をめぐる研究の発展の方向性を展望できることが明らかになった。 【4.結論】 ロイらの時間的秩序の編成に関する議論をヘルネスの類型を用いて編成を整理することによって、以下の3点の方向性が見いだされた。第一に、タイプごとに適した調査方法論があると考えられることである。第二に、とくに物質的分節化の水準は盲点となっておりアルゴリズムを用いた労務管理などの検討において重要になると考えられることである。第三に、タイプごとに生じている労働者の困難を理論的区別しながら論じることによって労働社会学における労働批判の有効性の向上が期待できることである。 参考文献 Hernes, T., 2014, A process theory of organization, Oxford: Oxford University Press. Roy, D., 1952, Quota restriction and goldbricking in a machine shop. American Journal of Sociology 57(5): 427-442. ————, 1959, Banana time: Job satisfaction and informal interaction. Human Organization 18(4): 158-168.

報告番号166

日本におけるギグワーカーの組織化
早稲田大学 申 在烈

【1.目的】世界的にプラットフォーム産業の成長と共に、配達産業で働いているギグワーカーの成長が目立つ。しかし、プラットフォームで働いているギグワーカーは雇われている労働者でなく独立自営業者であるため、企業の立場からみると、雇用保険、年金、健康保険などを払う必要がないうえ、雇用維持の義務もない。結果的に比較的に雇用安定性が高い日本型雇用慣行の軌道から大きく離脱するため、プラットフォーム労働者はリスクから無防備となる。このような状況の中で、プラットフォーム労働者の労働権を保護するためには法制度の整備に加え、組織化が必要である。しかし、海外諸国に比べて日本では組織化がほとんど行われていない。その理由は何か。本研究では配達産業を中心に日本のプラットフォーム労働・産業の現状を紹介した後、日本においてプラットフォーム労働者の組織化が遅れている理由を探索する。 【2.方法】① 労働過程論と比較制度分析等に基づいて、日本においてプラットフォーム労働者の組織化が遅れている理由を探索する。特に本研究では比較的にプラットフォーム労働者の組織化が進んでいる韓国を中心に国際比較を行う。②現場の配達員を対象にヒアリング調査を行い、彼女・彼らのウーバーイーツユニオンに対する態度と意見を確認しる。また、配達員以外にも、研究者、組合関係者、配達員委託企業関連者などを対象にインタビューを行った。インタビュー調査によって得られたデータを用いて配達員が労働組合活動に参加しない理由を明らかにする。 【3.結果】プラットフォーム労働者の組織化に関連して特に目立つ国は韓国である。韓国では従来の組合が積極的にプラットフォーム労働者の組織化に資源と人力を投入している。また、市民社会も組織化を積極的に支援している。その結果、配達産業ではプラットフォーム企業とギグワーカー組合の間で団交が行われ、協定書の締結も行われた。日本でも一部の配達員が集まってウーバーイーツユニオンを立ち上げ、ウーバーイーツユニオンの設立に当たって外部からの支援も受けた。しかし、韓国等とは違い、日本のウーバーイーツユニオンは外部からの支援を受けているにもかかわらず、2019年の設立以来、組合員の人数にはほとんど変化がない。実際に配達員たちもウーバーイーツユニオンに対する意見を訪ねると、否定的な見解を示している。インタビュー調査の結果をまとめると、配達員が組合に賛同しない理由の一つは複数のプラットフォームを自由に利用できるということである。しかし、実はこれは韓国でも同様である。配達プラットフォームのシステムを日韓比較すると、韓国では会社で評価、勤怠管理等に関するシステムを構築し、これをもって労働者を巧妙に統制している。いわゆる労働過程論でいう制度的統制といえる。しかし、今のところ日本では人手不足という状況と組み合って市場による統制だけが行われている。このような会社の配達員に対する統制システムの違いが労働者の組合活動に関する態度にも影響を与えているといえる。 【4.結論】新しい産業で組織化が行われるためには、制度の整備だけでなく、行為者の積極的な参加が重要である。また、行為者の態度は社会全体の雇用制度、労働市場における需要と供給、労働過程によって異なる。 【謝辞】本研究は、日本学術振興会科学研究費補助金(20K20787)の支援を受けた。

報告番号167

1990年代の日本石炭産業における現場職制と労働者の関係について
早稲田大学 清水 拓

1960年以前の日本石炭産業においては、現場の経営権が労働組合による職場闘争を通じて蚕食されていたことが知られている。平井陽一(2000)は、三井三池炭鉱を対象にその実態解明をおこなった。その結果、部分的な機械化による作業速度・労働強度のアンバランスを是正し、現場職制による恣意的で偏った番割を排除するために、労働者が輪番制と生産コントロールを通じて職場を管理していたことが明らかになり、それを労働者的職場秩序と呼んだ。1960年前後の三池争議を端緒とする全国的な合理化によって、現場の経営権は会社側によって奪還された。しかし、1960年代後半においても、実際の現場運営の場面では、経験豊富な熟練労働者が指導的な役割を果たしており、現場職制の職務は国家資格保有者として作業にあたる発破と保安のみに留まったことが、市原博(2012)によって明らかにされている。政府主導のエネルギー政策とともにあった日本石炭産業は、2002年1月に最後の大手炭鉱であった太平洋炭砿が閉山し、同年3月に一連の石炭政策が終了したことで終焉を迎えた。先行研究で明らかにされた現場職制と労働者の関係は、その後の30年間にどのように変化したのか、あるいは変化しなかったのか。島西智輝(2011)によって技術の革新性と労働の伝統性の共存と評された戦後日本石炭産業について、その最終段階の生産職場のありようを明らかにしたい。そこで本報告では、日本石炭産業の最終的撤退期である1990年代における生産職場について、太平洋炭砿の機械化採炭現場をとりあげ、そこで問題化していた「腹遅れ」という事象をみる。同事象は、採炭現場の自然条件悪化をもたらすため、現場運営上は回避すべきことであったが、人為的に引き起こされることがあった。それは採炭現場ごとの出炭の代理指標であった「延び」を水増しすることで生じた。同鉱の労働者の賃金体系は固定給であり、延びが賃金に反映されるようなインセンティブは存在しなかった。それでも、労働者たちは三交替制のもとで自らのプライドをかけて他チームと延びを競い合った。現場職制も延びをとることにこだわった。延びが人事評価と直結したためである。そして、そのプレッシャーから、なかには延びの水増しを企図してその計測点のみを余分に採掘するよう指示する者さえいた。しかし、その逸脱行為の指示に対して労働者は葛藤を強いられた。彼らは「ちゃんとした仕事」をするかどうかというインフォーマルな相互評価を重視するため、腹遅れによって現場を荒らすことには抵抗があった。現場職制からその後に報復的な扱いを受ける可能性を懸念しつつ、指示に従ったり背いたりした。本報告では、自然条件が常に変化するという生産職場の不確実性の高さを踏まえつつ、腹遅れという事例から、生産職場における現場職制と労働者の関係について検討したい。 【文献】 平井陽一,2000,『三池争議――戦後労働運動の分水嶺』ミネルヴァ書房. 市原博,2012,「戦後炭鉱職員の職務・教育資格・人事管理」杉山伸也・牛島利明編『日本石炭産業の衰退――戦後北海道における企業と地域』慶應義塾大学出版会,157-90. 島西智輝,2011,『日本石炭産業の戦後史――市場構造変化と企業構造』慶應義塾大学出版会. 【謝辞】 本研究はJSPS科研費20K22152「日本石炭産業の技術的到達点における炭鉱労働者の熟練に関する研究」による成果の一部である.

報告番号168

住宅は地域社会の共同性構築の場所になり得るのか――革新市政期横浜市から考える
神奈川大学 青木 淳弘

【1.目的】 我が国における賃貸住宅への入居に対する支援は不十分であり、低所得者は十分な居住水準を確保できない可能性が高くなる。住宅の確保が個人の資力に依存する現在の日本において、賃貸住宅は基本的に特定の個人や家族の利用に閉じられ、コモンズとして開かれているとは言えない。他方で1970年代は、日本全国の都市において社会福祉の拡充を掲げた革新自治体が隆盛し、住宅に関連する問題への対応についても盛んに議論がなされていた時期であった。本報告では、政府レベルでの持ち家取得の推進と、全国規模での公営住宅の供給の不足という背景において、当時の革新自治体は賃貸住宅をどのように位置づけようとしていたのかについて検討したい。 【2.方法】  本報告では、飛鳥田革新市政期(1963~1978)の横浜市を対象として、住宅政策の位置づけと特徴を明らかにする。横浜市は急激な人口流入や社会資本の整備といった高度経済成長期における典型的な都市問題が著しかったことから、地方自治体レベルでそうしたことがどのように住宅問題と結びついていたのかを考える上で重要な地域である。資料としては主に市の定期刊行物である『横浜市調査季報』を用いて、住宅問題をテーマとした記事について分析を行った。本資料はそのテーマの担当部署の職員による見解や調査結果などが豊富に掲載されており、時代ごとの自治体内部の事業とそれに対する方針や実践などを内在的に考える上では欠かせない資料である。 【3.結果と結論】 高度経済成長の横浜市は、公的な賃貸住宅の質と量の拡充にあたり、地価の高騰と超過負担という二重の問題を抱えていた。政府セクターは応能家賃方式や公的な賃貸住宅の建設のための補助金の導入を検討したが、実際には不十分な内容となった。そうした背景において、横浜市にできたことは、住宅問題を「宅地」の問題と読み替えて、宅地開発要綱の運用によって公共公益用地の確保を行うことであった。その先には社会福祉の実現としてのコミュニティ開発が志向されていたが、実際にはそれが具体的な事業や政策に結びついていくことはなかった。さらに高層高密な開発の広がりと要綱行政運用上の問題によって、横浜市飛鳥田市政において「コモンズとしての住宅」に結びつくような構想が発展的に継承されることはなかった。本事例が示唆するのは、2017年の改正住宅セーフティネット法に規定されるような「居住の支援」の更なる充実を俟つほか、持家偏重型の政策からの抜本的な改革なくして、賃貸住宅が広く開かれたものとなるのは難しいということである。

報告番号169

都市コモンズ研究における課題の整理 ――「コモン化する(commoning)」実践に注目して
東京大学大学院 桐谷 詩絵音

本研究の目的は、社会科学における従来のコモンズ研究から、都市社会学的な都市コモンズ研究までの蓄積を整理・検討したうえで、とくに都市コモンズをめぐる議論において、コモンズそのものが成立する局面に注目し、そのメカニズムを明らかにする必要性を指摘することである。  G. Hardinによる有名な「コモンズの悲劇」の議論と、M. Olsonによる集合行為論を発展させたE. Ostromのコモンズ論は、コモンズの維持管理をめぐる複雑な過程を、主体が資源にたいして行使する多様な実践におうじて整理・解明したものとして評価することができる。しかし、これらの主流派のコモンズ論がおもな議論の焦点としていたのは、農村的な場における伝統的共同体であり、すでに存在する資源についていかに持続的に維持・管理することができるかがおもな問いとなっていた。  いっぽう、この農村的なコモンズ論を都市という空間において応用する試みについてもさまざまな議論が蓄積されてきた。都市のコモンズを論じるさいには、すでに存続してきた伝統的なコモンズだけではなく、あらたな資源をめぐるコモンズが生成する局面も射程に収める必要がある。  都市におけるこのような局面に迫っていると考えられるのが、D. Harveyによる都市コモンズ論における「コモン化する(commoning)」実践の概念である。ここで都市コモンズは、政府や企業によって提供・管理される「公共財」を基盤とし、そこに人びとの「政治活動」や「日常的な活動や闘争」の諸実践が加えられることで、生成するとされている。いいかえれば、都市を生きる人びとの実践において形成される存在として、都市のコモンズをとらえる議論である。  しかし、それではこの「諸実践」がどのような実践を指すのか、つまり具体的にどのような人びとの実践において、所与の公共財が都市コモンズへと転換されるのかについては、Harveyも明確に論じてはいない。「コモン化する」実践とは、かならずしも特殊な運動形態をとるとはかぎらず、人びとの日常生活における日常的実践において生成することが示唆されるにとどまっている。  すなわち、都市コモンズ論においては、既存のコモンズが日常生活において維持・管理されていくメカニズムだけではなく、コモンズが人びとの実践をつうじてあらたに生成していくダイナミズムをとらえることが、課題として残されているといえる。  なお、コモンズを生成する人びとの実践を明らかにするうえでは、焦点となる資源を具体的な物体としてとらえ、その物体が人びとの実践のなかで資源として発見・確立されていく過程も重要な論点となりうる。実践において流動的に成立する資源の姿への着眼をひらく先行研究としては、「社会的共通資本」をめぐる議論があげられる。ここから、どの主体がどの物体にたいしてどのような実践を行使するかによって、たちあらわれる資源とそのコモンズのあり方も流動的に変化しうることが示唆される。

報告番号170

国が「成功」と評価する地方自治体の移住政策をめぐる論理と規範――2015年以降の移住政策成功事例集の分析より
一橋大学大学院 伊藤 将人

1,目的 本報告の目的は、国や地方自治体による政策的な地方農村への移住促進が拡大する現代日本において、地方自治体によるどのような移住政策が国によって「成功」と位置づけられるのかを分析することで、国による移住政策をめぐる統治の実践とイデオロギーを明らかにすることである。2,方法 本報告では国が作成した地方自治体や民間事業者による移住政策の成功事例を掲載した事例集の表象と言説を分析する。対象資料は「移住・定住施策の好事例集(内閣官房まち・ひと・しごと創生本部事務局, 2017)」「移住・定住事例集:しあわせな移住(総務省東北管区行政評価局, 2020)」「「地方への人の流れの創出」に向けた効果的な移住定住推進施策事例集(総務省, 2021)」など主に2015年以降に発行された事例集である。この期間は国が第二次国土形成計画(2015)において「地方移住・二地域居住・二地域生活・就労等を支援するための体制を充実させるとともに、国民的な運動を展開し、積極的に促進する」ことが表明されて以降の期間であり、地方創生による上からの地域活性化・移住促進が加速した時期にあたる。3,結果 分析の結果、国は人口の社会増減率がプラスに転じた、もしくは社会減の減少幅が縮小した事例を成功事例と位置づけていた。取り組みの成果は、移住者数や新規就農者数など量的な指標の形式をとっていた。また成功事例では成果として「起業者数(創業者・新規就農者数・起業支援件数など)」が強調される傾向が確認された。4,考察と結論 起業者としての移住者の増加を成功と位置づける結果は、フーコーが新自由主義論で示した「労働者自身が、自己にとってのある種の企業[Foucault,1979=2008]」となる、物質的・金銭的な報酬以外の自己実現を重視するアントレプレナー的主体像と重なる面がある[渋谷, 2011]。アントレプレナー的主体化は誰でも可能なものではなく移住者の一部であると考えられるにも関わらず、国や地方自治体がアントレプレナー的主体としての移住者の量を成果として評価する本分析結果からは、国の地方移住促進をめぐる統治の実践における新自由主義的な論理が垣間見える。事例集を通した国が内面化する価値規範と望ましさの分析により、掲載される成功事例を通して掲載されない事例の傾向(無視される移住)が相対的に浮き彫りになった。これは国による統治の実践と移住イメージの構築性を明らかにするとともに、ある種のモビリティが体系的に珍重されることを許さない(同じ、大都市から非大都市への移動行動であっても、評価され積極的に政策対象になるものと、そうでないものがある)イデオロギーの条件を明らかにすることにつながる可能性を示唆している[Frello, 2008]。文献 Foucault, M., 1978, Sécurité, territoire, population, Cours au Collège de France (1977-1978), 2004. (=2007, 高桑和巳訳, 『安全・領土・人口――コレージュ・ド・フランス講義 1977-1978年度』筑摩書房). Frello, B., 2008, Towards a Discursive Analytics of Movement: On the Making and Unmaking of Movement as an Object of Knowledge, Mobilities, 3(1): 25-50. 渋谷望, 2011, 「アントレプレナーと被災者―ネオリベラリズムの権力と心理学的主体」『社会学評論』61(4): 45-472. 謝辞 本研究は日本学術振興会特別研究員(22J12424)の研究成果の一部である。

報告番号171

TBLとPBLを活用したまちづくり分野の教育における教育効果に関する研究
高知県立大学 秋谷 公博

中央教育審議会(2012)が取り纏めた答申「新たな未来を築くための大学教育の質的転換に向けて」では、学生が主体的に学びながら問題を発見し、その解決方法を自ら導き出していく能動的教育への転換の重要性が指摘され、その方策としてアクティブラーニングを活用した教育の重要性も指摘されている。しかし、アクティブラーニング、とりわけ課題解決型学習(PBL)とチーム基盤型学習(TBL)を教育に取り入れた研究の数自体がいまだ少なく、且つそれらの多くが看護や薬学等の医療系の分野での研究が多くみられ、まちづくり分野をテーマとしてTBLとPBLを取り入れた教育の効果等について検討した研究は見られない。  上記を踏まえ、本研究ではS県にあるK短期大学のビジネス・キャリアコース1年生の必修科目で、学生がグループ毎によるフィールドワークを主体とした調査研究を通して、地域のまちづくりの課題解決を目的とした研究の成果を地域に還元することで地域貢献に寄与することを目的としている「地域貢献プロジェクト」の事例を通して、まちづくり分野での教育におけるTBLとPBLの教育効果等について明らかにすることを目的としている。本研究の取り組みは、TBLとPBLを取り入れた地域活性化を始めとしたまちづくり分野の教育において有益な示唆を与えるものであり、研究の意義があるものと考えられる。  本研究の目的を達成するために、長年商店街の活気の低迷等の課題を抱えている6つの商店街と4つの大型店によって組織されている「S市中心市街地商店街Dモール(以下、Dモール)」を対象地域として選定し、学生による調査研究を行った。加えて、TBLとPBLを活用したまちづくり分野の教育における教育効果に関するデータの収集を目的として、アンケート調査を行った。  本研究から得られた知見として、「TBL及びPBLを活用した講義の教育効果」において、33項目中実に32項目が平均値4点満点中3点を超えていることからまちづくり分野の教育においてもTBL及びPBLを活用した教育において教育効果があったことが明らかとなった。とりわけ、①PBLによる学習、②傾聴力、③チームメンバーとの信頼関係の構築、④自己の成長、⑤講義の満足度の5項目は平均値が3.60を超えるほど非常に高く、これらの項目に関する教育効果が顕著にみられた。その他に上記の②及び④を除いた能力の向上等の点から見ていくと、①主体性、②発信力、③フィールドワークにおける実践力、④視野の拡大、⑤自信を持って発言出来るようになった等にも大きな効果が見られた。  本講義で学生が提案した案がDモールに採択され学生がサークルを設立し、同モールと協同でまちづくりに取り組んでいる。つまり、まちづくり分野の教育にTBLとPBLを取り入れた能動的教育を実践したことで、学生が講義中に改善案を考えそれを自ら実際のまちづくりの実践へと繋がっていると言える。こうしたことから、本研究から得られた知見からまちづくり分野の講義等においてもTBLとPBLを取り入れた教育の有効性が指摘できる。  以上、本研究で得られた知見は今後のTBL・PBLを取り入れたまちづくり分野の教育において、有益な示唆を与えるものと考えられる。

報告番号172

コロナ禍における中国「社区居民委員会」の二重属性とその変容――「防疫日誌」の分析を通じて
東北大学大学院 傅 昱

1.目的 中国の社区居民委員会およびそこに所属する社区ワーカーは、社区住民から信任を受けた代表でありながら、国や行政から指揮監督にも応えようとする「中間位置」にある。したがって、コロナ禍対策として展開された社区づくりを通して、担い手としての社区居民委員会は、「行政の下請け」側面が強まるとともに、住民との信頼関係のある「自治組織」という性質との葛藤が生じ、あるいはその二重性が強まっていくとも想定される。本報告では、社区づくりを記録した「防疫日誌」の分析を通じて、社区居民委員会の二重性格のあり方とその変容について明らかにする。 2.方法 本研究では福建省泉州市豊沢区に所在しているH社区の事例をとりあげ、そこに2020年2月7日から3月6日までに、コロナ禍に対応する社区づくりを記録した「防疫日誌」に対する内容分析を行った。「防疫日誌」を用いる理由は、コロナ禍で地域に接近不可能な状況では得難い防疫現場から、住民生活を居住の地区ごとに組織・動員し、しかも全国的に展開するというほかの国に見えない中国に特有のコロナ禍対策を読み解ける点にある。 3.結果 戦争メタファーの統制・動員的な作用を確認することができた。一方、社区づくりの現場においてはなおそれに収束しえない共同性や自治の側面との葛藤も生じており、いわばその二重性の更なる追究という課題が示唆された。 社区ワーカーが社区づくりの「戦争モデル」の創出・維持に果たした重要な役割には、少なくとも次の四つがあると捉えられる。すなわち、派遣された地方政府職員を含め、社区ワーカーが政府=党による「戦争モデル」の指令を受け入れる、感染症の輸入・輸出を防止のため、「社区」をロックダウンして管理する、すべての人たちに感染症対策の実施を要請するとともに、社区づくりに係る他の主体(行政、市場、市民社会)の支援を活用する、という四つが彼らにとっての「戦争モデル」を構成している、と見ることができる。この点は、先行研究の指摘にもあるように、社区ないし社区ワーカーの役割が上からの統制ないし相互監視体制を強めてゆく側面を見ることができる。一方、その枠内ではあれ、日誌には、生じた出来事や登場する人物についての丁寧な叙述がなされざるをえず、つまりこの地域の個々の事例に密着した厚い記述も見られた。少なくとも日誌の書き手において、戦争メタファーはよそ者だった当の職員を従来から居た社区ワーカーと結びつける媒介でもあった可能性を、新たな仮説として考えることもできる。こういった「二重属性」は、相互に排他的なものであるよりも、相互補強的な関係、あるいは互いに不足している部分を補う相補関係にあるかもしれない。 4.結論 結論としては、日誌分析を通して社区づくりの原理が問われる。すなわち、コロナ禍のような危機によって、住民たちが知らぬ間に危機とその対応に巻き込まれる一方、社区ワーカーは最初から行政指令のもとで行動するけれども、それは住民たちの自立的かつ自発的に行動する意識を喚起しながら、多元的な主体を組織し、動員し、協働することで、「想像の共同体」としての安全・安心な社区づくりを掲げざるを得ない、ということである。 本研究は、JST 次世代研究者挑戦的研究プログラム JPMJSP2114 の支援を受けたものです。

報告番号173

災害と地域社会フリーライダー――熊本震災と武雄水害
九州大学 三隅 一人

1.目的 昨年の大会で、災害研究の文脈から、地域的な公共財の観点からコミュニティ概念を再考する意義を論じた。そこで提案したコミュニティモジュール複合の概念図式は、本来公共財の供給を阻む要因とされるフリーライダーを、社会関係資本ストックとして温存する視点をもっている。本報告は、そうしたフリーライダーの潜在的可能性を実証的に吟味することを目的とする。 2.方法 まず、コミュニティを、地域共有物を管理する社会システムとして定義する。地域共有物は、人びとの地域生活の安全や住みやすさにとって不可欠な非物質的な公共財・準公共財であり、地域の安全、レジリエンス、防災力、まちの活気、地域教育力、衛生、景観、自然環境等を含む。これらは多くの場合、利害関心が強い人たちが貢献し、利害関心が薄い人たちがフリーライドする形で供給されている。コミュニティモジュールは、そのコアな貢献者を中心とした部分社会をさす。そうするとコミュニティは、複数の異なるコミュニティモジュールが分業・分担するモジュール複合を構成する。 地域の人びとは必ずしもすべてのモジュールで貢献する必要はなく、どれかのモジュールで貢献し、他はフリーライドすることで、全体として地域生活に必要な地域共有物を供給し合えばよい。このようなコミュニティの可能態を、熊本県熊本市と佐賀県武雄市で実施した質問紙調査データを用いて吟味する。とくに災害にともなうコミュニティの顕在化、いいかえれば日常的フリーライダーの被災時の貢献に着目し、熊本市は2016年熊本地震、武雄市は2021年水害について、その動向を分析する。 3.結果 地域社会を包括的な公共財とみなし、地域関係の団体参加もイベント等への関与もない層を、地域社会フリーライダーとして操作的に捉える。どちらかがあれば、貢献層とする。 地域社会フリーライダーの地域関係以外の団体参加やイベント等関与は、熊本、武雄とも、「貢献層」との差はあるものの一定水準がある。このことから、地域社会フリーライダーが、地域運営以外の生活場面で何らかの地域共有物の供給に貢献している可能性が担保される。被災時の支援行為についても、貢献層との行動率の差は10%ほどにとどまっており、彼らが社会関係資本ストックとなりうる一定の根拠が示される。 被災時の支援行動は、災害時のコミュニティ顕在化の指標となる。熊本、武雄とも、被害がある(大きい)場合の方が被災時の援助行動を促す傾向あり、これはフリーライダーも変わらない。両地域ともに「災害ユートピア」の出現を確認できる。地域関係以外の団体参加やイベント等関与、いいかえればモジュール複合の多面性は、両地域とも、やや弱いながらも被災時の支援と結びつく。 4.結論 熊本震災と武雄水害のケースを通して、報告者がいうコミュニティモジュール複合が一定の現実形態をもって機能している可能性が確認された。さらに、こうしたコミュニティの将来的な顕在化の可能性をいくつかの貢献態度でみると、やはり微弱ではあるが、モジュール複合の多面性は貢献態度を強める効果をもつことがわかった。加えて、社会関係資本、とりわけ一般化された互酬性の規範が、両地区とも一貫して強い促進効果をもつことがわかった。

報告番号174

21世紀の職業変動とライフチャンス
大阪商業大学 佐野 和子

【目的】  本報告は、2000年代半ば以降の職業構造の変化が、仕事を通した人々のライフチャンスにどのような影響を与え得るのかについて、個人内、ならびに世代間の職業移動パターンに焦点をあてた分析により明らかにすることを目的とする。1990年代以降の欧米の労働経済学では、技術進歩、ならびに労働人材のスキル特性との関連の中で、職業構造の二極化説が検討されてきた。日本においては、アメリカほど顕著な二極化傾向はみられないものの、賃金分布の中間で大量の雇用を創出していた生産労務職と事務職のシェアが大きく減少し、賃金分布の<中の上>と<中の下>で医療福祉セクターの職業が拡大する傾向が確認されている(佐野 2021, 第94回日本社会学会大会報告原稿)。このような職業構造の変化は、労働市場を通した社会全体の平等性を表すマクロレベルの指標となるいっぽうで、個人にとっては、生涯のキャリアを通して獲得する職業上の地位、つまりライフチャンスに関連する問題となる。労働市場を構成する職業の性質や構成比が変化することにより、人々がより良い仕事に就くチャンスは高まっているのだろうか。またその状況は、出生年、性別、教育歴によってどのような差異があるのだろうか。 本報告は、技術進歩、高学歴化、労働市場制度の変化などと関連しながら生じた2000年代後半以降の日本の職業構造の変化の全体像を確認した上で、①世代の入れ替わりによる職業変動のプロセス、ならびに、②個人内の職業移動による職業変動のプロセスを分析することにより、近年の職業構造の変化が社会の構成員のライフチャンスにどのような変化をもたらしているのかについての知見を提示する。 【分析】 分析に用いるデータは統計法第33条に基づき提供を受けた「就業構造基本調査」2007/2017年の個別データである。また個人内移動についてはJGSSライフコース調査を援用する。 分析は3つの段階からなる。第1に、2000年代後半以降の日本の労働市場の変化の方向性を確認する。就業構造基本調査の職業を、賃金レベルに基づく5つの職業階層グループ(1-5Q)に分類し、各グループの2時点間の就業者の増減をみる。雇用形態や労働時間を問わず、すべての有業者を対象とすることで、非正規雇用者による職業変動への影響も検討する。 第2に、世代間移動に関する分析として、就業構造基本調査に含まれる5歳区分出生コホートの1-5Qへの分布を、2時点間で比較する。第3に、個人内の職業移動に関する分析として、就業構造基本調査に含まれる同じ出生コホートの2時点の1-5Q間の移動を、疑似的に同一個人の移動と見做して比較する。またJGSSライフコース調査のパネルデータを用いて、雇用形態や職のタイプに関する分析を加える。 【結果と結論】 世代間移動については、25-39歳では、男女共に、職業階層下位の1Qと、上位の4-5Qに就く傾向が高まり、若者の間で二極化傾向が進んでいる。個人内の移動については、男女の変化パターンは明確に異なる。男性の25-29歳では、リーマンショックの景気低迷期を経ても4-5Qへの上昇移動が維持されている一方で、女性はどの年代も1Qと4-5Qへの移動が拡大している。女性の内部でのキャリア分化の要因について、JGSSライフコース調査を用いた追加的な分析結果をもとに議論する。

報告番号175

非正規雇用から正規雇用への移動における階層間不平等――雇用関係とジェンダーに注目して
名古屋大学 鈴木 健一郎

【1.目的】 日本における不平等の規定要因について,欧米に比べて職種の重要性は相対的に低く,企業規模や雇用形態も挙げられる状況にある.このような多元性は,日本が「メンバーシップ型社会」(濱口 2021)であることから生じていると考えられる.一方,女性は「メンバーシップの周縁地帯」に位置づけられるとの指摘からは,女性にとっては職種が重要な変数になると考えられる.欧米的な階層研究の文脈で重視されてきた職種の効果が,日本に特徴的な雇用関係によって男女で異なる形で作用しているのではないだろうか.以上より,本研究は,正規雇用への移動について,職業特性の持つ効果が男女でどのように異なるのかを明らかにすることにある. 【2.方法】 「東大社研・若年パネル調査」「東大社研・壮年パネル調査」の合併データを用い,二次分析を行った.本研究では,Wave1からWave10までのデータを用い(追加サンプルは含めない),Wave1時に非正規雇用であったものを対象とした.また,転職による正規転換か,内部登用による正規転換かをそれぞれ区別するために,離散時間競合リスクモデルを用いて分析を行った.したがって,目的変数は転換していないときに「0」,転職による正規転換のときに「1」,内部登用による正規転換のときに「2」を取るカテゴリカル変数である.また,主な説明変数は,ゴールドソープの階級区分をベースにした職業階層の変数と,前年と同じ職種に就いているかどうかを示す職種の一貫性ダミーである. 【3.結果】 分析の結果,職業階層変数について,男性は,転職・内部登用ともに職業階層変数は有意にならなかった.女性は,監視が難しく特殊資本である場合,転職による正規転換が,監視が易しく特殊資本である場合,内部登用による正規転換が起こりやすいことがわかった.また,職業の一貫性について,男性の場合,職種が一貫していることが内部登用による転換に有意な効果を持たなかったのに対し,女性の場合,職種が一貫していると,内部登用による転換が起こりやすいことがわかった.以上より,女性にとっては,職業階層が正規転換について不平等を生み出す変数であり,また, 非正規雇用時に蓄積した職種特殊的な人的資本を生かして正規転換している傾向にあると言える. 【4.結論】 以上から,男性はメンバーシップ的な正規雇用へ移動し,女性はジョブ的な正規雇用に移動する,という類型を提示することができる.また,このことは,労働市場の中心部に参入し,メンバーシップに包摂されるにあたって,特定の職務をこなす職務遂行能力ではなく,勤めている企業の要求にフレキシブルに応える「能力」(濱口 2021)が重要であることを示唆するものである. 【付記】 二次分析に当たり,東京大学社会科学研究所附属社会調査・データアーカイブ研究センターSSJデータアーカイブから個票データの提供を受けました. 【文献】 濱口桂一郎,2021,『ジョブ型雇用社会とは何か : 正社員体制の矛盾と転機』岩波書店.

報告番号176

高度成長期における産業衰退と若年労働力の移出――北海道内炭鉱の企業・学校・家族と中学生の進路
早稲田大学総合人文科学研究センター 笠原 良太

1.目的  本報告の目的は、高度成長期の衰退産業における若年労働力の移出について、企業・学校・家族に着目して明らかにすることである。これまで、高度成長期における衰退産業から成長産業への若年労働力の移動は、企業・学校・職安の制度にもとづく農村余剰人口(農家の二・三男、新規中卒者)の都市部への移動として説明されてきた(山口(2016)、苅谷ほか編(2000)など)。しかし、産業・地域特性ならびに家族・世代に着目した研究が不足しており、若年労働者がいかなる資源を活用して進路を決定し、その進路が家族にどのような影響をもたらしたのかについて、十分明らかにされていない。本報告では、高度成長期に急速に衰退した石炭産業を例に、これらの点を明らかにする。炭鉱の衰退と閉山は地域の崩壊を意味したため、企業、学校、家族が若年労働力の移出を喫緊の課題として進めたと考えられる。 2.対象・方法  本報告の対象は、北海道旧音別町に位置した雄別炭砿社㈱尺別炭砿である。同炭鉱は1910年に開鉱し、戦中の休坑、戦後復興を経て、1960年代に衰退し、1970年2月に閉山した。炭鉱街の人口は4000人程度であり、大半が炭鉱関係者であった(嶋﨑ほか(2020)参照)。中学校は1校のみであり、同校同窓生の進路から当時の若年労働力の移出を把握できる。本報告では、1960年代の同校卒業生(転校生を含む、1944-53年出生コーホート)の中卒後進路を把握したうえで、彼らの定位家族(親・きょうだい)の移動について把握する。  本報告で使用するデータは、同窓会ならびに同郷会の会員を対象に実施した「尺別炭砿で暮らした人びと調査」(産炭地研究会(JAFCOF)との共同研究、2016~18年度実施、質問紙調査ならびにインタビュー調査)のデータおよび労働組合解散記念誌名簿、近隣高校の同窓会名簿などである。 3.結果  まず、1960年代の同校卒業生は、男女ともに隣町(白糠町)もしくは近隣の拠点都市(釧路市・帯広市)の全日制高校(女子は各種学校を含む)に進学し、卒業後、男子は道外または札幌、女子は釧路に就職する傾向にあった。彼らは早くから炭鉱の先行きを不安に思い、石炭産業の衰退を認識していた。そして、彼らは企業の就職斡旋や奨学制度、父母・近隣からの進学・就職に関する期待と支援、さらには教員による炭鉱の外を志向させる教育などをもとに、炭鉱を離れていった(離山)。  彼らの離山は、その後、炭鉱に残った父母・弟妹が移動する際の資源となった。炭鉱に残った家族は、1960年代後半にかけて、先行して離山した子どもを頼りに離職・離山していったのである(連鎖移住)。なかには、親子で同じ企業に就職するケースもみられた。 4.結論  以上のように、産業衰退期における若年労働力の移出は、企業・学校による各種制度に加えて、家族や近隣の期待や支援などに水路づけられて成立した。そして、成長産業都市に移住した彼らは、のちに親やきょうだいが産業転換を達成するための水先案内人となったのである。 参考文献 苅谷剛彦・菅山真次・石田浩編,2000,『学校・職安と労働市場』東京大学出版会. 嶋﨑尚子・新藤慶・木村至聖・笠原良太・畑山直子,2020,『〈つながり〉の戦後史――尺別炭砿閉山とその後のドキュメント』青弓社. 山口覚,2016,『集団就職とは何であったか――〈金の卵〉の時空間』ミネルヴァ書房.

報告番号177

コロナ禍前後におけるテレワーク利用機会の所得階層間格差の拡大とその要因
学習院大学 麦山 亮太

[背景] 労働者が可能な限りテレワークの実施を選択できることは、労働者自身が自律的に働けるようにするうえで重要な政策課題である。今般の新型コロナウイルス感染症の流行(コロナ禍)によって、国内外において、それ以前より推進されてきたテレワークへの関心は急速に高まった。コロナ禍における日本の特徴は、強制的にロックダウンが行われるのではなく、テレワークの実施および制度化が義務ではなく要請ベースであったことにある。こうしたなかで、テレワーク制度の利用が高所得層に限られるという格差が問題視されている。しかしながら、コロナ禍以前に比してどの程度所得階層間格差が拡大したのか、またそれがいかなる要因によって生じているのかは十分に明らかになっていない。 [目的] 本報告では以下の3つの問いを検討する。第一に、コロナ禍前後で所得階層間でのテレワーク制度適用、ならびにその利用における格差は拡大したのか。第二に、所得階層間格差の拡大はその従事する業務の違いによってどの程度説明できるのか。第三に、同程度テレワークに適した業務に従事していてもなお残る所得階層間格差の拡大は、勤める企業の組織的特性によってどの程度説明できるのか。以上の検討を通じて、コロナ禍において、高所得層が勤める企業においていち早くテレワーク制度の導入・利用が拡大したことが所得階層間格差の拡大に寄与したことを論じる。 [方法] 分析にはリクルートワークス研究所が実施する「全国就業実態パネル調査」の2020年、2021年データ(それぞれ2019年12月、2020年12月の状況を聴取)をパネルデータとして使用する。分析対象は2時点とも20–64歳かつ週30時間以上働いている被雇用者とする。従属変数は、職場にテレワーク制度があり、かつ回答者本人にもその制度が適用されているか否かを示す二値変数である。所得階層は2020年調査で聴取された昨年一年間の勤労所得をもとに5分位を作成する。職業レベルの業務特性は日本版O-NET(労働政策研究・研修機構)の数値情報より作成した指標、個人レベルの業務特性は個人が行っている業務の性質に関する指標、組織特性は2019年12月時点において回答者の職場で成果にもとづく雇用管理(成果重視の人事評価、目標管理制度、KPIによる成果管理)がどの程度なされているかを示す指標をそれぞれ用いる。分析には差分の差法(Difference-in-differences)を用いる。 [結果] コロナ禍前後でテレワーク制度の適用率ならびに実施率は大幅に上昇したが、高所得層においてその増加はより大きく、結果、所得階層間の格差は拡大した。この格差の拡大のうち約3分の1は、高所得層が主に従事するテレワークに適した業務においてよりテレワークの適用・利用が広がったことによって説明できることが示された。さらに、非転職者を用いた分析より、なお残る所得階層間格差の拡大の一部は、2019年12月時点で成果にもとづく雇用管理がより強くなされている企業がよりテレワーク制度の適用・実施に至りやすかったことによって説明できることが明らかとなった。

報告番号178

年齢による職域分離――趨勢と内実の検討
東京大学大学院 新田 真悟

【1.目的】任意の属性に基づく、従事する職業の分布の違いを指して、職域分離と呼ぶ。職域分離は賃金格差を生じさせる要因であり、日本を含め研究が蓄積されている。先行研究の多くは性別による職域分離の実態を記述しており、他の社会的カテゴリに基づく職域分離については検討の余地が残っている。本報告の目的はとくに年齢による職域分離に着目し、その水準を検討することにある。近年のインターセクショナリティをめぐる議論では、ジェンダーや人種、年齢といった社会的カテゴリは必ずしも相互排他的ではなく、むしろ複雑に絡み合いながら格差・不平等に作用することが論じられている。性別にとどまらない社会的カテゴリに基づく職域分離の実態を詳らかにすることは、翻って性別職域分離の関する先行研究にも寄与することが期待される。【2.分析】分析に用いるデータは1997-2017年の就業構造基本調査の匿名データである。分析対象は15-84歳までの有業男女である。分析にあたっては年齢を15-64歳(現役層)と65-84歳(高齢層)のカテゴリに分類し、それぞれがどの程度分離しているか、その分離はどのように変化してきたのかをダンカンの分離指数を用いて算出した。加えて、分離の内実を明らかにするため、各職業における年齢分布から、いかなる職業でいかなる分離がされているのかを探索的に検討した。【3.結果】1997年から2002年にかけて年齢による職域分離は拡大していたものの、2002年以降は一貫して縮小傾向にあった。さらに、その分離がいかなるものかを探索的に検討した結果、以下の事柄が明らかになった。はじめに、高齢層に分布が偏っている職業として、「居住施設・ビル等管理人」「家庭生活支援サービス職業従事者」「清掃従事者」「農業従事者」などがあげられた。以上の職業はいずれも従事している者の平均所得がおよそ200万円以下の職業であり、それゆえに高齢層は周縁的な職業に配置されていることが明らかになった。【4.考察】2006年以降は改正高年齢者雇用安定法が施行され、60-64歳の高年齢層ではあるものの、高齢者がますます労働市場へ参入するようになってきた。2007年以降の年齢職域分離の縮小は、こうした結果を部分的に指示している。しかし、内実を確認すると、高齢者が多く従事している職業は個人所得の水準が低く、なお高齢者は周縁的な職業に従事しているといえる。以上の結果は年齢によっても職域分離は存在しており、さらにその分離は高齢者が周縁的な職業に従事していることかたちで現れていることが示唆された。

報告番号179

大規模災害からの復興の地域的最適解に関する総合的研究 2022(1)――復興の「地域的最適解」をめぐる量的-質的研究の到達点
早稲田大学 野坂 真

本報告は、科学研究費(基盤研究A)「大規模災害から復興の地域的最適解に関する総合的研究」(研究代表者 浦野正樹)の中間報告として、本研究プロジェクトの中核的概念である「地域的最適解」をめぐる議論の成果を報告し、現段階における研究プロジェクトの到達点と課題を検討することを目的としている。  本研究プロジェクト全体の目的は、災害復興には地域的最適解があるという仮説命題を実証的な調査研究によって検証し、得られた知見に基づいて、次に予想される大規模災害からの復興の制度設計に関して政策提言を行い、研究成果の社会への還元をグローバルな発信を重視して積極的に行うことである。2020年度から2021年度までに、①ミクロ/マクロの重層的な地域レベル(広域都市圏-自治体-地区)と、短期的時間軸/長期的時間軸を設定した「地域的最適解」という視点の検討、②調査対象地として選定した複数地域における復興過程・復興状況の「レジリエンス」「サステイナビリティ」「インクルージョン」「エンパワーメント」「ウェルビーイング」という復興の地域的最適解を評価するための5つの視点からの検証を行い、その結果を日本社会学会大会にて報告してきた。  2021年度での報告以降、次に挙げる4つの調査・研究を行ってきた。 1)東日本大震災における津波災害に関する調査対象地として選定した計8の被災市町における住民を対象に、主観的な復興達成度などを質問紙調査(サンプリングによる調査対象者数8700人・回収数3993票・回収率45.9%)によって尋ねることで、自治体単位での復興の現状と傾向の相違の把握を試みてきた。これは、よりマクロな地域レベル・短期的な時間軸を中心に復興の地域的最適解という視点の検討を図るものである。 2)自治体よりミクロな単位の地域における、発災前までの地域振興史から復興後の地域像までをも含む被災・復興プロセスを、生活問題を集合的に認知し対応し続けるプロセスとして分析するため、各調査対象地における地域リーダー層へのインタビュー調査の実施を計画してきた。これは、よりミクロな地域レベル・長期的な時間軸を中心に復興の地域的最適解という視点の検討を図ると同時に、復興の地域的最適解を評価するための5つの視点のさらなる具体化を図るものである。1)の調査から観察される復興の現状と傾向の相違の背景要因を地域ごとの復興の最適解の相違として検討するねらいもある。 3)本プロジェクトでの質問紙調査と全国調査(JGSS)の設問を一部重ねて、結果を比較分析することも試みてきた。これは、災害研究関連のデータベース構築に向けた取り組みの一環でもある。 4)1)~3)の結果とともに、復興概念の問い直しや、被災・復興プロセスが被災した地域社会において集合的に認知され規範化されていくプロセスに関する理論的な災害研究の成果もふまえつつ、復興の地域的最適解の理論的フレームワークの精緻化を行ってきた。  今回は、「大規模災害からの復興の地域的最適解に関する総合的研究 2022」と題した一連の報告において、1)~4)の調査・研究による現時点までの成果を報告する。なお、前段落までに挙げた1)~4)の調査はいずれも本研究プロジェクトのメンバーが各々行っている研究プロジェクトと協働して実施したものである。

報告番号180

大規模災害からの復興の地域的最適解に関する総合的研究 2022(2)――「復興条件不利地域」における生活再建と社会変動
名古屋大学 室井 研二

1.目的 現在の東北被災地は震災や復興政策の影響と平時の趨勢的変動の影響が入り混じる状況の中に置かれており、そのような形で復興格差が生成、拡大しつつある。本報告では、震災後の人口流出が特に著しい宮城県南三陸町、女川町、山元町を「震災復興条件不利地域」として位置づけ、震災後の生活構造やコミュニティの生活環境条件について比較地域論的な観点から分析する。 2.方法 地域間比較の論点として重視するのが、①自治体の産業構造や広域経済圏における結節機能、②自治体の復興政策、③住民の生活構造である。本報告ではその中でも特に③に重点を置く。報告者は2021年10-11月に上記3自治体の津波浸水地の住民を対象にサーベイ調査を実施した(自記式、郵送法。有効回収率45.9%)。この調査結果から住民の生活構造やコミュニティの生活環境条件を量的に把握するとともに、質的調査による現場検証を踏まえて復興格差の地域差について検討を行った。 3.結果  まだ予察的な段階であるが、震災後の変化として以下のような知見が得られた。 自治体の違いに関係なく、全体として世帯の縮小が顕著である。住民のおよそ半数が居住地を移転し、コミュニティの分解が進んだ。地区の生活環境条件は全体として悪化しており、特に公共交通の便や雇用・就業条件に関してその傾向が顕著である。地区のまとまりや自治活動は震災前よりも停滞した状況にある。 自治体間の差異については以下のようなことが指摘できる。南三陸では土着的な地域的つながりが比較的維持されているが、物的な生活環境条件(特に公共交通の便)の現状評価が著しく低い。女川では世帯の縮小や地域活動の停滞が顕著であるが、生活環境条件の現状評価はそれほど低くはない。山元では生活環境条件が震災後むしろ改善された面があるが、住民の生活構造は流動的であり、地域的な社会的凝集性は低い。 以上のような復興の地域差には、自治体の復興政策(特に、コンパクトシティ政策)や産業構造(特に、原子力産業の有無や広域都市圏への包摂度)、地域の歴史的沿革の差異が密接に関係していると考えられる。また、自治体内部の生活環境条件の差異について分析したところ、個人の属性変数よりも地区特性(空間的変数)が一貫した規定力を有していた。このことは、復興格差の地域差を理解するためには自治体間比較だけでなく、自治体内部の地域差に着目する必要性を示唆するものである。これらのことを現地調査の結果を踏まえて議論し、災害復興に関する持続可能性の条件や課題について地域類型論的な検討を試みる。

報告番号181

大規模災害からの復興の地域的最適解に関する総合的研究 2022(3)――地域リーダー調査で見出されてくる復興ヘゲモニー更改の諸事例
専修大学 大矢根 淳

1. 目的  「復興」は既定復興※1の公告(公共土木事業の竣工)によって結審するものではないと捉え、被災者らが奮闘努力する生活再建の実相を調査していくと、そこに種々のステークホルダーが把握されてくる。科研(JP19H00613)グループ2021-22年度企画の一つである地域リーダー調査のデータから、そうした被災したローカルな現場の取り組みをピックアップして、そこにおいて、復興を巡って新たな型のヘゲモニーが立ち現れつつあることを報告したい。 2. 方法  これまで継続して実施して来たインタビュー※2で把握した事柄に基づき報告する。インタビューは、被災・復旧・復興の位相の転化に即して現出するローカルの諸課題を都度認知して、そこに対峙してきた諸グループの代表者に対して、2-3時間の半構造化インタビューを実施し、音声データを活字化した。対象地は宮城県石巻市の津波被災地である。  インタビュー項目は、自身の被災当日の対処行動から活動始動の契機、そうした活動に取り組むことになる自身の道筋(生い立ちや諸社会関係、業務経験等)、活動内容の展開と周辺機関との連携、被災10年を振り返っての成果・評価と課題(復興10年総括検証)、そしてコロナ禍の活動状況等。 3. 結果  「石巻じちれん」は仮設住宅で発足した石巻仮設住宅自治連合推進会が、復興公営住宅団地にリエゾンして展開している活動である。これを当初から側面支援している「日本カーシェアリング協会」が被災者の移動、「足」機能を支援(ヒト本能としての移動を補助)するとして参画し、市立病院開成仮診療所の医師グループがこの移動支援の医学的倫理的意義を特認・援用してこれに帯同して地域医療体制を展開した。そして、「石巻復興きずな新聞」が現場の声・実情を地道に取材して記録し、手渡し配達にこだわることで結果的に見守り機能を担保し続けている。そして「じちれん」活動の場には、子どもを持つ若い女性らのグループ(NPOべびースマイル石巻)や子どもの遊び支援のグループ(NPOにじいろクレヨン)がジョイントしていて、その場に世代重層的・ジェンダー視点の志向性を付与している。 4. 結論  「支援は場所にではなく(課題を抱えている)人に対して行うべき」と言われる。仮設住宅が解消されれば、表記上、仮設支援はその対象を失うこととなるが、被災者は生活再建上の課題を抱えたまま異動して奮闘努力を重ねている。そこには、災害サイクルの位相の転化に即して諸組織編隊を緩やかに組み替えながら、活動を継続する支援者がいる。  復興10年総括検証などでそうした取り組みが言説化されて、復興の重要なステークホルダーとして改めて認識・オーソライズされる回路が拓かれ、復興ヘゲモニーの更改が胎動している。  調査を継続・蓄積して、復興の実像と枠組み・その概念検討を深めたい。 ※1 復興を考える際、「復興都市計画事業」等、復興の名を冠した公共事業の竣工を「復興」と同定する視角を批判的に論ずる述語。 ※2 専修大学社会科学研究所・特別研究助成「復興ステークホルダーの探索的再構築」(2017)、「減災サイクルのステークホルダーと事前復興への取り組みの実相」(2018-21)、「復興アダプティブ・ガバナンスの理論的実証的探索」(2022)、の一連のグループ研究。成果報告書は例えば、以下を参照いただきたい。 http://www.senshu-u.ac.jp/~off1009/PDF/geppo2020/684/684-shozawaoyane.pdf

報告番号182

大規模災害からの復興の地域的最適解に関する総合的研究 2022(4)――JGSS-2021/2022による被災経験と避難への準備
大阪商業大学 岩井 紀子

1. 目的 気象庁が顕著な災害を起こした自然現象として命名した地震や豪雨の件数は近年増えており、被災を経験した人も増えていると考えられる。本報告では、2021年2月と2022年2月に全国(440地点;400地点)で実施した「日本版総合的社会調査JGSS-2021と2022」に組み込んだ災害関連項目を基に、①人々の被災経験と被災時期、②今後避難を必要とするような大規模災害に襲われるリスク認知、③避難準備と避難先、④復興政策を見直すことについての人々の意識を分析する。②③④については、「東日本大震災の復興に関する意識調査」の石巻市の地震・津波被災地の結果と比較する。 2. 方法 JGSS-2021の災害に関する質問項目から得られたデータを活用し、全国8地域ブロックごとの傾向をとらえる。分析に用いる具体的な項目は、<被災経験>と<被災時期>、<大規模災害に対するリスク認知>、<避難準備>と<避難先>、<復興政策を見直すことについて意識(復興観)>である。被災経験と被災時期によって、災害リスク認知や避難、復興政策への意識がどのように異なるのか、またその状況は地域によってどのような特徴を示すのかを明らかにする。 次に、<大規模災害に対するリスク認知>、<避難準備>、<復興観>は、JGSS-2021と同じ質問を含む「東日本大震災の復興に関する意識調査」の石巻市のデータから得られた結果と比較検討する。 さらに、避難勧告・避難状況については、NHKが2019年11月に実施した「災害に関する意識調査」、復興政策についてはNHKが2020年11月に実施した「復興に関する意識調査」を参照し、人々の意識や今後必要とされる政策についても考察する。 3. 結果 分析から得られた主な知見は次の通りである。大規模自然災害の経験は、1990年代(1991雲仙、1993北海道南西沖/豪雨、1995阪神淡路)に多く、2010年代には1990年代の数倍になった。大規模自然災害は身近になったといえる。災害発生のリスク認知は、全国では地震が、東北では地震は全域、津波は宮城で高い。石巻では、地震、原発、津波、水害については全国よりも高い。被災経験と災害リスク認知との関係をみると、全国では、地震、水害、豪雪、土砂災害、原発については、災害経験がある場合に、ない場合よりも、リスク認知が高い。石巻では、被災経験の有無と関係なく全般的に高い。避難準備については、非常持ち出し準備が4割で、避難場所確認37%、特に何もしていないが1/3。石巻では、何もしていないは13%で、準備は全国に比べて高い。 【謝辞】日本版 General Social Surveys(JGSS)は、大阪商業大学 JGSS 研究センター(文部科学大臣認定日本版総合的社会調査共同研究拠点)が、大阪商業大学の支援を得て実施している研究プロジェクトである。JGSS-2021/2022 は、京都大学大学院教育学研究科教育社会学講座の協力を得て実施し、文部科学省特色ある共同研究拠点の整備の推進事業 JPMXP0620335833、JSPS科研費 JP20H00089 の助成を受けて実施した。データの整備は、JSPS人文学・社会科学データインフラストラクチャー構築推進事業 JPJS00218077184の支援を得た。「東日本大震災の復興に関する意識調査」は、JSPS科研費JP19H00613(研究代表:浦野正樹)の助成を受け、データ整備の一部はJSPS人文学・社会科学データインフラストラクチャー構築推進事業 JPJS00218077184の支援を得た。

報告番号183

大規模災害からの復興の地域的最適解に関する総合的研究 2022(5)――災害の記憶: 災害遺構をめぐって
尚絅学院大学 田中 重好

「災害の記憶」は、認知的である(「どういった災害だったか」)と同時に、規範的(「災害に対して何をすべきか」)であり、「災害の記憶」に基づいて、人びとは防災行動を行うだけではなく、行政を含めた社会は防災対策を制度化し、防災対策事業を実施することにつながってゆく。こうした点を確認するならば、「災害の記憶」の問題は、「復興の最適解」のなかで重要なテーマである。  「災害の記憶」はさまざまな形でシンボライズされるが、ここでは、災害遺構を手がかりに、東日本大震災の「災害の記憶」がいかに形成されたのかを検討する。 災害遺構は社会的なプロセスをへて、「『被災構造物等』が『震災遺構候補』を経て『震災遺構』となる」(西坂、2019:25)。すなわち、災害遺構として保存を決定する過程は、社会が「災害の記憶」を構築する過程にほかならない。  現在、東日本大震災の被災地の災害遺構として保存されている建物に注目して「災害の記憶」がどう作られているかを見ると、建物の所有、保存のための財源、復興事業との関連、地域の合意などの条件を考慮しながら、市町村が中心となって、複数の社会的主体からの意見を調整する社会過程を通して、災害遺構保存がなされている。その結果、現在、災害遺構となっている建物はすべて、「津波から安全に避難できた」「犠牲者が出ていない」建物である。  「土地に刻まれた記憶」について見ると、そこで亡くなった人々の「記憶」は慰霊碑だけで、「なぜ亡くなったのか」を物語る記憶とはなっていない。高い津波に襲われた地域は危険区域に指定され、以前から住んでいた住民のなかの生存者が移転してしまったために、「死者を記憶する共同体」(村上興匡、2013:14)としての被災地コミュニティが消滅してしまった。土地に刻む主体であったコミュニティがなくなったことにより、「親しい人を失った」共通体験が成立しないために、犠牲者の記憶は個々人の、私的世界のなかでしか受けとめられなくなり、土地に災害体験を刻むことは少なくなった。  災害遺構として建物、土地双方において、「なぜ、この建物、土地で多くの人々が亡くなったのか」というメッセージを残していない。その結果、災害が自分に降りかかったとき、「何をすべきなのか」「何をなさざるべきか」という教訓が東日本大震災の「災害の記憶」には欠落している。  こうした全般的な傾向のなかで唯一例外的存在は、多くの犠牲者が出たが、地元の努力によって災害遺構として保存されている大川小学校である。大川小学校では、犠牲者が「私的な世界の存在だけではなく」パブリックな存在として、さらに、危険区域となり地元住民が移転してしまった後にも「死者を慰霊する共同体」が持続してきた。  現在、東日本大震災の「災害の記憶」が「犠牲者を出さず安全に避難できた」災害遺構の形で定着するのか、「災害の犠牲者をパブリックな存在として受けとめ、死者を慰霊する共同体」の延長上につくられていくのかの分岐点にある。

報告番号184

Withコロナ時代の日米価値観調査の分析(1)――調査概要と回答分布の比較
金沢大学 轟 亮

【1.目的】  「日米若年世代の価値意識の変動を解明する計量社会学研究」というテーマで4年間の共同研究を行っている(第2年度)。2022年1月に、日米の成人を対象として登録モニターを用いたインターネット調査を実施した。本報告では、調査概要と回答分布の単純な比較の結果を示す。 【2.方法】  「Withコロナ時代の生活と価値観に関する調査」(General Survey on Life and Values During the Period of Coronavirus Pandemic)は、2021年末時点で満18~69歳個人(1952~2003年生)が対象である。日米で、同一の内容の日本語と英語の調査票をそれぞれ使用した。回収目標数3000人とし、住民基本台帳/American Community Surveyをもとに、地域、性別、年代による標本割付を行い、それを回収目標数にして登録モニターから回答を得た。調査期間は2022年1月12、13日から約10日間。有効回収総数は日本3236、アメリカ3439であった。主な質問項目は、パンデミックに関する意識、対策行動、ジェンダー意識、格差観・階層意識、排外主義、社会的成功の要因観、環境意識、将来イメージ、自己イメージ、仕事に関する意識等である。 【3.結果】  意識の質問項目では、無回答を許容し、既存調査との比較の問題がない場合にはDK選択肢を設定した。約60項目について、国変数(日米の2値)とのクロス表によって回答分布の違いを検討した。  調整標準残差の検討から、無回答率に日米差はない。DK選択肢(わからない、Don’t know)の選択率には、多くの項目で有意差がみられた。ただし、これは中間選択肢(どちらともいえない Neither agree nor disagree)を設定しているかどうかという点や、質問の意味内容と関係しているおり、一般的に、日本でDK選択肢が選ばれる傾向が高いと判断することは難しい。中間選択肢の選択率はDK選択率よりも高く、多くの項目で日本が中間選択肢を選ぶという有意差がみられる。日米で回答を比較する際には、中間選択肢を選ぶ傾向の違いに留意することが必要である。 【4.結論と課題】  2018年実施の日米若年層調査で、ピアソンの積率相関係数を指標とすると(米国ダミー変数との相関。回答の平均の差を検討することを意味している)、生活満足度が最も大きな日米差を示した項目のひとつである(米がより肯定、US+と表記、以下同じ)。今回も、生活満足度の相関係数の絶対値は0.311と高い。これを上回る項目は、高いものから順に、(1)10年後の自分の生活(0.376、US+)、(2)将来よりもいまの人生を楽しむ(US+)、(3)コロナ感染は周囲の反応が心配(US-)、(4)外国人が隣に引っ越してきたら気になる(US-)、社会的成功に必要なことについて、(5)宗教的な心(US+)、(6)正直である(US+)、(7)権力をもつ人に従う(US+)であった。また0.3~0.2の相関を示した項目群に、政府は格差を縮小よりも国全体の経済成長を優先すべき(US+)が含まれた。ポジティブ感情項目に比べて、政治意識やジェンダー意識、環境意識の差異は大きくない。今後、年齢や性などの統制をして比較を進める予定である。

報告番号185

Withコロナ時代の日米価値観調査の分析(2)――将来の見通しは現在の幸福に影響を与えるか
群馬県立女子大学 歸山 亜紀

1.目的  新型コロナ感染症流行下で、人びとの生活満足度や幸福度は低下した。内閣府「国民生活に関する世論調査」によれば、生活満足度(「あなたは、全体として、現在の生活にどの程度満足していますか:4件法」)は、令和元年度(2019年6月)では全体で73.9%が満足(「満足している」または「まあ満足している」)と回答していたが、令和3年度(2021年9月)の同調査では、満足は全体で55.3%であった(ただし、「国民生活に関する世論調査」は令和元年度まで個別面接法が用いられていたが、令和3年度は郵送法であったため、データ収集法の違いも含まれている可能性があることには注意が必要である)。とくに若者(18~29歳)においては、満足は23.2ポイント減となっている。日本では、「若者の高幸福や高生活満足」がたびたび指摘されてきたが、コロナ禍は若者の生活満足度に大きく影響した可能性がある。ただし現在進行形の社会状況(たとえば外出や会食の制限)が現在の生活満足度に影響を与えるのはいわば当たり前のことともいえ、その状況が解消されれば満足度は再度高まるかもしれない。一方、「いま」ではなく「将来はこうなっている」という将来の状況判断は現在の生活満足度や幸福度の判断に影響するのだろうか。また、そうした関連は日本と米国で異なるのだろうか。 2.データ  本報告で用いるのは、2022年1月に日本と米国でおこなった非確率オンラインパネル調査データである。調査対象は18~69歳までの人々で、各国3,000ケースを年齢、性別、居住地(都道府県/州)の人口構成比に合わせて割り当て、回収目標数とした。有効回収数は日本3,236、米国3,439であった。 3.結果  生活満足度(あなたは生活全般に満足していますか、それとも不満ですか。/How satisfied are you with your life?:5件法)は、日本で中央値3.00、平均値2.99、米国で中央値4.00、平均値3.78で全体では日本のほうが低い。主観的幸福(現在、あなたはどの程度幸せですか。「とても幸せ」を10点、「とても不幸」を0点とすると、何点くらいになると思いますか。/How happy would you say you are?)は、日本で中央値6.00、平均値5.60、米国で中央値7.00、平均値6.50であり、こちらも日本のほうが低かった。将来の状況判断は「10年後の日本は今より良くなっている。/Ten years later, America will be better.:5件法」「10年後の私の生活は今より良くなっている。/Ten years later, my life will be better.:5件法」で尋ねた。  「10年後の日本(米国)の見通し」と生活満足度の相関係数は、日本r=0.230**、米国r=0.343**で、主観的幸福との相関係数は、日本r=0.166**、米国r=0.286**であり、いずれも米国のほうが関連が大きい。「10年後の自分の生活の見通し」と生活満足度の相関係数は、日本r=0.318**、米国r=0.310**、主観的幸福との相関係数は、日本r=0.257**、米国r=0.293**であり、こちらは日本のほうが関連が大きい。日本と米国では、将来の社会や自分の状況判断と、現在の幸福度との関連の大きさが異なることがわかった。日本の若年層(18~29歳)のみの分析では、「10年後の日本(米国)の見通し」と幸福度の相関係数は、日本r=0.079n.s.、米国r=0.294**で日本の若者が日本社会の将来判断と自分の幸福度を関連付けていないことがわかった。当日は、ほかの年齢層の結果も含めて報告する。

報告番号186

Withコロナ時代の日米価値観調査の分析(3)――人々は社会の為に個人の自由や権利を犠牲にするか
お茶の水女子大学 杉野 勇

【1.目的】  Covid-19によって,世界各地でいわば社会防衛と個人の自由・権利の深刻なトレイド・オフが発生し,価値観の違いや対立が先鋭化したと思われる。日本におけるいわゆるマスク警察や自粛警察と云った現象は今でも完全に過去のものとなったとは言い難い。本報告の目的は,ウィルスの拡散防止という共同善もしくは集団的利益の為に,個人の自由もしくは権利が引き換えにされることを人々がどの程度甘受するつもりがあるのかと云う,集団主義と個人主義の葛藤について,人々がどの様な態度を表明するのかを明らかにする事である。その際,どの様な言語表現の違い(wording)によってどの程度人々の回答傾向が変化するかをサーヴェイ実験を用いて明らかにし,更に日本の非確率オンラインパネルと合衆国のオンラインパネルで類似のサーヴェイ実験を行う事によって日米の違いを明らかにする。 【2.方法】  少しずつ異なるサーヴェイ実験を実施した複数の調査データを分析対象とする。2020年9月の非確率オンラインパネル調査(日本,一都3県+愛知),2021年1月の無作為標本調査(日本,一都3県+愛知),2021年10月の非確率オンラインパネル追跡調査(日本,一都3県+愛知),そして2022年1月の日米の非確率オンラインパネル調査(日本全国とアメリカ合衆国)である。  実施したサーヴェイ実験の基本形式は,「ウィルスの拡散防止に役立つならば,〔無作為提示〕かまわない」に対する5件法回答であり,〔無作為提示〕の部分に「自分の権利をある程度犠牲にしても/市民の自由をある程度制約しても」を使用したもの,「自分の自由がある程度制約されても/市民の自由がある程度制約されても」を使用したもの,「市民の権利を制約しても/自分の権利が制約されても」を使用したものがある。 【3.結果】  非確率パネルでも無作為標本でも,「市民の自由を制約」よりも「自分の権利を犠牲」の方が同意割合は明らかに少なかった。しかし「自分の自由を制約」と「市民の自由を制約」の間では,また「市民の権利を制約」と「自分の権利が制約」の間でも,差があるとは言えなかった。合衆国においても,”to have the rights of citizens restricted”と”to have my rights restricted”の間で差は無かった。日米比較では,合衆国の方が日本よりも権利の犠牲を厭わない回答が多いが,それに反対する割合も日本よりも多かった。 【4.結論】  自分自身の問題として尋ねられた場合とそうでない場合で回答に違いがあるかの様に見えたが,実はそうではなく「自由/権利」や「犠牲にする/制約する」と云う表現の違いによって回答傾向の違いが生じていると考えらる。日米比較では,合衆国の方が集団優先にも見えるが,むしろ両極に明確に回答が分かれたと言える。それ以外には,日本の60代は制約を甘受しがちであるが合衆国では「市民の権利」のヴァージョンにおいてのみ20代が制約に相対的に強い抵抗を示した。また,排外主義は両国ともに関連が無かったが,合衆国では(リベラルに対して)保守であるほど制約に明確に抵抗を示すと云う相違が見られた。

報告番号187

コロナ禍のイベント「自粛」は変わったのか――新型コロナ禍におけるイベントスペース利用者調査2022年から(1)
一橋大学院 栗原 真史

1. 目的  劇場、ホール、ライブハウスなどの都市の集合行為を支えてきた「イベントスペース」は、新型コロナ禍において危機に直面した。リアルな場所でのイベントが制限されるなか、この間、感染対策としての「自粛」やリモート参加への切り替えをはじめとして、イベント主催者や場所の運営者、業界団体による様々な対応・摸索がなされ、危機とレジリエンスの併行する状況が明らかとなった(石毛編2021; 町村ほか2021)。  だが他方で「ショック」として経験されたコロナ禍1年目から長期化を迎えたコロナ2年目において、感染拡大とイベントを媒介する位置づけをもつ個々のイベント参加者の行動がいかにして再編され変容してきたのか、この点は明らかではない。個々がずれをもちつつも全体の同調が志向されるコロナ対応のなかで「自粛」と「再開」の間を誰がどのように往還し、事態は新たな段階を迎えているのか。コロナ禍でのイベント「自粛」をどのような要因が規定し、コロナ1年目と2年目でいかなる違いが見られるのか。 2. 方法  報告者たちは、2022年2月が実施したWebモニター調査(「新型コロナ禍におけるイベントスペース利用者調査2022年」)に基づき、新型コロナ流行(2020年3月)以降の首都圏での各種イベント参加者の分析を行った。調査は、2019年から調査時点までに、劇場・小劇場、コンサートホール・音楽ホール、クラブ・ライブハウス、ボランティア・講演会・シンポジウム、同人誌即売会・ACGイベントへの参加経験のある者を対象とし、新型コロナ・感染防止対策・イベント参加のあり方に関する意識と行動を尋ねた(n=1560)。本報告は、科研費基盤研究(B)「『高さ』を疑う、『高さ』を背負う: 新しい都市ガバナンスの社会学」(19H01557)の成果の一部である。 3. 結果  コロナ禍2年目(2021年度)は、コロナ禍1年目(2020年度)よりも全体としてイベント参加数が増大し相対的な「再開」傾向が見られた。他方、第4回緊急事態宣言中・五輪開催期(2021年7月~2021年9月)とオミクロン株流行(2022年1月~2022年2月)の期間には、顕著な減少が見られた。感染拡大と緊急事態宣言の波のなかで、状況に応じてスペースからの離脱と一時的な再開を繰り返すイベント参加者のあり様が浮き彫りとなった。  また、イベント「自粛」のなかでも、個々の日常でのイベント参加の位置づけが大きい、あるいは特定のスペースや交流を重視する意識をもつ「コア層」と呼べる人びとは、1年目・2年目のいずれも参加を維持していた。ただし、クラブ・ライブハウスや講演会・ボランティア、同人誌即売会に熱心に参加していた層では、イベント参加の減少が続くなど、コロナ禍2年目には「再開」するスペースと危機の長期化するスペースの違いも顕在化した。 参考文献 石毛弓編,2021,『コロナ禍における体験型イベント』水声社. 町村敬志・長島祐基・栗原真史・杉山怜美・髙橋絢子・辰巳智行・Fung Wan Yin Kimberly・山内智瑛,2021,「COVID-19「自粛」とイベントスペース――「東京イベントスペース2020」データ分析から」『一橋社会科学』13: 91-115.

報告番号188

COVID-19「自粛」によるナイトライフ利用者の再編――新型コロナ禍におけるイベントスペース利用者調査2022から(2)
一橋大学大学院 山内 智瑛

1. 問題関心  新型コロナウイルス(COVID-19)流行に伴い、政府は感染源の一つとなったクラブ・ライブハウス・バー等の「ナイトライフ(nightlife)」を「夜の街」として新たにカテゴライズし、営業「自粛」を要請した(町村ほか 2021)。しかし、このように夜を昼とは別の時空間として周辺的に位置づける態度は「昼中心主義(daycentrism)」(Gallan & Gibson 2011)であると昨今批判されており、近年の社会学および隣接領域では、夜が昼の資本主義システムと交わりながらいかに変容しているのかという点に焦点が当たっている。ナイトライフを都市経済成長のための資源とする「夜間経済(night-time economy)」政策はそうした事例の一つであったが、COVID-19流行は当政策を新たな局面へと向かわせている。COVID-19は夜間経済政策、事業者、そしてナイトライフを消費する主体をいかに再編するのか。本報告は先行研究であまり取り上げられていない利用客に着目し、どのような人びとがコロナ禍においてナイトライフ(クラブ・ライブハウス)を利用しているのかを検討する。 2. 方法  報告者ら調査グループは、2022年2月に首都圏における各種イベント参加者を対象に、webモニター調査「新型コロナ禍におけるイベントスペース利用者調査2022年」(n=1560)を実施し、コロナ禍での利用のあり方や日頃の感染対策、社会に対する意識等を尋ねた。その中で本報告は、2019年以降もっとも熱心にクラブ・ライブハウスを訪れた人およびクラブ・ライブイベントに参加した人を「クラブ・ライブハウス利用者」(n=228)として分析の対象とした。以上の調査は、科研費基盤研究(B)「『高さ』を疑う、『高さ』を背負う――新しい都市ガバナンスの社会学」(19H01557)の一部である。 3. 結果 クラブ・ライブハウス利用者の全体的な傾向として、「場所重視」の度合いが高ければ高いほど(e.g.「個人的に気に入っている特定の会場がある」・「参加している組織や団体、サークルでよくお世話になっている特定の会場がある」)、コロナ禍以降も利用を継続していることがわかった。詳細に検討すると、「場所重視」の度合いが低く同居子供人数が多い人はコロナ禍においてクラブ・ライブハウスを一切利用しなくなる一方、「場所重視」の度合いが高く感染リスクを低く見積もっている人びとはコロナ禍でも一貫して利用を継続していた。さらに、コロナ禍長期化に伴う社会的孤立リスクを高く見積もっている人は、緊急事態宣言が発令されていない時期に利用する傾向にあった。以上からコロナ禍を通じて、クラブ・ライブハウスを自らの「居場所」と考える利用客が純化したと考えられる。 参考文献 Gallan, B. & C. Gibson, 2011, “New Dawn or New Dusk?: Beyond the Binary of Day and Night,” Environment and Planning A, 43: 2509-15. 町村敬志・長島祐基・栗原真史・杉山怜美・髙橋絢子・辰巳智行・Fung Wan Yin Kimberly・山内智瑛, 2021, 「COVID-19『自粛』とイベントスペース―『東京イベントスペース2020』データ分析から―」、『一橋社会科学』(13): 91-115.

報告番号189

新型コロナ禍の長期化に伴う対面・オンラインでの同人誌即売会の参加者実践の変容――新型コロナ禍におけるイベントスペース利用者調査2022から(3)
慶應義塾大学大学院 杉山 怜美

1. 目的  同人誌即売会(以下,即売会)は,非商業資本による自主制作を特徴とする同人活動の「作品発表」や「交流」(東 2013)の機能を担う場として,45年以上の歴史を持つ.対面での集まりを伴う即売会は,新型コロナ禍(以下,コロナ禍)1年目は当初中止や延期を余儀なくされ,その後オンライン開催や規模を縮小して対面での再開が試みられた(町村ほか 2021).  中村仁(2021)は即売会のようにリアルなコミュニケーションを主体とするイベントをオンラインで成立させることは難しいと指摘している.コロナ禍が3年目に入り長期化している現在でも,即売会は対面・オンライン双方で継続的に開催されているが,コロナ禍以後の実践の変化を捉えるためには,参加者によるイベントの位置づけを明らかにする必要がある.  そこで本報告では,コロナ禍2年目以降の対面・オンラインの即売会への参加状況や参加者による位置づけに着目して,コロナ禍の長期化に伴って即売会の実践がどのように継続され,また見直されてきたのか検討を行う. 2. 方法  報告者らは2022年2月にWebモニター調査(「新型コロナ禍におけるイベントスペース利用者調査2022年」(n=1560))を実施した.また,報告者は2020~22年に対面・オンライン即売会での参与観察を,2021年12月・2022年6月にコロナ禍以前に即売会参加経験がある人を対象にインタビュー調査を実施した.本報告は,科研費基盤研究(B)「『高さ』を疑う,『高さ』を背負う:新しい都市ガバナンスの社会学」(19H01557)の成果の一部である. 3. 結果  Web調査で2019年~調査時点に対面の即売会へ参加経験がある人は129名いたが,コロナ禍以後(2020年3月~)では71名に減少した.即売会を最も熱心に参加したイベントだと回答した人に限定しても,2019年~調査時点では60名いたが,コロナ禍以後は28名に半減した.コロナ禍以後のオンライン即売会については,コロナ禍以後の対面の即売会への参加者の71.8%が参加した一方で,不参加者の87.9%は不参加であり,どちらも参加するか,しないかに二分される傾向がみられた.  加えて,コロナ禍による行動規制が撤廃されたときに参加を希望する対面:オンラインの比率の平均は,即売会に最も熱心に参加した人で約7:3,それ以外の人でも約6:4となり,対面を重視する傾向が根強かった.コロナ禍以前に即売会参加経験がある人へのインタビュー調査では,オンライン即売会は同人誌の購入機会やジャンルの盛り上がりに寄与するが,交流を行うハードルが高く,参加しても物足りないという語りがあった.  以上から,オンライン即売会は対面の即売会の代替としてよりも「作品発表」などその機能の一部を担うものとして位置づけられていると結論づけられる. 参考文献 東園子,2013,「紙の手ごたえ――女性たちの同人活動におけるメディアの機能分化」『マス・コミュニケーション研究』83: 31-45. 中村仁,2021,「日本発ポップカルチャーを通じたファンコミュニティの交流――COVID-19とイベントのオンライン化に関する検討」石毛弓編『コロナ禍と体験型イベント』水声社,33-56. 町村敬志・長島祐基・栗原真史・杉山怜美・髙橋絢子・辰巳智行・Fung Wan Yin Kimberly・山内智瑛,2021,「COVID-19「自粛」とイベントスペース――「東京イベントスペース2020」データ分析から」『一橋社会科学』13: 91-115.

報告番号190

美の作用――茶の心と大枠
経済総合分析株式会社 木下 博之

1. 目的 美の作用の功罪について、権力者が体制を維持するための努力から捉えていく。それは、正の意味では、未来への萌芽であり、負の意味では、嫉妬である。ヴェルサイユ宮殿を見ればわかるように、権力はあり方たるその器を美しくそして大きく建設することで人心をつなぎとめようとしがちである。ルソーやサン=シモンは、それは重税を伴い市民を圧迫することを嘆いていた (ミュッツ, p.36)。国王のルイ16世の妃のマリー・アントワネットは、「パンがなければブリオッシュを食べたら良い。」という言葉で括られ人心を逆なでしてしまった。美の作用が、むしろ、その機能を疑われて、あげくには反感を抱かれて打倒されてしまったのである。ただ、ルイやマリーも、ただ、贅沢をもさぼっていたわけではなく、人知れず、未来を開こうとやり方を模索していた。こうした、あり方とやり方との行き違いは、前近代の教訓たるに止まらず、近代たる今日においても繰り返されており、未だに問題の根本的な解決は見られない。そこで、本論では、美術館や学校などのインフラのあり方とその活用方法ととしてのその道の究め方と伝術の仕方といったやり方とが、いかに大枠において結びつき、美を作用させていくのかについて吟味していく。 2. 方法 表面的なあり方の美とその裏にあるやり方の模索の双方を捉え、それらの連関をつかみ、さらに、それらが織りなす大枠を示していく。日本の戦国時代から江戸幕府樹立までの信長、秀吉、家康の頃と、フランスでのフランス革命の流れと影響を題材とし、政治と文化の関係を捉える。 3. 結果 秀吉は伏見城に、ルイ16世はヴェルサイユ宮殿に、あり方を豪華絢爛にして人心をつなぎとめようとする姿勢が表れている。一方で、秀吉は茶道に、マリーは服飾に、そのやり方の道筋を見出そうと努力している。秀吉は茶道において、大枠を自分に引き寄せようと家康の前で振舞った(田中, p.160)。洋の東西と時代の前後を超えて共通の枠組みを見出すことができる。 4. 結論 美とは、より良い生きざまである。洋の東西を問わず、権力者は美を誇示することで人心を従わせようとしてきた。その裏では、その美の根拠と今後の展開を必死に模索してきたのである。茶の心からは、見た目の美しさはもとより、未来への道筋を見出そうとする、暗闘苦闘の跡を偲ぶことができる。美はそのあり方そのもので、人々を引き付ける。権力と権威を得た後には、そして、それを続けていくためのあがきともいえるやり方の模索という営為が始まるのである。あり方とやり方とは、層と元を成し、こうして葛藤するだけの無為なる関係ではない。相互を参照して相互作用しつつ未来へ向かっていく、美の作用が今後とももたらされていくことを期待する。 参考文献 ピエール・ミュッソ、杉本隆司(訳) 『サン=シモンとサン=シモン主義』、白水社、2019年。 田中仙堂 『お茶と権力』、文芸春秋、2022年。

報告番号191

アート、外密性、社会
同志社大学 落合 仁司

アート、外密性、社会 目的  アートは、人間の親密なるもの intime を表現する創作の活動であると同時に、人間の親密なるものに印象を刻印し享受される対象である。言い換えればアートは、親密なるものを外部化すると同時に、親密なるものに内部化される。このアートの在り様を適切に表現する言葉として、外密性 extimité が、ジャック・ラカン及びセルジュ・ティスロンによって造語された。  ラカンは外密性によって、人間の親密なるものに内部化され享受される対象aを表現する(『セミネール』VII)。ティスロンは外密性によって、人間の親密なるものが外部化され表現されるメッセージを言い当てる(『過剰に露出された親密性』)。外密性はまさしく親密に享受されうる外部の対象aであると同時に、親密性を外部に露出するメッセージである、現代アートに相応しい述語である。 方法  親密 intime はラテン語の内部(形容詞)interus の最上級intimusを語源とし、ラテン語の外部(形容詞)exterusの最上級extremusを語源とする過激extrêmeと対立する。外密 extime はこの親密=内部の内部=内密と過激=外部の外部=外顕を合成した造語である。ちなみにまだ辞書には載っていない。ラカンとティスロンはこの造語によって、外部の対象aを内部化する親密あるいは外部化され内部が露出される親密という運動を表現しようとした。  しかし親密が露出される外部とは、あるいは親密に享受される外部とは、社会のことではなかったか。すなわち外密性とは親密と社会を結び付ける運動を表現する言葉ではないのか。したがって表現されるものであると同時に享受されるものであるアートは外密性と言う概念を媒介として社会と接合される。アートと社会の関係を論じるためにこそ外密概念は創造されたとも言えよう。 帰結  アートを対象とする学としての美学は、外密性の概念を通じて、社会学と接合する。報告者は『美学の数理―美の消費と芸術の生産―』(晃洋書房2022)において現代アートを生産し消費することの美学を提示した。その成果を踏まえ、本報告は美学と外密性の出会いが何を帰結するかを検討する。この帰結は『外密美学』とでも言うべきアートの社会学あるいは社会の美学として議論される価値があると愚考する。 文献 落合仁司, 2022, 『美学の数理―美の消費と芸術の生産―』, 晃洋書房. Lacan, Jacques, 1986, Séminaire VII L’Éthique de la psychanalyse, Paris, Éditions du Seuil. Tisseron, Serge, 2001, L’intimité surexposée, Paris, ―, 2008, Virtuel, mon amour, Paris,

報告番号192

音楽コンテストの研究
株式会社博報堂 森 泰規

アートは社会学になるのか?という本テーマセッションの問いに検討を加える. 社会学が取り扱う基本要素(階層性,共同性,システム性,生態系内在性,庄司(2016)参照)を念頭におくと,同じアートに携わる芸術家どうしの序列を設け(階層性),芸術家どうしの認識共同体が一定程度強固であり(共同性),能力開発とキャリアビルディングの方法論が固定化しやすく(システム性),しかしそもそも身体を駆動させて成果を生みだす点でいやおうにも自然の一部であることを認識させられる(生態系内在性,ただこれは身体性と呼び変えてもよいだろう),という「アートのコンテスト」はまぎれもなく社会学の対象となる.少なくとも上記に例示した内容に該当する「クラシック音楽のコンテスト」は対象となろう. コンテストは,開催される時代をリードする主要な人材(たとえば主要交響楽団や音楽大学での教授陣)が審査に参加し,「何をよしとするか」を世に示すものでもあり,いわば美の規範を示すものである.もちろんそうした規範を示すだけでなら,古くは勅撰和歌集,今日では主要文学賞のようなものがある.しかし身体性を伴う生の表現に対し加えられる評価である(生態系内在性)ことが,過去の作品に対して評価することもある勅撰集とは異なり,出場者と審査員がしばしば師弟関係にあること(システム性)は文学賞と異なる.一方受賞後の地位達成や,審査員の認識共同性については,文学賞と同様の傾向がある面も,考えられる. 今回報告では主要音楽コンテストの採点結果のうち,公開されているものを計量的に把握し,審査員間の判断にばらつきがあるのか,それとも一貫性があるのか,を検討後,可能な範囲で出場者の受賞後地位達成について検討する.一例を2002年に開催された日本音楽コンクールのある部門についてみる限り,通説に言うほど評価は分かれていなく,当時音楽大学の教授陣,管弦楽団の中核を担う演奏者を中心に編成された審査員は一定の方向に認識合意をしていたと推察される(共同性).わずかに審査員の嗜好性によるばらつきがみられても,11名の審査員の合算値を用いるため,個々審査員の影響力が相対的に小さく,有意な水準とは判断できない.なお,受賞者の地位達成についてみると,20年後の今,一位と二位の出場者は主要交響楽団の首席奏者(常勤ポストのうち最も格の高いもの,と考えてよい)に就任している.ただ三位以下については同じ傾向があてはまらない(階層性,システム性.効果についての限定的な影響). また,もう一例を2021年のショパンコンクール最終審査の3次予選(https://s3.eu-central-1.amazonaws.com/media.chopin2020.pl/c6a82597da4b4faabfcc744f1ac13e28.pdf, 最終参照2022年6月14日)について求めるとやはりこちらも出場者23名に対し,審査員16名の評価をクラスカル・ウァリスによる検定を行う場合,p > .01 となり,帰無仮説(出場者の得点の分布は審査員によって同じである)を棄却できず,こちらの例でも審査評価にばらつきがあったとはいえない.過去当該コンテストは評価をめぐって審査員間の軋轢や審査棄権といった事態をもたらしたことでしられるが21年の第三次予選においては審査採点にはほぼ一定の合意があった内容といえそうである。 文献 庄司興吉,2016,『主権者の社会認識』東信堂

報告番号193

美的関心におけるコスモポリタニズム――見知らぬ人への信頼とナショナリズムの効果の比較
台南應用科技大学 矢崎 慶太郎

本報告は、どのような社会的関心から美術に対する関心が生まれるのかを量的調査のデータを用いて明らかにする。これまでこの問いは、心理学においては説明されてきたが、社会学的な分析枠組みでは充分に説明されてきたとは言えない。そこで本論は、これまでニクラス・ルーマンの社会システム理論における芸術の社会的機能としての「偶発性の観察」に着目する。ファンタジーや未知のもの、主観的経験を含め、「見えないもの」を見るという偶発性の観察は、美術に固有の社会的動機を説明するのに役立っている。しかしこの概念の問題点は、測定の困難さにある。とくに質問紙調査において回答者にこの抽象的・専門的な概念について質問することは不可能である。そこで本研究は、「見知らぬ人の信頼」を代替の変数として用いる。この指標は通常、橋渡し型のソーシャル・キャピタルを測定するために用いられるが、他方で美術史的に言って「見知らぬ人(Fremde)」は、芸術に深く関連する概念でもある。本研究は偶発性の観察と見知らぬ人への信頼の概念は理論的に重なり合うことを指摘する。  したがって、見知らぬ人への信頼が高い人ほど美術への関心も高いというのが本報告の仮説である。  ただし「見知らぬ人への信頼」は、一般的回答者にとって理解(回答)可能であるものの、この概念もそもそも極めて抽象的である。誰が「見知らぬ人」なのか、その定義は回答者によって大きく異なる可能性が予想される。そこで本報告では、見知らぬ人への信頼と愛国心(national pride)の交互作用効果を検討する。ルーマンのいう「偶発性の観察」という機能を持った近代芸術は、同時に地域や民族性をも超えたコスモポリタニズム的な要素、つまり「世界芸術(Weltkunst)」としての特徴を兼ね備える。こうした点に着目すると、見知らぬ人への信頼が高いほど、美術への関心は高まるというポジティブな関係は、ナショナリティに対する誇りを持たない人々に限定されるということが予想される。  この仮説を検証するために、本研究は、台湾の芸術系大学の学生を対象にした授業内アンケート調査を用いる。代表性のない調査データではあるが、報告者はこの大学の教員であり、学生(回答者)たちの日常的な芸術経験についてある程度把握している。芸術に対する理解や経験は世代や集団によって異なりうるため、まずは現代美術に接触する頻度の高い学生たちを対象に調査した。  分析は重回帰分析を用い、その交互作用効果を検討する。分析結果は、見知らぬ人への信頼は美術への関心にポジティブな効果を持つという仮説を支持した。また見知らぬ人への信頼と愛国心のあいだにはネガティブな交互作用効果があることも明らかになった。 以上、本研究は美的関心にはコスモポリタニズム的な要素が含まれていることを指摘する。この分析結果から、芸術と社会との関係をどのように考えることができるのか、また美術に関連する教育現場において、学生たちにどのように社会との関連性を想起させることができるかについて提起したい。

報告番号194

流動形態の前衛芸術家のスモールワールド――メールアート・コミュニケーションの社会ネットワーク分析
京都産業大学 金光 淳

【1.目的】リトアニア出身の奇人ジョージ・マチューナスがオーガナイザーとなり1960〜70年代に活躍したFluxusという前衛芸術家ネットワークは、現在の関係性アートやソーシャリ・インゲージド・アート(SEA)の基盤を作ったと言われる(Bishop, 2011)。このアーチストの集合体は、ジョン・ケージのクラスを母体としてNYCに集うアーチスト、欧州のアーチスト、日本のアーチスト集団(ハイ・レッド・センターなど)などが融合したという側面がある。「メンバー」にはヨーゼフ・ボイス、ナムジュン・パイク、オノ・ヨーコなど名を連ね、日本人が多かった。1964年に渡米した塩見允枝子もその一人である。塩見は1年間のNY滞在の間、仲間と様々なアートイベントを行ない、マチューナスからの信頼も厚かった。今回の研究は、この前衛芸術家集団がいかに形成されていったかを、塩見允枝子のエゴ・ネットワークの分析などによって迫る。 【2.方法】塩見允枝子の『Spatial Poem』という詩集形式のメールアート作品は、マチューナスからもらった名簿をもとに、日本への帰国後、世界に散らばるアート関係者に向けて発送したパフォーマンス(=「イベント」と呼ばれる)を呼びかける招待メールに対する返信メール記録作品である。イベントは1965年から1975年まで9回行われ、各イベントで行われたパフォーマンス内容と時間と場所が「詩」の形で世界地図にプロットされている。その作品データから抽出した200人超の回答アーチストとパフォーマンスを行った場所のデータ(=二部グラフデータ)に加え、大阪在住の塩見允枝子自身に依頼した各アーチストとのイベント共催参加、影響関係などの関係性を尋ねるエゴ・ネットワーク調査(2021年実施)でデータを収集した(これらのデータは美術史的にも極めて貴重なデータである)。各アーチストとの関係性は強弱の絆に分類された。社会ネットワーク分析とロジスティック回帰分析などの統計的な分析が行われた。 【3.結果】1)1960年代のイベントでは回答者はNYC周辺に集中していたが、1970年代には世界中に分散した。2)回答者は1960年代では「純粋な」フルクサスメンバーが中心だったが、1970年代には狭義のフルクサス・メンバーを大きく超えて拡大している。これは所謂「イノベーションの普及過程」(Rogers, 2005)を表しており、少数の凝集的革新者集団から前期採用者への移行が拡散的に起こっていることを示す。また、以下の興味深い知見を得た。3)そもそも「回答」の多くは元の名簿にすらなかった「弱い絆」によるものでありコアなFluxusメンバーの回答は少ない。4)イベントの参加が減っていったときに強い絆による「バネ」が働き、強い絆の割合が一時的に高まり参加者も増えるが、その後は強い絆が減り、弱い絆が増えていく。 【4.結論】 Fluxusネットワークは、アートの中心がパリからNYCに移動すると同時に、急速に進んだ文化の世界同時現象の最中で、アーチスト間の強弱の絆で拡大し、やがて収縮する流動的状態にあった。関係性アートはこのように世界に拡大していった。これらの知見は、イノベーションの普及理論を補強するばかりでなく、ソーシャルキャピタル論、社会運動ネットワーク論などとも接合しうる。Milgram(1967)のスモールワールドのメール実験が行われた時期であり、それとの比較も極めて興味深い。

報告番号195

パーソナルデータ利用の正統性と説明可能性への期待――AIの意思決定支援の正統性と透明性(1)
公益社団法人国際経済労働研究所 山本 耕平

【1.目的】「説明可能なAI」というタームの浸透が示すように、AIをめぐる議論では、AIによる予測や判断の「説明可能性」がしばしば問題として取り上げられる。本報告は連携報告のイントロダクションとして、そうした説明可能性をめぐる態度が人びとのAIにたいする判断においてどのような役割を果たしているのかを、調査データの分析から探索的に検討する。 【2.方法】2022年3月に実施された「科学技術に対する意識調査」のデータをもちいて、回答者がAIによる判断に説明可能性を求めるかどうかによって、その回答者がパーソナルデータ(以下、PD)利用に認める正統性に差があるかどうかを検証する。同調査は20~69歳の男女を対象とするインターネット・モニター調査であり、回答者は性別・年齢・居住地(地域ブロック)の構成比率が人口推計に一致するよう割当抽出された。 同調査では、「犯罪をおかしやすい人を予測すること」など4つの項目について、(1)AIによってその予測が可能になれば社会がより良くなると思うか、(2)その予測のために回答者自身のPDが利用されることを認めるか、を尋ねており、各回答者の態度が4対の質問項目によって反復測定されるマルチレベル構造のデータが得られる。また、(犯罪をおかしやすい人をAIによって予測することについて)「AIの予測を参考にするとしても、最終的には経験のある刑事が判断すべきだ」という意見への賛否も尋ねており、この質問への回答を、AIによる判断に説明可能性を求める度合いと解釈して分析にもちいることができる。各回答者において、上記の(1)と(2)には正の相関がある(回答者がAIによる予測が良い結果をもたらすと予想する領域ほど、その回答者はPDの利用を認める)ことが予測されるが、AIによる判断に説明可能性を求める度合いの違いによってその相関が異なるのかどうかをマルチレベル分析により検証する。 【3.結果】分析の結果、平均的には各回答者において上記の(1)と(2)に正の相関があり、社会への良い影響が期待できない領域についてはデータの利用を認めない、良い影響が期待できる領域については認めるというように、各回答者はPD利用が正統化される領域とそうでない領域を弁別していることが示唆された。一方、この傾向と、AIによる判断に説明可能性を求める度合いとのあいだの交互作用は認められなかった。さらに、AIによる判断に説明可能性を求める回答者ほど、全般的にPDの利用を受容する傾向が見られた。この傾向は、回答者の黙従傾向を統制してもなお確認された。 【4.結論】回答者は全体としてPD利用が正統化される領域とそうでない領域を弁別しているが、AIによる判断に説明可能性を求める回答者ほど全般的にPD利用を受容する傾向にあった。この結果は、回答者がPDを利用した予測の受容可能性を判断する際に、それらの予測においても最終的には人間が判断をくだすものと想定していると考えれば説明できる。この解釈が正しければ、説明可能性への期待がPDを用いた各種の予測の正統性を支えている、と考えることができる。

報告番号196

民主政におけるAI の決定理由と人間の受容理由――AIの意思決定支援の正統性と透明性(2)
東京都立大学 堀内 進之介

1 目的 人工知能(AI)は,ある分野では人間の意思決定より優れている.特に複雑な問題を解決するために,高度な戦略的推論や膨大なデータの分析が必要な場合は,この傾向が顕著である.しかし,プライバシーや監視,偏見,不公平,不平等などの弊害が露見したことで,AIの「ブラックボックス問題」に耳目が集まり,AIに透明性や説明可能性,解釈可能性などを求める「説明可能なAI」が議論されている.こうした議論に対して,本報告はとくに政治分野でのAI利用に注目しつつ,その要求が過剰ないし不当となる可能性を主張する. 2 方法 本報告の基本的スタンスは,説明可能なAIや民主主義や特定の民主主義の形態を,それらの効果を評価する最初の段階で究極の目標にはできないということに基づく.このスタンスに反する仮定から出発すると,それらに挑戦するものは必然的に悪いものとなってしまう.より実りあるアプローチは,それらの価値に対して不可知論的な立場をとることである.そうすることで代替案を評価し,議論を活性化できる. 3 議論 政治は,人間が直面する最も複雑な問題を扱い,短期的な結果と長期的な結果のバランスを取る必要があり,その結果を完全に理解できないことを承知の上で,意思決定を行わねばならない.AIはこうした最適化問題への貢献が期待されており,実際,政府の大半の領域でAIがすでに採用済みとの報告がある.無論そこでもブラックボックス問題は浮上している.特に政治では,AIの透明性や説明可能性は,民主主義的な(認識論的な)正統性と抜き差しならない関係にある.ブラックボックス問題を危惧する論者は,人間がAIの決定を吟味し,最後の決定権を持つべきことを強調している.しかし,そこには困難がある.なぜなら,意思決定システムとしてのAIの出力に関する理由や説明(因果的な意味:How)と,影響を受ける有権者が納得するような正統性(正統化的な意味:Why)は必ずしも同一ではないからである.そこで本報告では民主主義的な正統化論を踏まえ,AI の決定理由と人間の受容理由の関係を再考する. 4 結論 論理的な結論として,問いへの正統化的な意味(Why)を重視すると,第一の場合には,政治分野(そして多分野)におけるAI利用の使用は冗長になる(=AIの有用性を著しく制限する)可能性が生じ,第二の場合には,政治的アクター(そして一般市民)が受容可能なようにHowからWhyを導出する「モデルのモデル化」が進み,それが問題のある決定における免責装置になる可能性が生じる.そこで本報告では回避策の一つとして,民主主義的な正統化要件,例えば参加可能性や学習可能性を政治過程から切り離し,OpenAI(共創AI)などへの議論に接続することで,政治分野でのAI利用と民主主義的な正統性を結び直す可能性を示す. 参考文献 Krishnan, M. (2019). Against Interpretability: A critical examination of the Interpretability problem in machine learning. Philosophy & Technology, 33(3), 487-502. Sætra, H. S. (2019). A shallow defence of a technocracy of artificial intelligence. SSRN Electronic Journal. Zerilli, J., Knott, A., Maclaurin, J., & Gavaghan, C. (2018). Transparency in algorithmic and human decision-making: Is there a double standard? Philosophy & Technology, 32(4), 661-683.

報告番号197

何がその背後に隠されているか――AIの意思決定支援の正統性と透明性(3)
東京女子大学 赤堀 三郎

1 目的 •本報告は、JSPS科研費「信用スコアの受容に関する社会学的研究」(課題番号20H01582)の成果の一部であり、解釈と受容意向という側面に注目し、新しい技術の社会的影響という広範なテーマを扱える社会学理論の創造を目的とするものである。 •本連携報告は、AI の意思決定支援の正統性と透明性という共通論題を設定し、報告者各々の観点からこの問題を扱う。本報告は山本報告、堀内報告に続く第3報告として、先行する2報告から得られた知見を踏まえつつ、AIと意思決定との関連を社会学は今後どう扱っていけるのかという問いに答える。 2 方法 •先行研究においては、AIというテーマに関して、解釈的社会学のアプローチの適用がなされている。本報告もこれを踏襲し、何がAIであるか(AI研究者・技術者にとって何がAIと呼べるか)とは独立に、AIに関する解釈枠組(AIが人々にどう解釈されているか)を問う(解釈の解釈=メタ解釈)。 •AIの意思決定支援というトピックに絞って言えば、信用スコアをはじめとする各種スコアリングも、予測的ポリシングも、セルフトラッキングの実践も、人格の数値化=データ化であって、そこには「世界の二重化(=複製)による行為や思考のパターン変更」を見出すことができる。さまざまなソースから収集されるデータの集積に基づいて構成される個々人のプロファイルと「リアルな」個々人との二重化プロセスを指すものとして「データダブル」という言葉もつとに使われている。そこで本報告では「二重化」というキーワードを導きの糸とし、メタ解釈の手法によって、AI解釈においていわば盲点となっているところへとアプローチする。 3 結果 •AIによる判断受容の是非については、「人間による最終判断」が分岐点となっている。「二重化」テーゼにおいては複製されるものと複製されたものとの区別が見られるが、ここから、AIをめぐる一連の解釈図式において背景化されているものが何であるかを指摘することができる。すなわち、意思決定するのは誰(あるいは何)であるか、あるいは何が意思決定と呼ばれているか、AIと「人間」との差異はどう引かれているか、といった諸点である。 4 結論 •AIが「社会」に何をもたらすかという問いについては、考え方としてのAIが合理主義の系譜に属することから、しばしば「近代性に付随する問題であって、今に始まったことではない」といった捉え方が見受けられる。だがこういった答えでは十分とは言えない。AIと意思決定との関連を社会学は今後どう扱っていけるのかという本報告が掲げた問いに対する答えは、「AIがもたらすのはユートピアかディストピアか」といった議論とは別に、「どこまでが近代の問題で、どこからがAI時代特有の問題なのかを、社会学は見極める必要がある」となる。 •AI時代の社会学への展望として、(1)「人間」概念の解体もしくは拡張、(2)「自己」および「意思決定」の概念の見直し、そして両者の帰結として、(3)「人間」からスタートする理論構成の放棄が求められることになるだろう。

報告番号198

アレクサ搭載のEcho Showを用いた遠隔コミュニケーション実験――アレクサが,誰が現在の期待された聞き手であるのかを社会的に判別しないこと、に起因するトラブルの研究
神戸市看護大学 樫田 美雄

(1)発想の前提: 我々の生活の中で使われている「機械」は「設計どおり」に使われているわけではない.「ブリコラージュ(器用仕事)」的利用こそ,人間の環境/道具利用のむしろ基本形である.それは,一見取扱説明書通りの利用に見えるスマートスピーカーても同様であり,実際の道具利用状況は,多様なので具体的な操作場面から把握する必要がある.そこで我々は,ビデオ・エスノグラフィー研究をすることにした.即ち,アマゾン社のEcho Showを用いたコミュニケーションに関する実験的調査を行い,分析を行った. (2)調査の概要: 具体的な実験調査場面は以下のとおりである.まず,80歳代半ばの単身居住女性である被験者Xさん宅に,ディスプレイ付きのスマートスピーカー端末を置いてもらい,Xさんに対して支援者A,Bがその利用法を教授した.そして,機器の利用場面と,機器の利用法の教授場面の両方を記録した. 場面としては,以下の6種の場面を撮影した. まず,第一に,Echo Show上のAI「アレクサ」に話し掛けて,天気予報等を聞く「アレクサ単純利用」場面(場面1). ついで第二に,Echo ShowをXさんと別居するお孫さんであるIさんにも送付し,XさんとIさんとでビデオ通話をしてもらう「アレクサ経由自由ビデオ通話」場面(場面2). そして,第三に,Xさんに場面1の内容をIさんに報告する課題を与えて,実行してもらう「アレクサ経由課題遂行ビデオ通話」場面(場面3) そして,第四に,Iさんから指示を受けながら,Xさん宅内の機器の操作を試みる「遠隔ICT教授」場面(場面4). 第五に,外に出た研究スタッフとZoom経由でコミュニケーションをしながら,Xさんが俳句を作る「遠隔代理吟行」場面(場面5). 最後に第六に,上記で作成した俳句をネタに,XさんとIさんとで,テレビ通話をしながらお茶会をする「遠隔お茶会」場面(場面6).この6場面である. (3)結果:アレクサが,誰が話者であるのかを社会的に判別しないことによるトラブル  本報告では,場面的固有性をもったトラブル2ケースに注目する.  第一のケースは,Xの支援者AやBが,アレクサに聞こえるか聞こえない位の小さな声でXに話す内容を,アレクサが自分に対しての発話であるかのように聞き,反応してしまった,というケースである.  第二のケースは,支援者Bが示唆する,被験者Xの働きかけの対象が,「目の前のEcho Show」ではなく,その「Echo Show」の向こう側に存在しているリアルな人間(孫I)に変化しているのにもかかわらず,被験者Xがこの状況の変化に適切に対応できない事態がおこってしまった,というケースである. (4)まとめ:  本研究では,「アレクサ」における,有能さと無能さのバランスの悪さが問題として発見された.つまり「アレクサ」は,人間とは異なり,場面内で誰が誰に話しかけをしているか,という「状況理解」をしていない.にもかかわらず,「アレクサ」は,「問いかけに敏感に反応」し,「あたかも状況を知悉しているかのように」反応する.この有能さと無能さの落差が,被験者Xほか関係者を混乱させているようだった. なお,本発表の一部は,すでに加藤・加戸・樫田2022「遠隔コミュニケーションに関連した共同作業のビデオ・エスノグラフィー」として『現象と秩序』16号に発表されている.

報告番号199

オンライン子育て支援の相互作用(1)オンライン子育て支援における「わが子の報告」
兵庫教育大学大学院 永田 夏来

目的  本報告の目的は、オンライン子育て支援で行われている「わが子の観察」の語りに注目し、子どもを持った親(主に母親)が一度取得した「(母)親役割」を手放し、「自分らしさ」と組み合わせて再構築する過程について記述することにある。子供を持つことによる家族機能の変化とそれに伴う夫/妻役割の移行は、国内では家族周期の一部として位置付けられており(森岡 1973)、海外においては心理学者らによる継続調査からその様子が記述されてきた(Cowan & Cowan 1992=2007, Belsky & Kelly 1994=1995)。これまでの研究では、子どもを持つという変化は妻だけではなく夫にも大きな葛藤をもたらすものであり、乗り越えることが難しい移行であると指摘されている。女性の社会進出が進み、高学歴でフルタイム就労していることも珍しくない現在の日本社会において、この困難はますます深刻になっているものと思われる。 方法  本報告で用いるのは、関西に基盤を持ち子育て支援を行っている法人Aが2020年より実施しているオンラインワークショップ事業でおこなった調査データの一部である。この事業は0〜6才の未就学児とその親を対象としており、全10回からなるプログラム(各回7-80分程度)が月に2〜3回、無料にて行われている。調査者は2020年10月より半年間ワークショップと各種ミーティングに参加し、アーカイブの提供を受けた。また、15名の協力者によるグループインタビューと10名を対象にした個別インタビューをおこなった。現在もワークショップ全参加者(2022年6月時点で85名)を対象としたアンケートを実施しながら、各種MTG等に参加している。 結論  個別インタビューからは、子育てに集中するあまり子どもの要求を先読みしようとしていたこと、子供と一体になりすぎていたことが反省的に語られ、ワークショップを通じて子供と適切な距離感を獲得したことが肯定的に語られた。これは法人Aによる「わが子を観察し、報告する」プログラムの成果であるといえる。また、各家庭をWEB会議システムでつないだ状態で互いに「わが子の報告」を実施・傾聴することは、参加者の一体感を形成しながらも境界線を保ち、結果として相手を尊重する効果があることが語られた。 森岡清美,1973,『家族周期論』培風館 Belsky, J., Kelly, J., 1994, The Transition to Parenthood: Delacorte Press.(安次嶺桂子訳,1995,『子供をもつと夫婦に何が起こるか』草思社.) Cowan, C.P,Cowan, P.A., 1992, When Partners Become Parents The Big Life Change For Couples:Routledge.(山田昌弘・関内文乃訳, 2007, 『カップルが親になるとき』勁草書房.)

報告番号200

オンライン子育て支援の相互作用(2)――「我が子の報告」を可能にする装置としてのWeb会議システム
神戸大学大学院 若狭 優

【目的】ZoomやCisco WebEX等に代表されるWeb会議システムはしばしば、パブリックとプライベートの境界を溶融していると指摘される。この特徴はWeb会議システムを介した相互行為に様々な問題を生じさせていると言われるが、オンライン子育て支援の場では、むしろ、プライベートな空間(自宅)がオンラインで接続されることで、安らぎを得ているということが調査の結果、明らかになっている。こうしたWeb会議システムの効果は「パブリック/プライベート」という区分では捉えきることができない。そこで本報告では、アーヴィング・ゴフマンの相互行為論において展開された自己に関する区分、「人間/個人」を採用し、Web会議システムの相互行為がどのようなものかを自己論の視点から捉え直すことを試みる。 【方法】ゴフマンは参与者が相互行為の過程の中で達成しなければならない自己を「人間」と「個人」に区別している。「人間」とは、「ふつうの外見」に代表されるように、人間一般に関する自己イメージであり、「個人」とはかけがえのない唯一無二の個としての自己イメージである。相互行為ではこれら二つの要素が同時に達成されなければならない。それはビデオチャットの相互行為でも同様である。こうした自己論の観点から、Web会議システムのハードウェア・ソフトウェア的特徴を考察することで、「パブリック/プライベート」だけでは得られない新たな知見を得られることができる。 【結論】対面的相互行為において、他者のプライベートな空間にアクセスしようと思えば、参与者は必ずパブリックな空間を通って移動する必要がある。そのとき、参与者は「人間」の呈示のための準備を必要とする。しかしながら、Web会議システムはプライベートな空間をオンラインで直接つなぎ合わせることができる。それゆえに、自己呈示は「個人」の呈示に偏りを持つことになる。したがって、「個人」の呈示よりも「人間」の呈示が重要となる場面のWeb会議システム(例:オンライン授業、オンライン面接)は、特有の対処が必要とされる。ウェブカメラ・マイクのオン・オフ、バーチャル背景はその一例と言えるだろう。しかしながら、「個人」の呈示に偏りを持つというWeb会議システム特有の特徴は、オンライン子育て支援のような場では、利点として機能している。ウェブカメラが参与者だけでなくその後ろに広がる自宅の様子を捉えてしまうこと、マイクがあらゆる音声を拾ってしまうこと、こうしたハードウェア的な特徴が参与者の「個人」をどこまでも強調することで、オンライン子育て支援という場は、オンラインだからこそ、安らぎを得られる場として機能していると考えられる。

報告番号201

オンライン子育て支援の相互作用(3)――「わが子の報告」の組織化と機能
奈良教育大学 粕谷 圭佑

【目的】オンラインで行われる子育て支援では、自分の子どもの様子を他の参加者に向けて報告する局面が随所に織り込まれている。こうした活動は、いかにして子育て中の親に対する「支援」となりえているのか。本報告の目的は、この「わが子の報告」活動がどのように組織されているのかを記述することを通して、その活動に含まれる、子育てに対する緊張感を和らげる仕掛け(機能)の一端を明らかにすることにある。 【方法】本報告はエスノメソドロジーの「ワークの研究」(Garfinkel 1986) の方針に基づいた記述を行う。「ワーク」は人々の方法の実践によって、固有の活動を達成する作業のことを指す。なかでも、固有の知識やスキルを必要とする専門的な領域を対象にした研究を「ワークの研究」と呼ぶ。オンライン子育て支援はWeb会議システムを利用した親子同士のやりとりであるが、その活動は、固有の仕方で組織されている。そのため、この活動の参加者は、その固有の組織化を読み取り、自らも組織化に参画していくことが求められる。とくに「わが子の報告」では、「なにを、どのように」報告するかが一定程度定まった形で共有されている。本報告は、この固有な活動の組織化が、参加者にとっての独特の「居心地の良さ」をいかに生み出し得ているのかを、参与者による方法的な組織化(ワーク)に着目することで見通していくことを目指す。 【結果】分析の対象となるのは、オンラインでのワークショップ事業を展開する民間の子育て支援法人Aによる子育て支援プログラムの録画データである。当該プログラムにおいては、「朝のごあいさつ」や(事前に告知していた作画などの課題を子どもに行わせる)「子どもの時間」などの名称を用いて活動が区切られていく。各活動においては基本的にオーガナイザーが司会者となり、参加者に発話機会を割り当てていく。分析断片において、指名された参加者は、その場に居合わせている子どもに関する報告を行うが、その際の報告の形式の多くは、現在の状況に至るまでの背景からはじまり、親の取り組みや、子どもの反応などが時系列に沿って語られ、親の思惑どおりにならない子どもの姿がしばしば「オチ」として登場する。この「物語を語る(storytelling)」形をとった独特の報告形式が、この活動の基調になっていることが観察された。 【結論】子育て上の心配事や不安は、いつでもだれにでも語れるものではない。それだけでなく、そもそも心配事や不安として語ること自体が、親としての役割規範に動揺を与えうるものである。それに対して、オンライン子育て支援の「わが子の報告」活動で使用される物語りの形式は、子育て中の親が抱える様々な葛藤を表現する媒体となり、上述の緊張を和らげるものとなりえていると考えられる。 (文献)Garfinkel, H. ed., 1986, Ethnomethodological Studies of Work. Routledge.

報告番号202

「全部割り切ってしまうと仕事できなくなってしまうので」――障害者介助における医学的知識の参照とその困難
東京都立大学 石島 健太郎

1.問題の所在 進行性疾患である筋萎縮性側索硬化症(ALS)は、重度の身体障害の一方で多くの場合に認知機能は侵さないとされてきた。しかし近年、前頭葉型認知症や情動制止困難が症状として知られ、ALS患者に経験的にはしばしば見られてきた「怒りっぽい」「こだわりが強い」と表現される特徴に医学的説明が与えられるようになりつつある(van Es et al. 2017)。初期の医療化論に従えば、こうした患者の易怒性も、医学的説明によって免責されるかもしれない(Conrad 1975)。しかし、免責の成否は個別具体的な事例に即して異なる。また、易怒性に対する介助者の感情管理をめぐる諸論も、これを心構えとして捉えるに留まる。精神症状についてますます医学的な説明がつくようになり(Rose 2007=2014)、また多職種連携の下で医療的ケアを受けながら地域で生きる人々が増える現在、介助現場に浸潤する医療的知識がいかに参照されているのかが問われる必要がある。 2.方法 患者支援を行うNPOを経由して介助者を紹介してもらい、主にWEB通話で1時間程度、半構造化面接を行った。データは逐語的な文字起こしの上で分析された。 3.結果 医学的知識は、たしかに個々のエピソードを説明・理解する資源となっていた。一方で、そうした知識は必ずしも患者の免責や介助者の感情管理を容易にしてはいない。まず、患者の情動が本当に症状なのかどうかは、介助者には判断できない。とくに、患者にとって親しい人ほど怒られやすい・より専門職性の高い訪問看護師には怒らないといったように、患者の情動は孤立した症状ではなく、具体的な文脈のなかで観測されるため、医学的な説明は腑には落ちても確証を得られない。また、介助がその対象のニーズを汲み取りおこなうものである以上、そうした情動もまた患者が求めるもののシグナルとして理解されるため、単に医学的な症状として片付けることを介助者は忌避していた。このように、介助という営みがもとより個別の症状への対応ではなく患者の全人的な生活史や人となりを踏まえた支援を志向するものであるために、医学的知識による一貫した説明は困難になっていた。 4.結論 障害学においては、障害を社会的構築物とみなす社会モデルを基軸としつつも、心身の物質性をいかに把握するかが今日まで理論的な争点となっている。しかし、地域在宅生活のなかでの医学的知識の参照は実践上の課題でもある。しばしば医療専門職から提言される医学的知識の教示では解決しない、むしろ知識があるがゆえの困難を捉える必要がある。 文献 Conrad, P.,1975, The discovery of hyperkinesis, Social Problems, 12(1): 12–21. Rose, N., 2007, The Politics of Life Itself, Princeton University Press. (=檜垣立哉監訳,2014,『生そのものの政治学』法政大学出版局.) van Es, M. et al. 2017, “Amyotrophic Lateral Sclerosis.” The Lancet 390: 2084–98.

報告番号203

ケアの脱家族化は可能か?――障害者政策の中の家族
金沢大学 高橋 涼子

障害のある人、高齢者や子どもなど様々な支援ニーズをもつ当事者のケアを家族に依存する日本の福祉政策の下で、ケアの脱家族化はどのように可能だろうか。本報告では、障害者政策と障害のある当事者の運動および家族の運動との関係に着目し、社会学、家族社会学、障害学などにまたがる先行研究を参照して障害者政策における家族の位置づけを検討した後、ケアの脱家族化の可能性を考察する。 障害受容や介護方法の教育を通して家族を支援すべき対象として捉える社会福祉学的アプローチではなく、なぜ家族が介護者として当然視されるようになったか、家族はどのようにその期待や役割を引き受けるのか、障害のある人の親(主に母親)への聞き取り調査を通して社会学的にアプローチした要田(1999)、春日(2001)、土屋(2002)等の先駆的研究は、家族が「介護/扶養者」としての役割を政策的にも社会的にも強く規定されてきた経緯と現状を示した。また「介護/扶養者」つまり「ケアラー」としての家族は、障害者福祉の充実を、障害のある人の「代弁者・アドボケイト」として行政や社会に対して訴える主要なアクターであったが、入所型施設の建設を求めるなど、障害当事者運動から批判され対立する側面もあった。1990年代半ばから日本の障害者政策は脱施設化と地域生活支援へと方向転換してきたが、藤原(2015)は、利用できる福祉サービスが増えることで新たに親、特に母親には、サービス利用に伴う「コーディネーター」としての役割が加わり、母親のケア役割は縮小されないと指摘する。障害者家族はケア役割、ケア責任から未だ解放されていない。 2014年の障害者権利条約の批准を経てもなお、地域での暮らしの場を含む社会資源が不足しているため、入所施設がこれを補い施設入所待機者の数が退所者の数を上回ること、入所施設やグループホーム以外では家族同居が圧倒的に多いことが指摘され、家族依存を前提とした政策が地域での暮らしの場を含む社会資源不足を招き、障害のある人が誰とどこで暮らすかを選択する権利を行使できないと批判されている(日本障害フォーラム2021)。 障害者家族はその運動の中で、自分たち自身の介護・ケア負担の軽減や自己実現を公然と主張することはなかった。しかし家族依存の政策は家族自身の権利行使はもちろんのこと、障害当事者の権利行使も同時に妨げていることを確認し、家族を障害当事者とは異なる独自のニーズをもつケアラーと捉えその権利を保障することは、障害のある人の施設以外での生活、親に依存しなくてよい生活の選択肢を広げ障害当事者の権利保障と不可分であるという視点が、ケアの脱家族化の戦略に必要であることを最後に述べる。 参考文献:藤原里佐(2015)「障害児家族の困難と支援の方向性」『障害者問題研究』42(2): 250-257, 春日キスヨ(2001)「障害児問題からみた家族福祉」『介護問題の社会学』岩波書店, pp. 77-114,日本障害フォーラム(2021)「障害者権利条約 日本の総括所見用パラレルレポート」, 高橋涼子(2022)「家族からの排除/家族への排除-日本の障害者政策の課題-」金沢大学人間科学系研究紀要14: 20-37, 土屋葉(2002)『障害者家族を生きる』勁草書房, 要田洋江(1999)『障害者差別の社会学―ジェンダー・家族・国家』岩波書店)

報告番号204

介護保険制度下でケアにおける情緒的な絆にはどのように対処されているか――訪問看護を事例として
立命館大学大学院 中田 明子

【1. 目的】私的領域で情緒的な絆がある家族によってなされてきたケアは、現在は介護保険制度の創設により、対価が支払われるサービスとなっている。訪問介護の先行研究では、画一化されたケアに変容していることが指摘される一方で(山根 2014)、同一のスタッフの訪問を希望する利用者がいることも示されている(角 2021)。後者の研究は、家族ではない者によるケアであっても、ケアの受け手と担い手との間で何らかの情緒的な絆が生じていることを窺わせる。よって本報告では、医療相談の業務が含まれ会話が不可欠な点で、訪問介護よりも情緒的な絆が生じやすいと考えられる訪問看護を調査対象として、介護保険制度下では、ケアにおいて情緒的な絆が生じうるということに対して、現場でどのように対処されているか明らかにする。【2. 方法】2018年9月〜2019年6月に、訪問看護の事業所で勤務する、または勤務経験のある看護師に対して半構造化面接を実施した。本報告で分析対象とする26名は、全員女性で年齢は30代〜50代である。26名の同意を得た上で面接内容はすべてICレコーダーに録音し、逐語録を作成した後に、各質問に対する回答を傾向ごとに分けて分析している。【3. 結果】訪問する看護師の限定を求める利用者は存在し、その理由は、利用者が身体的・精神的な脆弱性を抱えていて、画一的なケアに耐えられない状態にあるからであった。調査対象者の所属する事業所では、様々な働きかけをしながらも、スタッフを限定する求めに応じていた。応じざるを得ない根本的な理由は、訪問看護のサービスがなければ利用者の生活は継続できないという利用者の「退出可能性」の不在を、看護師が認識することで、看護師の側も「退出可能性」を喪失するからで、また他の看護師が訪問しているので自分はその利用者に訪問しないで済むというように、看護師は互いに「負い目」を感じ合う関係にあるため、「退出可能性」を喪失するからである。このような利用者の訪問日には特定のスタッフは休まないようにしたり、スタッフの固定を求める少数の利用者を複数の看護師で分散して担当したり、訪問の際には「深層演技」の対処方法を採ったり、という諸々の工夫がされていた。さらに調査対象者の多くは、スタッフの固定をやむを得ないことと否定的に捉えていたが、一部の看護師は、自分のみが特定の利用者に対して訪問できることに優越感を感じるという共依存的な関係を、そのような利用者との間で築いていた。【4. 結論】本報告の調査結果は、何らかの点で脆弱性を抱えた人は、ケアにおいて情緒的な絆を求めがちであり、事業所側が様々な工夫によってそれに応じることで、そのような状態の人々が介護保険制度のサービスに包摂されていることを示している。しかし以上の訪問看護の実態は危ういバランスで成立しており、訪問看護に関して事業所を大規模化する施策が進められているため、今後については看護師の労働強化か、あるいは脆弱性を強く抱えた人の介護保険制度のサービスからの排除が生じるリスクを抱えている。【参考文献】堀田義太郎, 2008,「ケアと市場」『現代思想』36(3): 192-210. 角能, 2021,『ケアをデザインする――準市場自体の自治体・サービス主体・家族』ミネルヴァ書房. 山根純佳, 2014,「介護保険下におけるホームヘルプ労働の変化――「業務化」する個別ケアの現場」『日本労働社会学会年報』(25): 3-21.

報告番号205

対人支援における熟議アプローチの実践
国立保健医療科学院 松繁 卓哉

【1.目的】 近年、健康や疾病に対する個人の考え方がますます多様化し、また、感染症拡大などによる健康不安や生活困窮がますます増大する中で、支援対象者の多様なニーズを見極めることは困難になってきている。健康問題に加え、生活困窮や引きこもりなどの生きづらさを含めた複合的課題を抱える人々にとって、比較的短い期間内に直線的に問題特定から解決へと向かっていく、いわゆる問題解決型アプローチ(problem-based approach)ではうまく対処できないことが問題となっている。そのような状況の中、これまでは主に政治や科学技術に関する人々の合意形成をはかる目的で用いられてきた熟議アプローチ(Deliberative Approach)を、対人支援の臨床場面で活用しようとする試みが現れてきている。熟議アプローチの特徴は、科学者と市民、医療従事者と患者のように、専門性・立場・視点・思考様式の異なる者が、一定の手法に則って対話をおこなっていくことで相互理解を深められるところにある。その一方で、保健医療領域における熟議アプローチの具体的な実践方法については未だ確立されておらず、保健・医療・福祉の現場で活用されるためには、手法の確立と、その評価が必要とされる。 本研究では、保健・医療・福祉の従事者が支援対象者のニーズや課題等を適切に理解するための手法として熟議アプローチの実践方法を開発する。 【2.方法】 アクションリサーチのアプローチを用いて、保健・医療・福祉の専門職従事者の協力を得て、実践と情報収集・考察を循環的に繰り返しながら、熟議を対人支援に応用するための知見を導き出していく。具体的には、これまでの先行研究で得られている知見を基に、熟議を実際に体験する試行プログラムを作成し、保健・医療・福祉の従事者らの協力のもと、プログラムの試行・振り返り・考察を重ね、精査していく。プログラムの試行を重ねながら、その評価を行う。 【3.結果】 現在は、情報の収集と分析を継続中である。熟議プログラムでは、感染症対策等の健康不安や、福祉的課題を持つ人々のセルフケア、治りにくい疾患を有する人々への療養支援の他、市民と医療従事者のコミュニケーションにおける課題等の具体的なテーマを設定し、参加者らがそれぞれの異なる考え方を形成している背景情報(e.g. 過去の経験、生育環境、体質、信仰、その他)について理解を深め、相互理解をはかるためのステップを試行的に構成している。 【4.結論】 テーマに関して、熟議の参加者自身が自らの考えを述べることに加え、「なぜ自分がそのような意見を持つに至ったのか」を自ら再考する機会を経て、自己理解が促される契機となっている。また、そうした作業を複数人が共に実践し、共有することで、当該テーマに関する人々の多様な見解の存在を認識し、その中に自身を位置づける機会を設けることが、相互理解を促すための有意義な実践となっている。

報告番号206

都市部におけるつながりと生活課題の実態
大阪公立大学 野村 恭代

【1.目的】  本研究の目的は、①都市部における生活実態及び地域における支え合い活動の程度を明らかにすること、②生活課題及びニーズの実際を把握すること、③潜在化した地域における生活課題を顕在化させること、④顕在化した生活課題を住民と専門職の協働により解決すること、の4点である。本報告では、主に①、②に焦点をあて都市部に居住する住民相互のつながりと生活課題の実態について報告を行う。 【2.方法】  本調査は、A地区安全・安心のまちづくり実行委員会(以下、実行委員会)を立ち上げ実施した。この実行委員会には、地域に関係するさまざまな団体、専門職、住民等が参画している。調査はA地区に居住するすべての住民を対象に実施した。自記式回答用紙を個別訪問により配布し、回収場所を周知したうえで、実行委員会メンバーによる対面での回収を行った。調査票の内容は、回答者の基本属性に加え、世帯の生活上の困りごと、自治会や町全体における近隣とのつきあいの程度を尋ねる項目から構成されている。なお回収の際には、調査項目の内容に沿って各世帯の生活課題を中心に聞き取りを行った。調査時期は、2021年4月~7月である。 【3.倫理的配慮】  本研究調査の回答結果については、個人名や施設・機関名、特定の地域情報が明らかにならないように配慮し、プライバシー保護のため匿名で調査を実施した。回収データについては、統計的に処理を行い、本研究の目的にのみ使用した。また、調査の実施にあたっては、研究内容及び個人情報等の取り扱いに関する説明文書を同封し、同意の得られた調査対象者にのみ回答を依頼した。なお、本調査は、大阪市立大学(現大阪公立大学)大学院生活科学研究科内に設置する研究倫理委員会の承認を得て実施した。 【4.結果】  調査票配布数は502票、回収238票(有効回答数同数票)、有効回収率は47.4%であった。回答の得られた238世帯のうち、8割にあたる192世帯が何らかの生活課題を抱えていることが明らかになった。コロナ禍であることも影響し、「病気や健康のこと」「将来の生活について」「収入・生活費のこと」を困りごととして挙げる世帯が多く見られた。また、子育てに関する困りごとについては、対象世帯の9割の世帯に困りごとがあることが明らかになった。困りごとの主なものは、「子どもの学力」「養育にかかる費用」「仕事と子育ての両立」「食事や食育」「子どもとのかかわり方」である。 【5.結論】  「居住年数」と「近隣との付き合いの程度」に関しては、相関関係はあまり見られない。居住年数に関わらず「ほとんどつきあいがない」層が1割程度いることから、居住年数が長いからといって近隣付き合いの程度が深くなるとは限らないことがわかる。また、生活課題を抱えながら地域とのつながりがなく、近隣に頼れる親族もいない層が一定数いることも明らかになった。このような世帯に対しては、地域のなかでつながりをつくることによって安心して生活することのできる体制を構築することが重要である。生活課題の背景や要因は多様で深刻であるが、いずれも早期のアプローチが必要であり、また社会的居場所づくりや就労の機会等、地域支援に加え支援のための環境整備が不可欠である。

報告番号207

韓国において性売買を語ること――性売買経験当事者の発信に注目して
岩手大学 古橋 綾

【1.目的】本研究は韓国の性売買経験当事者たちの発信を分析することにより、当事者たちの語りによって性売買が内包する搾取の構造がいかに明らかになってきたのかを考察することを目的としている。韓国では2004年に性売買禁止法が成立して以来、性売買で被害にあった女性たちを支援する仕組みが構築されてきた。性売買「被害者」として支援をうけてきた当事者たちは、自身を性売買「経験当事者」と位置づけ、自らの経験を語り始めた。2010年代からは、それらの語りをSNSやイベント等で発信し始め、2021年には書籍としても刊行している。性的な被害をうけた人たちがその経験を語ることは容易ではないが、彼女たちにはなぜそれが可能だったのだろうか。本研究では、第一に、経験当事者たちが語り始めた契機はなんだったのか、第二に、経験当事者たちはどのような経験をいかに語ってきたのか、第三に、経験当事者たちの語りによりいかなる変化がもたらされたか、という3つの問いに答えていくことにより、彼女らの語りによって性売買が内包する搾取の構造がいかに明らかになってきたのか浮かび上がらせ、「当事者の語り」が持つ意義についての議論を発展させることを目指す。 【2.方法】性売買経験当事者に関わる記録――経験当事者自らが執筆した書籍・手記集、新聞への寄稿、SNSでの発信等――を研究対象とし、その発信が行われた時期の社会状況を踏まえて分析を行う。報告者は、2016年以来、韓国の反性売買運動団体及び経験当事者のグループが開催するイベントや展示会、デモなどで参与観察を行ってきた。その過程で、経験当事者たちが声を震わせながら語る姿を何度も目の当たりにしてきた。本研究において個別のインタビューを行わなかったのは、新たに語ってもらうことが経験当事者に対する別の搾取になり得ることに留意し、彼女らがすでに行っている発信を通じ、本研究の目的を十分に達成できると判断したためである。 【3.結果】経験当事者たちは自身の経験を語ることに最初から積極的ではなかったことが分かった。互いの経験を少しずつ話していくことにより、本人たちも予想していなかった連帯感が生まれ、安心して話せる空間を確保していった。しかし、経験当事者同士が語り合った内容を外部に向けて発信するためには数年の期間が必要であった。経験当事者たちは自身の話を既存の常識の枠組みで理解しようとする行為を拒否し、その主張を絶対視することも拒否した。自分たちが経験したことを多様なメンバーが淡々と語ることで、その搾取の構造を示していった。2016年に起こった江南駅殺人事件以降、韓国社会でミソジニーや性暴力被害が注目されるようになると、性売買の搾取の構造にも関心が寄せられるようになった。 【4.結論】韓国の性売買経験当事者たちの語りとその受容の過程は、当事者が語ること及びそれを聞くことが持つ意味の再解釈の必要性を迫る。声をあげにくい問題を抱えさせられた当事者の語りに耳を傾けるというような単純な理解を超えて、誰がどう聞くのか、当事者の語りはどう位置づけられるべきかという問いへと歩を進めるものである。

報告番号208

女性非触法ペドファイルの語り
愛知大学 湯川 やよい

1.研究の背景と目的 本報告は、「非触法ペドファイルNon-offending pedophiles」(子どもへの性加害を行ったことがない小児性愛者)のうち、特に女性当事者の経験に焦点をあて、彼女(たち)がそのセクシュアリティをどのように認識し社会の中で自身を位置づけているかを、当事者の語りから考察する。 ペドファイルの研究は、医学、心理学などで数多く蓄積されてきた。だが、そのほとんどは子どもへの性加害で逮捕されたり加害者臨床の対象となった人々(Child Sex Offender:CSO)のみを対象としている点で、大きな偏りがある。近年の海外研究では既存研究のこうした偏りが、「CSO=ペドファイル」という巷の誤謬を増幅させ非触法者の社会的排除(若年当事者の自死等)につながると共に、子どもへの性加害にかんする正しい理解(CSOには小児性愛でない者も多いこと等)に照らした適切な被害防止を妨げることも指摘されている。一方、ジェンダー・セクシュアリティ研究では、M. フーコーなど、現代社会で悪魔化されたペドファイル表象を批判的に検討する対抗言説群があるが、それらの探究もまた、非触法者と触法者を原理的に区別しない分析に留まっている。 以上を踏まえ報告者は、「触法行為によってのみ社会的に可視化される」という非触法当事者に特有のジレンマに注目し、不可視化された非触法当事者の意味世界に接近することを目指している。本報告では、研究蓄積の極めて浅い非触法者の中でもとりわけ議論が乏しく、ときに「実在しない」とさえ言われてきた非触法女性ペドファイルの経験に注目する。 2. データと調査概要  報告者がインタビューを行ってきた非触法当事者のうち、唯一の女性対象者(20代・女児への指向)への調査結果を用いる(2021年~2022年、計3回)。なお、類似調査の動向を踏まえれば1例でも検討の意義はあると考える。たとえば海外の先行調査(非触法女性6例の心理学研究)では、チャットなど顔が見えない調査環境で女性を騙る男性回答者がいた可能性を排除できていないことが、その限界とされている。それに対して、本報告では対象者女性と一定のラポールを形成し、本名、現住所等を確認の上で顔を見ながらの聞き取り(ビデオ通話)を実施できた。なお、考察では、男性対象者の事例も比較対象として補足的に参照する。 3.考察 考察の結果、以下が明らかになった。まず、学齢期に周囲と異なる指向・嗜好に気づくプロセスや「結婚し子をもつ」標準化されたライフコースからの逸脱に対する不安などは、他の非触法男性当事者のケース、およびペドファイルではない若年性的少数者の研究が示す知見と共通する点も多い。一方で、ペドファイルを男性性の病とみなす言説と女性ジェンダーとの間での混乱を伴う自己認識過程は、この事例に特徴的であった。また、彼女の語りでは、社会的排除への警戒、自分が触法行為に及ぶ万一の可能性に対する漠然とした恐怖と苦悩、さらに自身の苦悩が触法行為の正当化に利用されることへの警戒が複雑に絡み合っている。 「どこかにいるかもしれない自分と同じ女性たち」に向け手探りで自身を言語化=現実化しようとする彼女の語りは、性暴力のない社会の実現とペドファイルの脱スティグマ化を同じベクトルを向く課題として位置づけ直す対抗言説の一例といえる。

報告番号209

軍事性暴力をめぐるポジションについての状況分析――沖縄の米軍基地引き取り運動に携わる女性たちへの対話的インタビューをとおして
九州大学 里村 和歌子

【1.目的】 沖縄県が日本に復帰して50年。0.6%の国土面積の沖縄県に70%もの米軍専用施設が偏在する状況がつづいている。そんななか、沖縄の米軍基地の過重負担は「本土」による植民地主義の結果であるがゆえに「本土」に基地を引き取るべきだという考えとそれに基づく社会運動が全国に広がっている。一方でこの運動に対し、軍隊とはそもそも性暴力と切っては切り離せないものであり、「痛み」は引き取れないという批判もなされてきた。そこで本報告では、基地引き取り運動に実際に携わる女性たちに焦点を当て、なぜ彼女たちは基地を引き取ろうとするのか、軍事性暴力をめぐる批判をどうとらえているのか、その理路を対話的インタビューによって明らかにする。 【2.方法】 調査対象は大阪、埼玉、京都で基地引き取り運動に携わる6名の女性たちである。運動への距離はさまざまであり、中心的に参加しているメンバーから、ときどきイベントを観覧する程度の女性も含む。方法は半構造化インタビュー調査を1から2時間ほど行った。分析方法としては、「状況分析 Conjunctural Analyses」(Hall 2019)を用いる。歴史的・時代的状況を留意しつつ日常生活を構築する権力関係を分析し、その利害関係のフレームを対話的に浮かび上がらせる分析方法である。行為者たちの「声」をベースにポストコロニアリズムとジェンダーというふたつのポジションがぶつかりつつも節合する地点を読み取っていく点に特色がある。 【3.結果】 女性たちへの対話的インタビュー調査をとおし、彼女たちの行動原理が歴史的・時代的状況との相互作用によって醸成されていることがわかった。つまり、ひとりの人生においてさまざまな差別問題と向き合ってきた経緯が、メディアや経験をとおし沖縄の状況に触れた際、それは差別問題であり、自身を「加害者」のポジションにいると認識させる理路があることがわかった。つまり、軍事性暴力という「痛み」については認識しつつもそれを沖縄という一地域の女性たちに与えているのは自分たちこそであるという視点に立つため、「『痛み』は引き取れない」という批判は意表を突くものであり、かつ沖縄の女性たちに対する差別的言説であるととらえていることがわかった。 【4.結論】 基地引き取り運動に携わる女性たちへの対話的インタビューをとおし明らかになったのは、自分たちを沖縄と「本土」というポストコロニアリズムのポジションにおいて加害側にいると認識していること、その加害状況を変えるには、まず沖縄の基地の過重負担を解消することが目指されるべきであるととらえられていることであった。そこでは一見ジェンダー関係のポジション、つまり軍事性暴力をめぐる被害と加害にかんする葛藤が不在のように見えるのであるが、対話的インタビューをとおしおぼろげながらつかんだのは女性間の差異を温存しながらの女性同士の連帯は可能かという新たな問いである。本報告は行為者たちへの質的調査をとおして境界空間にこそ可能性と課題を見出す点で意義があると考えている。 参考文献:Hall, Stuart., 2019, “ESSENTIAL ESSAYS VOL.1”, Duke University Press.

報告番号210

奥むめおの婦人運動における社会的ネットワーク形成
富山県立大学 濱 貴子

【1.目的】  本報告の目的は,戦前から長きにわたって生活に根ざした婦人運動・消費者運動に活躍した奥むめおに注目し,彼女を中心とした運動の成長と持続を支えた社会的ネットワーク形成を明らかにすることである. 【2.方法】  本報告で分析に用いた主な資料は,主婦連合会機関紙『主婦連たより』(1号〔1948(昭和23)年12月〕-478号〔1989(平成元)年6月:奥むめおの主婦連名誉会長就任が報告された号〕)である.各号を通読し,毎月の「運動日誌」に記録された活動について,年月日,活動内容,活動主体(名前・役職),活動の場,連携団体・連携者,参加人数をデータ化した.また,『主婦連たより』や周年記念誌などから,活動内容の詳細を補足した.以上のデータを用いて,時期を4つに区分し(第1期:結成〔1948〕-58年度,第2期:1959-68年度,第3期:1969-78年度,第4期:1979-88年度),①日常的活動,②連携的活動について,幅・深さ・持続に注目し質的・量的両面から奥むめおが率いた主婦連合会の社会的ネットワークの分析を行った. 【3.結果】  第1期には,草創期に主婦を対象としたイベントや,講演会・座談会を開催するなかで首都圏の地域婦人会を中心に全国の婦人会を主婦連合会に包摂し,組織作りを進めた.主婦連結成後は,部会活動を継続的に実施し,成果を奥の参議院議員という立場を通じて国会・政府に要望・陳情していった.そのなかで,主婦連メンバーも政府審議会委員に就いていった.また,1956年には全国消費者団体連絡会の結成に参加し,全国の消費者団体との連携を深めていった.  第2期には,1956年に多方面からの寄付により完成した主婦会館を場とする部会活動が活発化するとともに,メンバーの政府や都の審議会・委員会・懇談会委員としての活動も活発化し,部会での調査・研究をもとに国会・政府や都に要望を行うスタイルが確立されていった.加えて,講演活動も活発化し,消費者運動の方法や成果・動向を主婦連構成団体や全国各地の婦人団体へ伝えていった.  第3期には,暮らしや食にかかわる問題の複雑化とともに,調査・研究活動の高度化・専門化が進んだ.消費者活動基金をもとに調査・研究を一層活発・緻密に行い,成果パンフレットを公刊していく一方で,各地の婦人団体・婦人会への講演活動は減少した.他方で,ジュース裁判・ヤミカルテル灯油裁判などの訴訟の展開により法学者や弁護士などとの連携が密接化していった.また,複数の規模の大きな消費者団体・婦人団体・市民団体と連携した運動や政府や政党・企業への申入が増加した.  第4期には,国際的な課題への取り組みが進んだ.国連婦人の10年を機に女性の地位向上をめざし,婦人団体・消費者団体などに加え労組婦人部との連携も進んだ.また,反核・軍縮といった平和運動の盛り上がりのなかで,被爆者団体・青年団体・労組など幅広い団体との連携も進んだ.さらに,IOCUなど国際的な連携も活発化していった. 【4.考察】  戦後から高度成長期,安定成長期へと時代が移り変わるなかで,主婦連合会の運動の内容も変質・高度化・複雑化していき,他団体との連携も多様化していったが,「いのちとくらしを守り,人間を大切にする」という奥むめおが一貫して掲げ続けた理念は,様々な団体との連携を可能にし,運動の深化・拡大を促進させていった.

報告番号211

〈社会問題の教育化〉のアナロジーとしての脱魔術化――ウェーバー学問論を手がかりに
京都大学 倉石 一郎

本報告は〈社会問題の教育化educationalization of social problems〉現象をめぐる探求を深めるにあたり『職業としての学問』を中心とするM・ウェーバーの学問論が有用な基点となることを主張するものである。社会問題の教育化とは、ある顕在化した社会問題に対して直接的解決措置をとるかわりに、学校システムへの小幅な投資と共に教育に解決への期待と責任が押し付けられる現象を指す。たとえば環境問題において、直接的解決措置の一つが企業活動に対する環境規制の強化だとすれば、その代わりに「環境教育」プログラムを多くの学校等で実施し、それを通して環境配慮型行動を人びとに促すことで対処しようとするのが社会問題(環境問題)の教育化である。「教育化」による問題解決の実効性は控えめに言っても不透明で、その選択に疑問符が付く場合も多い。にもかかわらず教育化という方途が繰り返し選択される現象をどう見るかが焦点となる。報告者は〈社会問題の教育化〉について、問題の解決という主意主義的領域に、教育や学知という主知主義の産物が越境ないし動員されるクロスオーバー事象として把握する視座に立つ。この認識を補強し深める手がかりをウェーバーの議論に求める。  導きの糸となるのが脱魔術化Entzauberung概念である。周知のように『プロ倫』等一連の宗教社会学においてウェーバーは、魂の救済が呪術によらず生活の合理化を通じてなされるに至った事象を脱魔術化と呼んだ。一方で『職業としての学問』での脱魔術化への言及では、主知化・合理化は人間が置かれた生活条件に関する全般的知識の増大を意味せず、例えば現代人は路面電車の駆動原理を知らずただそれが呪術によらないと固く信じているだけだとされている。二つの脱魔術化は一見別のようだが、主意主義世界と主知主義世界との間に深く穿たれた断裂、しかしそこに宿命的に架橋せずにおれない近代人の事態を把握した点が共通する。 こう考えると『職業としての学問』の各所の記述も、主意主義と主知主義の狭間での煩悶の主題化と理解できる。大学での講義に「体験」を求め、教師に「指導者」の幻影を追うドイツの学生たちを戒める箇所があるが、この学生たちは徹底して合理化・主知化した学知という回路を通して魂の救済を追求している点でウェーバー宗教社会学でいう「脱魔術化」過程の渦中にいる。そして「教壇禁欲」を説いた周知の箇所で彼は、生の現場で切実に求められる意味や価値の判断根拠を、大学教師は学生に与えられないと喝破した。学生たちの価値判断を代行することは不可能だが、かと言って自身が価値判断と無縁に生きることもできず、その点で緊張感をもって自らの仕事を見直せと説いた。  主知主義世界への安住は、ウェーバーが皮肉ったように時に精神世界の貧困化を伴う。一方で解決を要する問題への直面によって主意主義世界へ引きずりだされた人間は、問題解決を純技術的・没価値的にはなしえず、朧げな学知を手に何らかの価値判断を下していくほかない。脱魔術化概念は、〈社会問題の教育化〉が含意するこの事態を的確に写し取ったアナロジーなのである。  またここから一歩進めて、社会問題の存立にあたり教育化を不可欠の契機とみなす議論も考えられる。いま、個々の価値判断の苗床となる主体の精神・内面の立ち上がりを教育化とみなすとすれば、狭義の教育との相関を問うことが次の課題となる。

報告番号212

Social Capital and Self-efficacy of Nursing Students :Proposing Sociological Contribution to Interdisciplinary Research
沖縄県立看護大学 山口 賢一

1. Research Objective This study aims to explicate how social capital contributes to maintenance of self-efficacy among nursing students. Self-efficacy “refers to beliefs in one’s capabilities to organize and execute the courses of action required to manage prospective situations” (Bandura, 1995, p.2). Proceeding studies illustrate that nursing students have relatively lower self-efficacy scores than the general youth population. Takahata et al. (2015) argue that when a nursing student with the low level of self-efficacy experiences difficulties, he/she can develop anxiety. In this study, we focus on social capital, that is, “the aggregate of the actual or potential resources which are linked to possession of a durable network” (Bourdieu, 1986, p. 51) as a factor that can mitigate nursing students’ anxieties and maintain their self-efficacy. 2. Methods In 2020, our interdisciplinary research team – consists of instructors of nursing, medicine and sociology – conducted survey research at “N” nursing college in “A” prefecture to investigate factors that are associated with self-efficacy scores among nursing students. The research results show that difficulties regarding nursing practicum and specialized subjects are associated with lower self-efficacy scores. Nevertheless, this survey research did not clearly show variables and mechanisms that contribute to maintenance of self-efficacy. In 2021, as subsequent research, we implemented interviews with nursing students at the same college. We randomly selected 50 students, and 12 students agreed to participate in the interviews (response rate: 24%). Of the 12 participants, we had 2 first year students, 4 second year students, 3 third year students and 3 fourth year students. 3. Results and Discussion Initially, our research question was concerned with students’ motivation and its contribution to their sense of self-efficacy. Nevertheless, in the course of interviews, we noticed that the participants with good relationships with classmates, instructors, and/or family are able to cope with challenging situations and maintain self-efficacy. In particular, students with good networks with classmates – or social capital – answered that they were able to overcome anxieties by receiving encouragements in times of difficulties. At this conference presentation, we will introduce some examples that the participants could build positive social relationships with classmates. We also examine types of difficulties that the participants were able to overcome by employing their social capital at the field of education. 4. Conclusion In the course of our interdisciplinary project, we developed and broadened our research questions and methods. Sociological concepts – reference to social capital – could offer fruitful insights in this research project. References Bandura, Albert. 1995. Self-efficacy in Changing Societies. Cambridge: Cambridge University Press. Bourdieu, Pierre. 1986. “The Forms of Capital.” Handbook of Theory of Research for the Sociology of Education, edited by John Richardson, 46-58. Westport, CT: Greenwood Press. Takahata et al. 2015. “An Influence in Students for Nurses of Occupational Identity and the special quality-like Self Efficacy: Among Different Age Groups of Nursing Students.” Bulletin of Faculty of Nursing of Chukyo Gakuin University, 5(1): 27-39.

報告番号213

マンガからみる養護教諭へのまなざし
三育学院大学大学院 篠原 清夫

【1.目的】 T.パーソンズ(1964)は社会機能を考える上で手段的(instrumental)と表出的(expressive)役割があることを示しているが、養護教諭の成立期から考えるとその役割は【手段的役割】→【手段的役割+表出的役割】→新型コロナウィルス禍への対応による【手段的役割の増加+表出的役割】という拡大が起こっている。このような状況下の養護教諭に対して人々はどのようなまなざしを向けているのか。その手掛かりとしてマンガ作品から示唆を得る。戦後マンガは社会的観点・論点を発信する存在となり、人間や社会を観察・表現する力量の点で文学を凌駕しだしたと評されている(澤村2020)。そこで本研究は日本特有の職種である養護教諭の役割やジェンダー・セクシュアリティについてマンガ作品から抽出し、養護教諭へのまなざしについて分析することを試みる。 【2.方法】 D.W.プラース(1980)は日本人の成熟に関して社会学的分析を進める際に「人物や事件を写実的に描いたものであるかぎり、(中略)小説を事例記録と同列に扱ってはいけないという理由はない」とし小説を一種の事例として扱った。「レトリックは、社会の多くの人びとにとって共通の意味をもつシンボルやイメージによって伝達されなければ作用しえない。私たちは、(中略)『仮構の』物語を通しても学ぶことができる」との指摘から、マンガ作品にも人々のまなざしが照射されていると考え分析を試みることにした。素材は養護教諭がメインに描かれている2015年以降の紙媒体マンガ作品とした。2015年以降としたのは中央教育審議会答申(2015)でチームとしての学校において養護教諭の重要性が示されたためである。 【3.結果】 実際の男性養護教諭は現在も極めて少数(0.2%)であるにもかかわらず、複数のマンガで扱われていた。その中で男性養護教諭は当初児童生徒や一般教師から違和感を持たれ、養護教諭は女性であるのが普通というジェンダー観が示されていたが、次第に受け入れられていく様子が描かれていた。女性養護教諭がメインの作品は年齢が若く未婚で、生徒のセクシュアリティの対象と見なされていることが多かった。養護教諭の役割として保健室内での心理的相談、身体的相談活動が多く描写されている一方、救急処置や救護の具体的描写は乏しく、学校保健計画・保健室経営・保健組織活動が取り上げられることはまれであり、手段的役割が扱われることは少なかった。また学校内で養護教諭と一般教師が相対する場面が描かれた作品はあるが、教師全般との関係性はあまり描写されていなかった。 【4.結論】 マンガにおいても養護教諭は女性が普通だとするまなざしがあり、相談相手としての表出的役割が多く求められていた。養護教諭に多様な役割が期待される中、チームとしての働きが求められているが、その様子が描かれることは少なかった。 【文献】澤村修治,2020,『日本マンガ全史』平凡社. Parsons, Talcott,1964,Social Structure and Personality, Free Press.(=2011,武田良三監訳『社会構造とパーソナリティ(新版)』新泉社.) Plath, David W.,1980,Long Engagement: Maturity in Modern Japan, Stanford University Press.(=1985,井上俊・杉野目康子訳,『日本人の生き方』岩波書店.) 【附記】本研究は日本学術振興会(JSPS)科学研究費補助金(20K02566代表者:篠原清夫)の助成を受けた研究成果の一部である。

報告番号214

移民的背景のある子どもの学力に関する分析
京都大学大学院 中原 慧

【1.目的】本報告は、移民的背景のある児童の学力について、小学校入学時点と小学校4年生時点の学力の関係を、日本人児童との比較を通じ、分析を行った結果を報告するものである。また、分析に際しては、移民第一世代と移民第二世代を区別し、分析することで、よりミクロな関係性の把握を試みるものである。ニューカマーと呼ばれる人々の増加に伴い、彼らの子どもたちは日本の学校に通っている。こうした状況において、ニューカマーの子どもを含む移民的背景のある児童(本報告では、両親のうち最低限いずれかが外国出生のもの)の教育に関して、彼らが小学校や中学校において学習をする上で困難に直面していることが繰り返し指摘されてきた。困難を生む要因の一つには、就学時における準備不足が指摘されている。保育園や幼稚園での言語的な支援が、現状、少ないことや家庭内の文字環境など、移民的背景のある子どもが、リテラシーを培っていくことは困難であり、すでにそうした面での移民的背景のある子どもたちの困難さが明らかにされつつある。一方で、算数力など、他の面での就学時点での学力の分析は少ない。また、これら就学時点の学力と入学後の学力の関係を扱う研究も限定的なのが、当該分野の現状である。本報告は、移民的背景のある児童と日本人児童の間の学力格差を媒介する要因として、就学時点でのリテラシー能力や算数に関する能力の影響を分析する。【2.方法】分析には、TIMSSの2015年と2019年の小学校4年生のデータを結合したものを用い、目的変数には、算数と理科の推定値(Plausible Values)を用いる。就学時点でのリテラシーや算数力は、保護者が回顧的に回答した、就学時点で、子どもができたリテラシーや算数に関する項目から合成された変数を用いる。移民的背景の区別には、親の回答を基に、両親のうち最低限いずれかが外国出生の児童を、移民的背景のある児童としている。また、そうした児童内部で、日本生まれの者を移民第二世代とし、外国出生の者を移民第一世代としている。TIMSSが階層的な抽出方法であることを考慮し、マルチレベルモデルを使用する。【3.結果】第一に、移民第一世代と日本人児童の間には統計的に有意な差があるが、移民第二世代については推定されなかった。第二に、就学時点でのリテラシーや算数力は小学校4年生時の学力に正の効果があり、これらを統制することで、移民第一世代と日本人児童の間の学力格差は縮小した。第三に、移民世代と就学時点でのリテラシーや算数力の間には、統計的に有意な正の交互作用がある。【4.結論】分析結果から、小学校4年生時点の移民第一世代の子どもの学力上の困難の一部は、就学時点でのリテラシーや算数力の低さにあることが示された。また、これらの効果が、日本人の子どもよりも大きいことは、逆に言えば、就学時点でいかに準備ができているのかが、移民的背景のある子どもにとっては、より重要な要素になっていること示唆している。

報告番号215

不利のなかで育つ子ども・若者支援における支援者の形成/変容
一橋大学大学院 山野上 麻衣

1.目的  子どもの貧困対策をめぐる議論に顕著なように、貧困家庭など不利のなかで育つ子ども・若者に、学習支援の場を提供することで教育機会の平等を確保し、正規雇用に就かせることが不平等の「解決」であるとの風潮が強い。メリトクラシーが生存保障を切り崩すという懸念は、教育社会学においては教育と生存保障の境界問題として議論されてきた。そこでは、教育とは切り離して十全な再分配を行うことが解決策として提示される。しかしながら、現にある不平等が人びとに受容(あるいは受忍)されているのは、メリトクラシーの規範が内面化されているからである。そのように考えたときに、再分配を志向する社会を立ち上げるためには、メリトクラシーの規範を緩める契機を探る必要があるということになる。本報告は、不利な子ども・若者に〈教育〉的な支援を行う人びとが(日本の)メリトクラシーをどのように相対化していくのかを、支援者の調査結果から分析していく。 2.方法  具体的には、不利のなかで育つ子ども・若者の例として、日本で育つ移民二世の子ども・若者に着目する。そのような子ども・若者を対象とし、学校教育制度の外で〈教育〉的な支援活動(学習支援や就学・進学支援、日本語教育など)を創始し、現在に至るまで続けてきた4名の支援者・支援活動の場の調査を実施した。これらの支援者は、それぞれに〈教育〉的な支援を軸とした活動を展開しながらも、それとはベクトルの異なる無条件の生存保障志向の実践(食料支援・子ども食堂、居場所づくり、自由な表現活動、在留資格案件への対応、福祉制度の利用支援等)も行っている。なぜこのような異なるベクトルの実践が混在するのか、考えられる原因として、大きく分ければ①支援者のもともとの関心や志向性、②支援のなかでの気づきの2つの可能性があげられるだろう。これらを検討するために、そもそもどのような問題意識で活動を始めることになったのか、どのようなきっかけで、なぜ生存保障志向の実践を取り入れるのか、支援者の生活史と重ね合わせる形でインタビュー調査を実施した。同時に、活動において〈教育〉・生存保障の論理がどのように現れるかを検討するために、参与観察も行っている。 3.結果  「ほうっておけない」「やめるわけにはいかない」ために、活動は長く続けられているが、支援開始時における強い問題認識や理念が活動を貫いているというわけではない。支援活動が始まったのは、それぞれの支援者の生活史における種々の状況と社会的変化が重なり合うなかの、ある種の偶発性によるものであった。それゆえに状況に応じて子ども・若者(あるいはその家族)のニーズに柔軟に対応することができているとも言える。報告においては、支援のなかでの気づきがどのように生じ、それがメリトクラシーの規範をいかに揺るがすのか/揺るがさないのかを議論していきたい。

報告番号216

学習の達成に志向した教示作業のデザイン――エスノメソドロジーの視点から
北海学園大学 五十嵐 素子

1 目的  本発表では、小学校の授業実践を検討し、そこで教師と生徒のやりとりが進行する過程で、生徒がある行為を学習できるようになることに強く志向した形で、その行為の教示作業がなされていく現象に着目する。エスノメソドロジー・会話分析の先行研究の整理に基づき、上記の現象を考察することで、これまで明確に論じられてこなかった「学習の達成」に志向した教示作業の組織化のデザインの特徴を再確認し、今後の教育実践研究に生かすことを目指すものである。 2 方法  本研究では、まずエスノメソドロジー・会話分析の研究群において、学校教育にとどまらず、教示作業の相互行為のありかたについての研究の事例を検討する。そこから「学習の達成」に志向した教示作業を含む先行研究の事例を検討しこの現象を検討するにあたっての考察の方針を定める。さらに、その方針のもとで、学校教育等の教育実践の事例を用いて、学習の達成に志向した教示作業のデザインの特徴がどのようなものであるのかという点を具体的に検討する。 3 結論と結果  教示作業の相互行為のあり方に関わる先行研究を概観すると、その考察の焦点となりうる事象は二つに大別できた。一つは、①教示作業の組織化を支える行為規範の利用が、学習者の参加のありかたや学習過程にどのように寄与しているのかについて(Mehan1975,Heap1985,1988, Lerner 1995,西阪2008,Zemel&Koschmann2009, Lindwall & Ekstrom 2012)、二つ目は、②教示対象がいかに知覚の焦点として構造化されるのか、またそれがなされるにあたっての環境上の資源の利用のされ方について(Goodwin1994, Lynch&Macbeth1998, 西阪2008, Lindwall & Ekstrom 2012、五十嵐2016 )である。 特に②の研究群の中の西阪(2008)と五十嵐(2016)は、学習の達成に志向した教示作業を事例に含む研究であり、西阪(2008)で示された知見においては、教師による教示作業で示された人工物(道具)や身体の構造化が学習者に利用可能なものとなっていたことが示唆されていた。また五十嵐(2016)では上記に加えて、学習すべき行為の基準とそれを可視化する測定のシステム(Sacks1988)が教示作業の中で示され、それが学習者に利用されていることが示されていた。  これらを踏まえると、学習の達成に志向した教示作業の組織化のあり方は、①の視点から教示作業の過程を明らかにするだけでなく、教示作業のなかでいかにある道具や身体の組織化がなされているのか、また学習すべき行為の基準とそれを支える測定のシステムがいかに導入されているのかという2点に着目しつつ、またこれらがいかに学習者に利用可能なものとなりえているのかについて、考察することが可能であると思われた。  本研究ではこの方針に基づいて、学習の達成に志向した教示作業の具体的な事例を検討した。その結果、そこでは、教師と生徒が教室によく見られる相互行為の形式に参加しながら、教師はそこで学習すべき行為の基準を様々に生徒に可視化し、またそれを生徒の身体の構造化を伴う形で、生徒に利用可能となっていることの詳細を明らかにすることができた。 文献  五十嵐素子(2016)「『教示』と結びついた『学習の達成』」『概念分析の社会学2』pp.177-194,ナカニシヤ出版 西阪仰(2008)「何の学習か」『分散する身体』, pp.53-118, 勁草書房

報告番号217

日本で就職する外国人留学生の増加要因の考察――「技能移民」の社会的構築の観点から
金沢大学 眞住 優助

コロナ禍による中断があるものの、世界における留学生の移動が近年顕著である(Han, Gulanowski, and Sears 2022)。知識経済のおける革新と成長を担う高技能移民の予備軍との想定のもと、多くの受入れ国では今日、卒業した留学生の確保を目指す試みが行われている。しかし、卒業後ホスト国で実際に就労する留学生の割合は限定的であり、先行研究では、おもに就労を阻害する要因の調査が行われてきた(e.g., Blackmore, Gribble, and Rahimi 2017; Liu-Farrer and Shire 2020)。この状況に鑑みて、日本での近年の留学生の就職者割合の増加は注目に値する。日本学生支援機構によると、2019年度に大学など高等教育機関を卒業した留学生のうち36.9%が日本で就職した。10年前の数値と比べて約19%、5年前と比べても約10%の上昇である。  本報告は、「技能移民」の社会的構築という観点から、この就職者割合の増加要因を考察する。留学生の日本での就職割合を5割に引き上げるとした「日本再興戦略2016」に象徴されるように、日本政府は近年、より積極的な留学生の就職促進を目指している。そして様々な施策を講じている。本報告は、「技能/高技能移民」の社会的構築に関する研究に依拠し(Boucher 2019; Hercog and Sandoz 2018)、就職促進政策を、ホスト国での就職に必要な「スキル」を留学生に付与する、またはそうした「スキル」を(再)定義する国家の実践であるととらえる。そのうえで、日本の各種の施策はどのように、適切な「スキル」をもつ「技能移民」の構築を企図しているのか、またそれはどの程度、実際の就職促進に寄与しているのかを考察する。分析はおもに行政・政策関連文書および公式統計データを用いる。  おもな発見は2点ある。第1に、政府は留学生の就職を、国内の人口変動による人手不足の緩和策ととらえ、就職に要する「スキル」の付与と再定義の双方を行っている。付与には、「留学生就職促進プログラム」等に見られるように、ビジネス日本語や日本型就職活動のノウハウなど、内定を得るための「スキル」を直接的に供与する施策のほか、卒業までに就職できなかった者に対して、卒業後一定期間、就職活動を継続することを認めることで、間接的に「スキル」の醸成を促すものがある。再定義については、在留資格の支給のさいの「ゲートの新設」と「拡大」がある。留学生のほとんどは就職のさい、在留資格「技術・人文知識・国際業務」の支給を受けるが、そのためには予定業務が「専門的能力」を要し、かつ業務と専攻科目に関連があると当局に判断されなければならない。政府は就職促進のため、日本語能力を支給可否の主要な判断基準とする「特定活動(本邦大学卒業者)」など、別の技能基準による在留資格を新たに設けている(「ゲートの新設」)。「技術・人文知識・国際業務」の付与についても、政府はおもに専門学校生を対象として、在留資格の支給基準を緩和している。結果、支給の許可基準として「専門学校修了」それ自体の重要性が増している(「ゲートの拡大」)。第2の主要な発見として、「ゲートの拡大」が留学生の就職促進に果たす役割の重要性を指摘する。他の手段も一定の貢献があるものの、利用可能な統計データから判断する限り、「既存ゲートの拡大」が留学生の就職割合の上昇と最も関連している。

報告番号218

コロナ禍が技能実習制度に及ぼした影響――日越間の「移住インフラ」から捉える
武庫川女子大学 加藤 丈太郎

【1.目的】 技能実習制度は送り出し国と日本の間におけるヒトの循環を前提としていた。しかし、2021年1月21日から2022年3月まで日越間におけるヒトの循環はコロナ禍によって中断した。本研究では2つのリサーチクエスチョンを探求する。1つ目は「コロナ禍によるヒトの循環の中断は日越間にまたがる技能実習制度にどのような影響を及ぼしたのか」である。2つ目は「コロナ禍を機に技能実習生が従来有してきた一時性は変化したのであろうか」である。 【2.方法】 本研究はインタビューを中心とした質的研究方法を用いた。2020年5月17日から2021年12月21日にかけて、ベトナム人技能実習生(経験者)14名、送り出し機関役職員9名、監理団体役職員11名、技能実習生受け入れ企業役職員3名、計37名にインタビューを行った。全てのインタビュー対象者に、論文および発表でデータを用いることに同意を得た。研究倫理は日本社会学会倫理綱領にもとづく研究指針に準拠している。 【3.結果】 移住インフラ(Migration Infrastructure)を分析の枠組みに用い、一時的移住労働者の受け入れ政策(TMP)に技能実習制度を位置づけて分析を進めた。「移住インフラ」とは、移動を促進し、また条件づける、技術・制度・動作主体を連結する概念(Xiang and Lindquist 2014: 124)である。「移住インフラ」の要素は5つ挙げられる。5つとは、1)商業的要素、2)規制的要素、3)技術的要素、4)人道的要素、5)社会的要素である。本研究では、要素間のつながり、あるいは相反に着目して技能実習生(経験者)並びにそのステイクホルダーを捉えた。 コロナ禍により、移住インフラのある要素が欠落する場合、別の要素が補い、技能実習生(経験者)の生活及び就労を成立させていた。たとえば、規制的要素による帰国困難な技能実習生(経験者)への保護が足りていない場合は、それを人道的要素、商業的要素が補っていた。しかし、長期にわたってある要素の欠落が続くと、欠落が別の要素にも波及する。技術的要素(航空機)の不在は、商業的要素(送り出し機関・監理団体)に悪影響を与え、規制的要素に変化(帰国困難者の特定技能人材への変更が容易に)を招いていた。技能実習生の新たな往来が困難な中、技能実習経験者が日本国内で特定技能人材となり、代わりに労働需要を埋めてきたのである。コロナ禍はヒトの往来を不安定にし、循環を困難にしてきた。技能実習経験者がより長期の在留を可能とする特定技能制度に移行することで、技能実習制度が従来有してきた「一時性」はコロナ禍を機に変化した。 【4.結論】 佐藤(2021:303)は「技能実習制度は・・・技能実習法によって日本版TMPへと転換し」、「技能実習と特定技能との接続は批判の的になっているようであるが、日本版TMPにとって、両者の接続は必然である」と述べる。しかし、技能実習制度が技能移転による「国際貢献」を目的とする一方、特定技能制度は「人材を確保することが困難な状況にある産業上の分野において、一定の専門性を有し即戦力となる外国人を受け入れる」ことを目的としており、両制度の目的は大きな隔たりがある。両制度を接続するのであれば、少なくとも技能実習制度の名称をその実態に合わせたものに変え、制度の目的を見直す必要がある。

報告番号219

浜松市の外国人材支援制度の現状と課題――外国人材活躍宣言事業所認定制度を中心に
静岡大学 藤岡 伸明

【1.目的】  日系ブラジル人をはじめとする外国人の集住都市として知られる浜松市では,これまでに数多くの多文化共生施策を打ち出してきた。その1つである外国人材活躍宣言事業所認定制度は,外国人材を雇用し,就業環境の改善や教育・訓練の充実に努めていることが認められる市内事業所を認定し,各種の支援や優遇措置を与える制度である。本報告では,2021年度から実施されている同制度において実施されている施策を検討することにより,同市ならびに日本全体における外国人市民の就業環境改善に向けた取り組みの現状と今後の課題を明らかにすることである。 【2.方法】  浜松市外国人材活躍宣言事業所認定制度に直接的・間接的に関与している団体・組織・個人に対するインタビュー調査によりデータを収集し,分析する。具体的なインタビュー対象(候補)は,制度を管轄する浜松市国際課,同制度の実務を委託されている浜松国際交流協会,そして2021年度に同制度の認定を受けた20の事業所である。浜松市国際課へのインタビューでは,同制度の設置背景・目的・現状・展望について聞き取る。浜松国際交流協会へのインタビューでは,実務面での課題を中心に聞き取る。事業所インタビューでは,外国人材を対象とする就業環境改善策や教育・訓練制度の具体的内容について聞き取る。事業所インタビューでは,当該事業所で雇用されている外国人へのインタビューも可能な限り試みる。 【3.結果】  浜松市によれば,認定事業所に対する主要な支援策・優遇措置は,①事業所のイメージアップ支援,②市の発注業務での優遇措置,③多文化共生分野の助言・サポート,④外国人材等日本語学習支援補助金の上限額引き上げの4点である。①のイメージアップ事業については,3月の認定証授与式に報道各社を招待して報道を促すなど一定の効果が見られる。それ以外の事業の実施状況については今後の調査で明らかにしていく。  すでに認定を受けた20の事業所は,製造,金融,運輸・交通,不動産,福祉,卸売,サービスなど多岐に渡る業種から選ばれている。各企業における上記事業の実施状況と,業種ごとの課題などを今後の調査で明らかにしていく。  同制度の実務を委託されている浜松国際交流協会に対しては,予備調査を兼ねたインフォーマルな聞き取りを実施済みである。多文化共生分野の助言・サポートや日本語講師派遣といった事業を効果的に実施するためには,企業活動と外国人材の両側面について理解できる高度な人材が必要となるため,地域全体でアドバイザーや日本語講師の育成を図る必要があるということがすでに明らかになっている。引き続き聞き取りを実施して,さらなるデータ収集と分析を進めていく。 【4.結論】  外国人市民が地域社会に包摂され,日本人市民と同様の生活水準や生活機会を享受するためには,外国人市民に対する就労面での支援が不可欠であり,就労面での支援を拡充するためには地域で活動する企業・事業所の理解と協力が欠かせない。浜松市の外国人材活躍宣言事業所認定制度のような施策を通じて,外国人市民の社会的包摂という課題に対する企業・事業所の関与を強化することが,今後ますます重要になってくると考えられる。 【5.文献】 浜松市,2020,『浜松市における地域日本語教育の総合的な体制づくり推進事業 地域日本語教育実態調査【調査結果報告書】』

報告番号220

起業とエスニックコミュニティ――上海の日本人現地採用者を事例に
追手門学院大学 松谷 実のり

【1. 目的】  本報告の目的は、2000年代以降の上海における日本人コミュニティの構造と特徴に着目しながら、現地に進出した日本企業に現地採用された日本人が起業に至るプロセスを明らかにすることである。企業の海外展開に伴って形成される日本人コミュニティは企業および駐在員を中核としたホスト社会からの独立性の高い空間となっており、町村(1999)はこれを「日本企業『城下町』」と評する。中国のWTO加盟後に日本企業の進出が相次いだ上海では、このような日本人コミュニティが生じた。  現地採用とは日本企業の海外拠点が直接従業員を雇用することを意味する。企業が従業員を駐在員として派遣する他に、現地で直接日本人を雇用する例が近年増加している。現地採用者の多くは後に帰国するが、一部に現地で起業する事例が見られる。彼らにとって起業という選択肢が持つ意味を確認した上で、日本人コミュニティが起業に必要などのような条件や制約をもたらすかを分析する。 【2. 方法】  2012年9月および2015年3月を中心に、上海において、現地採用としての就労経験があり、その後起業した人物を対象としてインタビュー調査を実施した。主な質問項目は日本での教育歴と職歴、上海に移住した経緯、現地採用としての就労を選択した理由、起業を決断した理由、起業に至る経緯であり、日本在住時からの時系列による語りを得た。 【3. 結果】  第一に、現地採用としての就労の継続や帰国ではなく、起業が選択される理由には、現地採用の昇進・昇給の限界という雇用上の問題があるだけでなく、起業を望ましいとする特有のキャリア意識があった。第二に、起業に必要な支援を得る上では、個人が移住後に築いた日本人および中国人とのネットワークが重要な役割を果たしていた。第三に、起業の業種は、現地在住日本人の生活を支えるものと、日本企業の外注業務とに大別できた。つまり、日本人コミュニティと日本企業そのものが起業の可能性を生み出していた。 【4. 結論】  日系ブラジル人の事例を調査した梶田他(2005)によれば、商業的な移住システムを介した移民の起業においては、移住前に存在したネットワークよりも、移住後に再編されるネットワークがより重要となる。また、ブラジル人コミュニティとそこで生まれるエスニックビジネスを、コミュニティの中核を担うリーダー的な存在(定住した起業家)と日本企業で働く多くの短期滞在者(エスニックビジネスの顧客)との関係の中に位置づけている。本報告の事例においても多くの共通点が見られるが、「日本企業『城下町』」として成立した日本人コミュニティにおいては短期滞在の駐在員が中核となっているという違いや、日本企業との関係の強い業務の存在といった独自の点も見られる。特に後者は、現地採用と起業の連続性、ならびに企業の海外展開と雇用の関係を考える上で重要である。

報告番号221

和平プロセスの社会学に向けて――北アイルランドにおける母親たちの子育て戦略
クイーンズ大学ベルファスト 大森 優美

【1. 目的・背景】 本報告は、武力紛争を経た民族集団が平和的共存に向かう過程において、家族という社会組織が果たしうる役割を、紛争下・紛争後の北アイルランドで子育てを行ってきた女性たちのライフヒストリーを通じて問う試みである。北アイルランドでは、1960年代末より約30年間、カトリック系とプロテスタント系の市民・武装組織間の抗争が続き、約3,500名が犠牲となった。同地は現在でも、深い紛争の禍根を抱える分断社会である。本研究は、紛争後社会において、過去の対立を抱えた集団間が、日常レベルで平和的に共存に向かう過程を「社会的和平プロセス」(Brewer 2010, 2022)と捉える。武力紛争を経た分断社会に生きる人々が、日常生活で慣習的に、しかし主体的に選択する行動・思考様式が、対立する集団間の関係をゆるやかに転換していく過程を、社会学的分析視角で明らかにすることが、本研究の目的である。 【2. 方法】 報告者は、2020年12月から約1年間、北アイルランド・ベルファストにてフィールドワークを行った。内容は、ライフヒストーリー法による半構造化インタビュー28名、フォーカス・グループ・インタビュー2件、女性団体や受刑者家族の支援団体における参与観察・非参与観察である。紛争で特に甚大な影響を受けた地域(都市の労働者階級居住区)で子育てをした経験を持つ母親を中心に、カトリック側とプロテスタント側でほぼ同数の女性の参加を得た。 【3. 結果】【4. 結論】 母親たちの証言において特徴的なのは、紛争のトラウマを次世代に継承してはならないとする社会規範である。調査参加者の多くは、紛争の過去について家庭内で戦略的に沈黙することを選び、その戦略によって子世代の価値観の転換に寄与してきた。上記の規範はジェンダー化された規範であり、多くの母親たちは紛争後社会の求める「良い母親」像に従いつつ、自らもその規範の醸成に加担している。ただし、データからは、過去をめぐる沈黙や、異なるコミュニティの子供との交流促進などによって次世代の価値観の転換に積極的に関与することは、一部の母親たちやその子供たちにとってリスクを伴うことも示された。本研究は、女性たちが子育てを通じて紛争後社会に生きる次世代の価値観の転換に主体的に関与することで、社会的な和平プロセスに陰の貢献を果たしていた側面があることを明らかにした。一方、その貢献を手放しに称賛することは、紛争地において強化されたジェンダー規範を再生産し、「良い母親」像に合致しない女性たちを沈黙させ、周縁化するという暴力性を伴う。社会的な和平プロセスを論じる際、ジェンダーや階級の力学がナショナリズムと交差して構築された社会規範が、紛争後社会において誰を沈黙させるのかを批判的に問うていく必要がある。 参考文献 Brewer, J.D. (2010) Peace processes: a sociological approach, Cambridge: Polity. Brewer, J.D. (2022) An advanced introduction to the sociology of peace processes, Cheltenham: Elgar.

報告番号222

北アイルランドにおける中絶をめぐる言説の再構築――分断社会における社会変革の可能性
愛知県立大学 福岡 千珠

本発表では、近年北アイルランドにおいて中絶が合法化されたことに注目し、分断を抱えた社会において、差異を尊重しつつ生産的な議論することがどのように可能となるのか、またどのように社会的変化をもたらすことができるのかを検討する。 北アイルランドでは長きにわたって一枚岩的な「プロライフ文化」があるとされ、中絶を禁止する古い法律が効力を持ち続けていたが、2019年英国のうち北アイルランドのみでその法律を撤廃することが決められた。しかし、この決定は、北アイルランド議会ではなく英国議会でなされたものであることから、北アイルランド社会において、中絶のような社会を二分する問題に関して議論し、変化をもたらすことが困難であることの証左であると考えられた。実際、紛争を経て社会が分断した「分断社会」であり、差異を尊重する多極共存型民主主義の制度が導入されていることもあり、北アイルランドでは自治議会などの公式な場では中絶に関する議論はほとんど進まなかった。 しかし、J.ドライゼクによれば、分断社会における熟議は、直接の対話・議論でなくとも、「言説」的アプローチをとることによって可能となる。ドライゼクは、分断を形成する「文化」を固定的なものではなく、「言説」によって構成されるものとして捉え、それを変化させてゆくことによって、分断社会においても相互理解や生産的な議論が可能となると主張した。さらに、それらのプロセスは、多様な組織から成るネットワークなど、国家から「ゆるやかに分離した」公共圏で可能となるとした。上記の観点から検討すると、北アイルランド社会においても中絶をめぐる言説は確かに変化が見られ、分断社会に可能な形や場での「熟議」が進み、中絶に対する理解が進んでいたということができる。 上記の議論をふまえ、本発表では北アイルランドにおいて、プロチョイス組織が行った「言説」をめぐる戦略に注目する。その際、以下の点に注目する。一つ目に、北アイルランド社会には伝統的に、「独特のプロライフ文化」があるとする言説が支配的であった点である。二つ目に、上記の「プロライフ文化」言説と厳しい取り締まりの下で、「中絶」が不可視化されていた点である。三つ目に、北アイルランド社会には、中絶よりも重要な「争点」があるとされた点である。それには、宗教や国家帰属の問題が含まれる。本発表は、主に上記の点に注目し、北アイルランドにおける中絶をめぐる言説の再構築のプロセスを明らかにする。 Dryzek, John S., 2006, Deliberative Global Politics: Cambridge: Polity. 田村哲樹, 2017, 『熟議民主主義の困難』ナカニシヤ出版.

報告番号223

ベトナム・ヌン族女性移民の生存戦略に対する国家の影響――中国広西省徳天村を事例として
上智大学大学院 KE YEJIA

研究目的:近年、東南アジアの山岳地帯の少数民族について、近代国家による支配から相対的に自立した存在として論じられてきた[Scott 2009; Schendel & Kalir & Malini 2012;荻野 2017;王 2011]。これに対して本報告は、中国壮族と同系のベトナムヌン族の越境活動を事例として、東南アジアの山岳地帯の少数民族に対して近代国民国家が及ぼす影響を明らかにする。 研究方法:本研究では、東南アジアの山岳地帯に属する広西省崇左市大新県徳天村と川で隔てられいるベトナムカオバン省ダムトィ社を対象として、ベトナムヌン族の女性移民の越境活動について考察する。移住者ベトナムヌン族と中国壮族は言語、文化ともに同系の民族である。両民族はパスポートなしで双方の村を訪問できる。両民族間では祭りへの参加、親戚友人訪問、市場での販売のための越境が日常的に行われている。しかしながら、出稼ぎ移住や結婚移住において、越境は単方向にしかみられない。つまり、ベトナムヌン族の中国徳天村への移動のみが起きており、逆は発生していない。そこで、中越の国境により分けられた中国壮族とベトナムヌン族という同系の民族間の越境活動が、なぜ単方向にのみ存在するのかを本稿の問題意識として、その原因として中国広西省とベトナム北部高原という山岳地帯における国家の影響を考察する。 結果:徳天村が属する広西省は「一帯一路」政策により観光業が発展している。これにより、雇用機会も増大し、徳天村は生活状況が改善されているとともに、隣村から結婚移住してきたベトナム人ヌン族女性も多くなっている。ベトナム人ヌン族の女性は出稼ぎおよび結婚のために徳天村に来る。徳天村は滝などの自然資源に加えてベトナム人と交流できるなどの「ベトナム・エキゾチズム」が観光の際の宣伝として用いられた。このため、現地で多数のベトナム土産物商店が営業しており、ベトナム風のエンタテイーメントも多く見られる。この状況下で、隣村ダムトィ社のベトナム人女性は地元である徳天村の女性より就職しやすくなっている。また、徳天村での出稼ぎにより、地元の徳天村男性と知り合い、結婚するベトナム人ヌン族女性もいる。徳天村で結婚後、夫の家の資源をもとに徳天村でベトナム土産物商店を経営するベトナムヌン族の女性も少なくない。彼女らは同じベトナム土産物商店を経営する中国人経営者に対して優位な立場にあるため(ベトナム人であることから、観光客からの信用を得ることができたり、直接ベトナムから商品を入荷できるため、元手が少なくてすむため、地元の他の店と比べて利益率が高い)、店の経営で成功し、家庭内ないし村内での地位を向上させることもある。その過程で、ベトナムヌン族女性移住者はゲストからホストとなるのである。 結論: 同じルーツという民族的な理由により民族の越境活動は双方向となる。徳天村は中国「一帯一路」政策により、観光業が盛んである。これにより徳天村では生活レベルの改善ともに、就職機会も増えた。このために、ベトナムダムトィ社からヌン族が徳天村に誘致されている。地理的には、中国徳天村とベトナムダムトィ社という地域についてはゾミアなどの研究が主張するような自立した地域に属しているが、実は結婚のための越境活動が単方向である理由を分析すると、国家の影響を強く受けていることが明らかになった。

報告番号224

アメリカ合衆国における生得的市民権と市民の境界 ――反出生地主義の政治に着目して
同志社大学 南川 文里

「生まれ」によって自動的・無条件に市民権・国籍の獲得を保障する生得的市民権(birthright citizenship)は、現代国民国家システムの根幹と言える。現在、ほぼすべての国家が、自国民の親のもとに生まれた子どもに国籍・市民権を与える制度を持っており、アメリカ合衆国を含む約30カ国で、領土内で生まれた外国人の子ども(移民第二世代)に自動的に市民権を保障する出生地主義を採用している。移民国家を自認するアメリカ合衆国では、出生地主義が、国外出身の移民とその子孫を市民社会のメンバーとして包摂するための制度枠組と考えられている(南川 2022)。 しかしながら、20世紀後半以降、生得的市民権と出生地主義による市民権獲得を否定的に捉える傾向が見られる(Schuck and Smith 1985)。たとえば、ドナルド・トランプは、2016年大統領選挙時の公約として「生得的市民権の廃止」を掲げ、市民権から「生まれ」を切り離すことを求めた。「生まれ」と市民権の分離は、現代アメリカの排外主義運動の主張の一つである。その典型的な言説として、米墨国境から入国して米国内に滞在する非正規移民の子どもへの市民権付与の是非をめぐる「アンカー・ベビー」と、アメリカ国内での出産を目的として短期滞在する外国人に焦点を当てた「バース・ツーリズム」が挙げられる(Chavez 2017)。この二つの事例は、アメリカ市民権の危機として、メディアや政治家によって過度に誇張されているが、排外主義運動は、「アメリカ市民」としての資質や忠誠心を欠く者にも市民権を付与する出生地主義の矛盾の象徴として、頻繁に言及している。そして、出生地主義を廃止し、実質的な市民権政策の採用を求めている。 本報告では、いずれの言説も「出産」という事象を対象としていることに注目し、生得的市民権論争が、いかに「アメリカ市民」の資格を、人種/階級/ジェンダー/セクシュアリティが交差する選別性によって再定義してきたかを議論する。そして、生得的市民権をめぐる論争が、いかに市民権の現代的条件を組み換え、入植者社会(settler society)としての制度的基盤を揺るがしているかを示す。 Chavez, Leo R. 2017. Anchor Babies and the Challenge of Birthright Citizenship. Stanford UP. 南川文里. 2022. 『アメリカ多文化社会論[新版]:「多からなる一」の系譜と現在』法律文化社. Schuck, Peter H. and Rogers M. Smith. 1985. Citizenship Without Consent: Illegal Aliens in the American Polity. Yale UP.

報告番号225

宗教的人種化とナショナルな文脈:2000年代以降のフランスにおけるムスリム差別研究と論争をめぐって
名古屋大学 鶴巻 泉子

1. 問題の所在 9.11以後テロが頻発する西欧において、「ムスリム移民」のテーマは多くの学術研究を生み出した。しかし西欧の他者として集合的に異化される「ムスリム」とは、それぞれの社会において文脈化された具体的存在である。本報告はフランスで2000年代以降急激に増加したムスリム差別研究を整理し、これらの研究をめぐって起きた社会的論争が持つ意味をフランスのナショナルな記憶に着目して議論する。そして最後に「ムスリム移民」というカテゴリー化がもたらす影響についても考察を加える。 2. 差別研究の増加とその背景 2000年代以降のフランスの差別研究は、移民やその子孫の統合が社会問題化する国内の状況と、差別解消に向けてEUの取組が積極化する1990年代以降の動きとを原動力として加速した。これらの研究は、その手法と問題意識に着目すると1「計量的アプローチ」、2「経験アプローチ」、3「差別と共和国思想アプローチ」の3つに分類できる。1は主に人口学者や一部の社会学者に担われ、米国の実証主義的な差別研究、特に計量的手法に影響を受け、エスニックな出自、宗教、居住地区、ジェンダー等の変数と、警察の職務質問や住居、労働等の領域における不平等な処遇との連関を分析する。2は社会学研究が主であり人々が彼ら自身の差別経験をどのように語り意味づけるかに注目する。3は歴史学や政治学、政治思想研究に担われ、フランス共和国主義そのものに潜むとされるレイシズムや植民地主義の遺産を、歴史的アプローチを通じて検証する。 3. 差別研究をめぐる論争 これらの研究は主に米国・英国で発展した差別研究に影響を受け、一部では「インターセクショナリティ」や「イスラモフォビア」概念を導入して論じ、特に宗教的差別がクローズアップされた。それがフランス国内の状況を無視するものとして批判を招き、特に後者の概念は政治家やオピニオン・リーダーを巻き込む論争を引き起こした。宗教を一つの差別要因と見なすこと自体が問題視されている。またこれらの研究は差別への国家の関与・責任を示唆するという特徴があり、それが論争をさらに加熱させた。 4. 考察と結論:「ムスリム移民」とネイションの記憶の再構成 フランスでは、イスラムをめぐる問題は植民地主義の歴史や共和国主義のダブルバインド的性質を直接に想起させ、長らくタブーとなってきた。さらにライシテの原則は宗教を理由とした差別についての議論を阻んできた。ヨーロッパでの横並びの「イスラム差別」の問題化によって、国家のタブーや欠落を抱えたネイションの記憶とに正面から向き合う必要が生じている。対して英国やスペインでは宗教的差別に関する議論はタブーではなく、また移民とイスラム問題が結びつくフランスや英国に対して、スペインではイスラムは(アンダルシアや北アフリカ飛び地など)“土着の”問題でもあった。しかし「ムスリム移民」問題のヨーロッパ化により、フランスと同様に特定の移住者をその代表として扱いつつ国家とヨーロッパ双方にとっての他者として表象する傾向が現れる。 他方では、フランスの差別研究はフランス独自の文脈の抽出にこだわり、論争の効果と相まって、議論がやや閉鎖的になる傾向がある。差別研究におけるカテゴリー化の問題と、文脈化の問題との、複雑な関係が指摘される。

報告番号226

タイ北部チェンマイのストリートユースから見える日常と非日常――1997年~2000年ごろの「家出」 を中心に
関西学院大学大学院 長谷川 愛

“【1. 目的】 この報告の目的は、ストリートユースから見える日常と非日常の世界、および双方との曖昧な境界に 生きる彼らの生活実践を、タイ社会のマジョリティとの関係性のもと、ストリートユース当事者の視 点から社会学的に調査し描写・分析することにある。今大会発表においては彼らが家出を選択するま での家族との複雑な関係性や、どのような思いで家出を選択し、どのようなルートでストリート世界 にたどり着き参入し、仲間入りし、いつ自身がストリートユースだと自己認識を持つに至るのか、彼 らのストリート世界への参入過程を明らかにし考察する。
【2. 方法】 1997年からタイ国チェンマイのストリートチルドレンを支援するNGO活動に従事しつつ、25年間にわた りストリートユースとの関係性を継続してインサイダーに近づいた立場から彼らのライフヒストリー およびライフストーリーを回顧的に分析する。特に当事者視点に光をあて、彼らを取巻く社会との関 係性を主として考察していくにあたり、今回は90年代後半にタイ北部の実家から「家出」をして地方 都市チェンマイ市のストリート世界に参入した男子1名(当時17歳)と、女子1名(当時21歳)のいず れもタイ国籍を有するタイ人ストリートユースの事例を用いる。
【3. 結果】 ストリートユースは家族との多様な関係性の中で育ち、当事者や家族の良かれと選択した日常的実践 の連なりのなかで、だれもが気づかぬうち徐々に関係性の脆弱化へと移行していく。そして、本格的 な「家出」=「ストリート世界」への参入を果たす前に、思春期に入ると「仕事をしに行く」との一 見自発的な選択から「プチ家出」または「プレ家出」とも呼べる短期的・単発的家出を経験する、 「ストリート世界」と「一般社会」とを行き来する時間幅がある。その時間幅に身を置くとき、彼ら はストリート世界へとつながる関係性を構築して「生き延び戦略」を蓄積することで、ストリート世 界にソフトランディング的に参入を果たしていく実態が明らかとなった。【4. 結論】 ストリートユースの意思、選択、彼らを取巻く家族との多様な関係性を、当事者視点から描き捉える ことで、「ストリート世界」と「一般社会」との両義性や交差性を有する曖昧な境界部分における彼 らの動態が明らかとなった。それは、「社会復帰」を自明とする問題関心にもとづく開発援助論の二 分法概念に立脚した既存研究では見逃されてきた、二つの世界の境界を行き来するストリートユース の無数の日常・非日常という「生」のリアリティがいかに私たち「一般社会」の日常の地続きにある のか、そして従来ストリートチルドレン/ユース現象の根源とされる「貧困」の社会構造や出自のカ テゴリー化に関わらず、元々家族や社会の一員として「姿の見える存在」であった「ゼロ地点」か ら、だれもが社会の中で不可視かつ周縁化される存在になり得るというプロセスである。”

報告番号227

新幹線はいかなるものとして語られてきたか――東北新幹線を扱う全国紙の分析から
早稲田大学大学院 増田 拓弥

【1.目的】本研究の目的は、1970年代から建設が開始された東北新幹線を扱った新聞の記事を取り上げることで、新聞という媒体において東北新幹線がどのように語られ、意味づけられてきたのかを解明することを目的とする。新幹線に関する社会科学的な研究は、交通学や都市社会学といった工学的な視点(日本都市学会 1985, 2016)、そして受益圏と受苦圏という概念を用いながら、新幹線公害を事例として公害に対する運動の経過を記述した社会学的な研究(船橋ほか 1985, 1988)などが挙げられる。新幹線はその建設の是非や安全性、公害、経済発展、都市と地方、さらに近年には既存の在来線の存廃問題のように、様々な視点から語られ、そして意味づけられてきた。しかしこれまでこうした交通メディアは、学問分野においてある「機能」と「効果」をもつ、都市や地域の変化を説明する説明変数の一つとして扱われてきた。しかし、前述したような多様な文脈において、こうした「機能」や「効果」に着目し措定することは、実際にそれらを用いる人々の多様な営みを見逃してしまう可能性がある。これ踏まえ、実際の交通メディアの一つである新幹線が、認識主体やマスメディアによってどのように意味づけられ、表象されてきたのかを調査することで、近代日本において移動することのあり方の多様性と多層性を考察する一つの道程としたい。【2.方法】全国新幹線鉄道整備法が策定された1970年から東北新幹線が上野-盛岡間で開通した1985年までの、朝日新聞と読売新聞の全国版における東北新幹線やそれに関連する新幹線の記事の内容を分析した。【3.結果】東北新幹線に関する新聞記事として、(1)政府や国会によって策定された新幹線計画(2)国鉄の路線選定方法や騒音への懸念などの理由による反対運動(3)東北地方の政治家や商工業者の反対運動や国鉄に対して早期開業を求める動き(4)大宮暫定開業に対する反応(5)事前予測に反する利用客の多さ(6)国鉄との協議による反対者の和解(7)上野開業に対する反応、が析出された。【4.結論】最初に開業した新幹線である東海道新幹線を念頭に策定された全国新幹線網計画は、その条文にあるように開発色が強く、それまで鉄道などの交通・速度体系の遅れから「後進性が目立って」いた東北地方は、新幹線の開通を、貧困や過疎から「抜け出る道」として意味づけた。一方で首都圏においては、都知事の発言などから開発への懐疑的な見方が強く見られ、新幹線に対する肯定的な意味づけが行われず、名古屋新幹線訴訟などを念頭に政府対市民という対決のレトリックを用いながら、新幹線開通を是とする政府や国鉄に反対運動を展開した。これらの一次的な言説や行動を新聞という二次的な主体が取り上げることで、さらなる言説や行動、そして「東北新幹線」に対する意味付けがなされていく。貨物輸送も行う従来の鉄道とは異なり乗客のみを輸送する新幹線であっても、鉄道に多く見られる開発の意味付けが有効に働き、反対運動が一定程度成果を挙げながらも、新幹線を人が高速に移動できる手段として肯定的に捉えてきたことが判明した。この結論から、現在に至るまで建設が進められている新幹線が、どのような文脈で建設されているのかを考える、一つの視点を提供する。

報告番号228

近代名古屋における土地区画整理事業と商工業地域の形成
Waseda University 武田 尚子

1 関心の所在  本報告は、名古屋市中心部の中村区を調査対象地とし、歴史的アーカイブを用いて、近代における土地区画整理事業と商工業地域形成の関連を明らかにする。現代の都市再開発では、再開発地域に隣接して「再都市化遷移地帯」が併存していることがある。「再都市化遷移地帯」の内部には、再開発が進行している「遷移エリア」と、条件不良で再開発の対象にならない「非遷移エリア」がある。社会的には条件不良が多重的に蓄積している「非遷移エリア」が問題となる。  「非遷移エリア」形成の要因を歴史的に探ると、「戦前都市化」段階で、都市中心部に近代工業地域が形成された時期に、その隣接部の農村地域がスプロール開発され、基盤不備のまま、低湿地などの「条件不良」地域が狭小区画に細分され、低家賃の住宅地域になった事例が散見される。本報告は名古屋中心部における「再都市化遷移地帯」に着目し、「遷移エリア」と「非遷移エリア」の形成プロセスについて歴史社会学的視点から考察する。 2 調査地域の概要  名古屋市中村区はJR名古屋駅の西側地域である。現在は都心部商業地域であり、都市再開発された「ささしまライブ24地区」がある。中村区にはリニア中央新幹線駅の西側地上出口が設置予定で、現在、リニア駅建設が進んでおり、駅周辺の再整備が行われている。中村区はリニア開業による都市再開発で注目を集めている地域の一つである。その一方、中村区は名古屋市のなかでも一貫して生活保護率の高い区であった。低所得層が集積する地域社会構造が近現代に形成されたと推測される。  地形的には庄内川から数本の小河川が貫流して南部に流れ(現在は暗渠化)、低湿地と微高地である自然堤防が併存しているという特徴があった。微高地には中世の荘園にルーツをもつ自然村が形成され、小河川流域の低湿地は水田として土地利用されていた。近代都市化の時期、このような「微高地の自然村」と「低湿地の水田」が商工地域として開発されていった。 3 中村区における戦前期土地区画整理事業と影響  名古屋市の戦前期都市化の特徴は、東京市、大阪市よりも都市化の時期が遅かったことで、東京・大阪を参考にしながら都市基盤整備を進めることができたことである。具体的には大正8年(1919)に東京市区改正条例の準用指定を受け、都市計画事業の実施が可能になった。のち中村区となった地域では名望家層による土地買収、開発計画が立案され、土地区画整理事業が実施された。このとき区画整理の対象になったのは「低湿地の水田」である。つまりスプロール開発ではなかった。近代都市化の時代に条件不良地域の区画整理が実施された。一方、居住条件が良好な「微高地」は従来から自然村が立地していたため、区画整理の対象にはならなかった。結果的に、この時期に区画整理が実施されなかった「旧自然村」地域の都市基盤整備が遅れ、高度経済成長期に生活保護率の高い地域になった。

報告番号229

1960・70年代日本における「ふるさと」表象とその構築プロセス――NHK総合「ふるさとの歌まつり」を手がかりとして
法政大学 武田 俊輔

【1.目的】  本報告では1960〜70年代の日本において、「ふるさと」という言葉を通じて、地方がどのように表象されたのか、またその表象の構築プロセスがいかなるものであったのかについて明らかにするものである。そのための手がかりとして、1966年〜1974年にかけてNHK総合において主に生放送で放映され、看板アナウンサーの宮田輝氏がゲスト歌手と共に当地の郷土芸能や生活文化を紹介し、高い視聴率を誇った芸能番組「ふるさとの歌まつり」を手がかりとする。 番組の表象分析だけでなく、放映に際してNHKの各地方局による情報収集や、地域社会の諸アクターが番組の制作プロセスへの関与していったのかを論じる。表象分析を中心とした、従来のこうしたテーマをめぐる歴史社会学的な研究においては、こうした表象を構築する側の内部の力学について、また地域社会のアクターがメディアのまなざしに対し自らをどう提示しようとしたかは十分分析されてこなかった。本報告の焦点はそこにある。 【2.方法】 まず本報告の基礎資料となる当時の番組については2020年度第1回NHKアーカイブス学術トライアル研究への採択により、現存する35本について閲覧・分析を行った。その上でNHKにおいて宮田氏の秘書的な役割を務めた関係者、およびNHK芸能局演芸部や拠点局で番組の制作にたずさわった元ディレクターに、番組の制作過程の聞き取りを行った。 さらに宮田氏のご遺族の協力を得て、氏のパートナーであった恵美氏が整理して現存している番組制作資料を閲覧した。資料には台本、取材メモや放送中の手控、NHK地方局や自治体・保存団体・住民・出郷者から送られてきたハガキ・手紙、番組制作のスケジュールや会場図、出郷者・視聴者からの手紙、地方紙の番組に関する記事や自治体の広報等が含まれている。これらを通して、1で示した分析を行う。 【3.結果】【4.結論】  この番組の制作意図には東京への出郷者に対して「ふるさと」の側の活力やお国自慢的なものを示すという面があったが、その一方で東京からその地域を訪れる情報番組としての側面も意識されていた。客席も含めた会場全体をステージに見立て、歌手や地元出演者、客席と宮田との掛け合いと一体感の中で「ふるさと」が紹介される。 制作過程においては地方局の各地の状況をふまえつつ事前に放映の年間計画が立てられ、放映予定日から約3ヶ月程度前から、地方ブロックの中央局および県域の地方局によって事前の取材が行われる。これはNHKによる受信サービス活動や営業活動とも連動しており、また各地方局での人脈も活用され、自治体・教育委員会、さらに出郷者も含めた個人からのからの出演にふさわしい郷土芸能および保存団体、郷土史家についての情報提供があり、商工会による協力が見られる事例もある。こうした地域社会の諸アクターの協力が不可欠であり、また地元紙においても大々的にとりあげられた。時には学校の校庭を用いて実際の祭礼さながらに大がかりに山車祭りが行われるなど、地域社会における大きなスペクタクルとなっていた。そうした諸アクターの関係の中で番組が生み出されるプロセスを本報告では具体的に明らかにしていく。

報告番号230

1960年代前半の低所得層の経済的離陸――1961年神奈川県民生基礎調査の再分析から
上智大学 相澤 真一

1.はじめに  本論では,1961年に神奈川県で行われた「神奈川県民生基礎調査」を再分析することにより,高度経済成長前期に低収入層として専門家たちに注目された「ボーダー・ライン層」がいかなる貧困層だったのかに注目する.特に,本報告では,ボーダーライン層の状況について,クロス表と対応分析から注目してきた相澤(2020)を踏まえて,潜在クラス分析を用いて,経済的離陸可能性において,どのような形でボーダーライン層と名付けられた低所得世帯が分類できるのかに注目する. 2.神奈川県民生基礎調査(「ボーダー・ライン」層調査)の概要  神奈川県民生基礎調査(「ボーダー・ライン」層調査)は,1961年6月に神奈川県全県にて実施されている.この調査では,「生活保護基準×1.1の世帯収入の世帯を下限」とし,「市町村民税の均等割だけを負担する世帯の収入水準の上限」とする操作的定義を行って,低所得世帯を推定している.当時の神奈川県内では,総世帯814,995世帯のうち,99,117世帯がこの層に相当すると推計された.ここから地域別に14,692世帯を抽出し,12,551世帯分から回収し,11,395世帯が分析対象となっている.現存する調査票の数は後者の数にほぼ一致する.以上の数値から,集計可能な回収率を割り出すと81.1%となる.  本調査票原票について,東京大学社会科学研究所図書室の特別利用許可を得て,現存する個票すべての撮影を行いディジタル・データにした上で,神奈川県のほぼ東半分にあたる横浜市,川崎市,横須賀市,逗子市,大和市,三浦市の6市6,152票を入力した .本報告の分析では,この入力データを太田昌志(追手門学院大学)などの協力によってデータクリーニングされて,東京大学社会科学研究所のSSJデータアーカイブから公開されたデータ(調査番号1331)を用いる. 3.これまでの分析結果  まず,調査票で分類されている世帯類型を用いたクロス集計表によって,どの世帯類型においても住宅資金と職業に困っていた.しかし,その低収入による生活の困難さは若干世帯類型によって異なることが見えてきた.まず,多子世帯は本サンプルのなかで比較すれば収入は高く,また,教育費よりも住居の状況の方が問題と見ることができる.母子世帯は,現時点での稼得能力が多子世帯,完全5人世帯よりも低い点に大きな違いがあり,比較すると被保護経験が高く,また修学資金に困っている割合も高かった.長期療養者を抱えた長期療養世帯は医療費に困っており,非活動的な高齢世帯の貧困は,雇用されない世帯が収入面で追いつくことができなくなってきたことから問題が浮き彫りになりつつあった. 4.今回の分析方針と見通し  これまでの本データの分析では,そもそも分類自体に若干の問題を含んだ質問紙内での世帯類型分類を用いた上で分析を行っていた.むしろ本収集データから帰納的に上記の分類が妥当かを見出すために,潜在クラス分析を用いた検討を予定している.この分析結果を中心に当日の分析を行う予定である. [謝辞]  二次分析にあたり,東京大学社会科学研究所附属社会調査・データアーカイブ研究センターSSJデータアーカイブから「ボーダー・ライン層調査」の個票データの提供を受けました.加えて,「労働調査資料」の特別利用をはじめ,東京大学社会科学研究所に全面的にご支援を頂いていることに,記して感謝申し上げます.

報告番号231

「貧困化の過程」の語り方・描き方――『失業対策事業就労者の職業歴・生活歴調査』をもとに
筑波大学 森 直人

【1. 目的】 通常の社会階層・社会移動研究(以下SSM研究)では、①人びとの社会移動の各時点における社会的地位への分類図式や、②ある時点におけるどのような資源配分の違いが次の時点のどの資源配分の差異を決定するかといった因果図式のモデル設定を、あらかじめ研究者が用意して出発する。こうしたSSM研究の手続きは、しばしば次のような問題設定をもたらす。研究が明らかにする「実態」に対して、人びとの意識や考え、言論のほうが「歪んで」しまうのはなぜかといった問題設定である(森 2008など)。  本研究ではそれとは違ったSSMへのアプローチを試みる。人びとはどのようなカテゴリーや方法を用いて自らや互いを不平等な社会の構成要素として分類し、位置づけたり、もろもろの出来事を関連づけたりしながら、ある人生の軌跡を理解可能なものとして描き出すのか。その一連の活動の連鎖のなかにみられる表現形式の使用法に注目した考察を行う。 【2. 方法】 『失業対策事業就労者の職業歴・生活歴調査』(東京大学社会科学研究所調査報告第16集,1981年2月)を資料として検討する。1955年に東京都飯田橋の公共職業安定所に登録された日雇労働者のうち、失業対策事業適格者(以下、失対日雇)を対象として実施された聞き取り調査である。調査後に成果がまとめられることはなく、調査資料だけ残っていたものが1981年になって調査報告書として刊行された。男性38名、女性17名の計55名に対して職業歴・家族歴・住居歴を中心に生活歴を聞き取った記録が「面接記録(ケース・レコード)」と「ケース・レコード整理票」として収録されている。失対日雇という現状にいたるまでの生活歴を尋ね、応答し、それをノートに書きつけ、集めたノートを一定のフォーマットのもとに文章化してケース・レコードを作成し、さらにそれを整理票にまで落とし込む、という一連の活動の連鎖のなかで、どのように「貧困化の過程」が描き出されることになるのかを検討する。 【3. 結果】 1つのケースについて、ケース・レコードとケース・レコード整理票という2種類の資料がセットで作成される。ケース・レコードでは、聞き手がよくわからなかった語りはそのままの言葉をもとに記述される一方で、わかってしまう叙述は短く定式化する単語に置き換えられる。同じことがケース・レコードから整理票への変換においても行われる。また、他の調査者へのインデクスとしての整理票の記述は当然ケース・レコードよりも簡略化されるわけだが、ケース・レコードの記述のうち整理票に何を残し・何を残さないのかという観点で見ると、系統的に整理票から脱落するのは、経済的困窮につながった原因として語られる、自分の人生にかかわりをもった個人や出来事のエピソードであることがわかる。 【4. 結論】 以上の分析結果から、整理票の記述を起点にケース・レコードの記述を読むという実践が、ケースの個別性と「貧困化の過程」の一般性とを区別するように組織化されることがわかる。貧困化の原因帰属の異なる組み立て方の一端を明らかにすることができた。 ※本研究は科学研究費補助金20K02556の助成を受けている。 【文献】森直人,2008,「「総中流の思想」とは何だったのか――「中」意識の原点をさぐる」東浩紀・北田暁大編『思想地図』2: 233-270.

Back >>